Sunteți pe pagina 1din 157

PROBLÈMES DE

THERMODYNAMIQUE (L3)

et leurs corrigés

Christian Carimalo
TD1

I. Formes différentielles, facteur intégrant


1◦ ) Soit la forme différentielle à deux variables x et y :

D = X(x, y) dx + Y (x, y) dy (1)

Rappeler la condition nécessaire et suffisante pour que (1) soit la différentielle d’une fonction
Z = Z(x, y). On dit dans ce cas que D est une différentielle totale dZ. Indiquer alors
comment on peut obtenir la fonction Z(x, y).
2◦ ) Les formes différentielles suivantes sont-elles des différentielles totales ? Si oui, déterminer
la fonction correspondante, à une constante près.

y 2 dx + x2 dy a b by − ax
D1 = 2
; D2 = dx− dy + dz ; D3 = dx+xdy ; D4 = yzdx+dy +dz
(x + y) z z z2
RT RT
D5 = Cv dT + dV ; D6 = − dV ; D7 = D5 + D6
V V
Dans ces expressions, a, b, Cv et R sont des constantes.
3◦ ) Pour les formes qui ne sont pas des différentielles totales, peut-on leur trouver un facteur
intégrant ?

II. Coefficients thermoélastiques


Les trois variables thermodynamiques pression P , volume V et température T d’un système
binaire sont liées par une équation d’état que l’on peut écrire sous la forme

F (P, V, T ) = 0

L’une quelconque de ces trois variables peut donc être considérée comme une fonction des
deux autres, ces dernières devant alors être considérées comme indépendantes. Si l’on se
donne V et T , P est alors déterminé : P = P (V, T ) ; si P et T sont données, V est
déterminé : V = V (P, T ) ; si P et V sont données c’est T qui est déterminé : T = T (P, V ).
1◦ ) Etablir les relations

∂P 1 ∂P ∂V ∂T
       
= , = −1
∂V

∂V T ∂V T ∂T P ∂P V
∂P T

2◦ ) On définit les coefficients thermoélastiques

1 ∂V 1 ∂P 1 ∂V
     
α= , β= , χT = −
V ∂T P P ∂T V V ∂P T

Christian Carimalo 2 TD de Thermodynamique - L3


a) Nommer ces coefficients. Préciser leurs caractères extensif ou intensif.
b) Montrer que α = P βχT .
c) Déterminer α, β et χT pour
• une mole de gaz parfait d’équation d’état P V = RT ;
• un kilogramme de gaz parfait ;
a
 
• une mole de gaz de Van der Waals d’équation d’état P+ (V − b) = RT .
V2

III. Détermination d’une équation d’état à partir des coefficients thermoélastiques


A/ Montrer que pour un système binaire, la connaissance de deux coefficients thermoélastiques
permet de déterminer l’équation d’état. On donne

1 3a 1 a
   
α= 1+ , χT = 1+
T V T2 P V T2

V étant le volume molaire et a une constante.


a) Quelle est la dimension de la constante a ?
b) Vérifier que les expressions des deux coefficients sont compatibles avec les propriétés des
dérivées partielles d’une fonction.
c) Déterminer l’équation d’état correspondante. On imposera à l’équation trouvée d’avoir
comme limite l’équation d’état des gaz parfaits pour les grands volumes.
d) Faire le même exercice si l’on se donne

R(V + a0 )2 R(V + a0 )
α= , β =
P V 2 (V + 2a0 ) PV 2

V étant le volume molaire, R la constante des gaz parfaits et a0 une constante.


B/ Des mesures des coefficients α et χT de l’eau pour des températures entre 0◦ C et 10◦ C
et pour des pressions inférieures à 20 atm. ont donné les résultats suivants

α(T, P ) = 2A(T − T0 ) + B(P − P0 ) , χT (T0 , P ) = χ0

où A, B, T0 et P0 sont des constantes positives, T la température et P la pression ; χ0 est


une constante positive indépendante de P .
1◦ ) Déterminer l’expression de χT (T, P ).
2◦ ) Déterminer l’équation d’état de l’eau dans le domaine considéré, sachant que pour P =
P0 et T = T0 , le volume massique prend la valeur v0 .
C/ Démontrer qu’un fluide pour lequel

R RT RT
α= , χT = , et lim V (T, P ) =
RT + bP P (RT + bP ) b→0 P

RT + bP
a pour équation d’état V (T, P ) = (V est le volume molaire).
P

Christian Carimalo 3 TD de Thermodynamique - L3


IV. Echelles de températures
Dans le domaine de températures comprises entre 0◦ C et 816◦ C, une résistance de platine
varie en fonction de la température Celsius t selon la loi

R(t) t t
  
= 1+µ t−ν −1
R0 100 100

où R(t) est la valeur de la résistance à t◦ C, R0 sa valeur pour t = 0◦ C ; les paramètres µ


et ν ont pour valeurs respectives µ = 3, 92 10−3 , ν = 1, 49.
Dans l’intervalle [0◦ C,100◦ C], on veut utiliser cette résistance comme grandeur thermométrique
pour définir une échelle centésimale de température θ = A R(t) + B telle que θ = 0 pour
t = 0◦ C (R = R0 ) et θ = 100 pour t = 100◦ C (R = R100 ).
1◦ ) Montrer que l’on définit ainsi une échelle de température différente de l’échelle Celsius.
2◦ ) Déterminer l’écart θ−t en fonction de t. Pour quelles valeurs de t l’écart est-il maximum ?
3◦ ) Jusqu’à quelle température t au-delà de 100◦ C peut-on utiliser l’échelle θ de telle sorte
θ−t
que l’écart relatif | | reste inférieur à 1% ?
t

Christian Carimalo 4 TD de Thermodynamique - L3


Corrigé TD1

I. Formes différentielles, facteur intégrant


∂X ∂Y
1◦ ) et 2◦ ) L’égalité = est la condition nécessaire et suffisante pour qu’une forme
∂y ∂x
différentielle D à deux variables x et y soit la différentielle d’une fonction Z = Z(x, y). On
vérifie que seules D1 et D7 la satisfont. L’intégration de D7 qui ne fait intervenir que la seule
variable T est immédiate. On trouve Z7 = Cv T + constante. Le cas de D1 est un peu plus
∂Z1 y2
compliqué. On doit avoir = X1 = , d’où
∂x (x + y)2
y2
Z1 (x, y) = − + ϕ(y)
x+y

∂Z1 x2
où ϕ est fonction de la seule variable y. Comme on doit avoir = Y1 = =
∂y (x + y)2
2y y2
− + + ϕ0 (y), on trouve ϕ0 (y) = 1 et par suite ϕ(y) = y + constante. On a
x+y (x + y)2
xy
finalement Z1 (x, y) = + constante.
x+y
Dans le cas de formes différentielles D = Xdx + Y dy + Zdz à trois variables x, y et z, il
faut et il suffit que soit vérifiées les trois conditions 1

∂Z ∂Y ∂X ∂Z ∂Y ∂X
Rx = − = 0, Ry = − = 0, Rz = − =0
∂y ∂z ∂z ∂x ∂x ∂y

ax − by
Seule D2 les satisfait et l’on trouve F2 (x, y, z) = + constante.
z

3◦ ) Le facteur intégrant d’une forme différentielle D qui n’est pas la différentielle d’une fonc-
tion est une fonction µ telle que le produit µ D soit la différentielle d’une fonction 2 . Une
forme D à deux variables possède toujours des facteurs intégrants. Pour trouver leur forme
générale, on procède de la façon suivante en prenant D3 comme exemple. On résout tout
d’abord l’équation D3 = 0, ce qui donne ici la relation u3 (x, y) = ln x + y = constante.
Considérant une fonction arbitraire Φ3 (u3 ), un facteur intégrant µ3 de D3 satisfait les
∂u3 ∂u3
équations µ3 X3 = Φ03 (u3 ) et µ3 Y3 = Φ03 (u3 ) , ce qui conduit ici à
∂x ∂y
1
µ3 (x, y) = Φ03 (ln x + y)
x

On vérifie notamment que 1/x est l’un des facteurs intégrants de D3 , conduisant à la fonction
F3 (x, y) = y + ln x + constante telle que dF3 = D3 /x. Mais (ln x + y)/x est aussi un facteur
1 1
intégrant, conduisant cette fois à la fonction F3 (x, y) = ln2 x + y 2 + y ln x + constante.
2 2
1. Qui signifient que les composantes Rx , Ry et Rz du rotationnel du champ de vecteurs de compo-
santes X, Y et Z doivent être nulles.
2. Voir J. Bass : “Cours de Mathématiques”, Tome I, p 577, Masson et Cie ed., 1968.

Christian Carimalo 5 TD de Thermodynamique - L3


Par le même procédé, on trouve

1 0 V 0
µ4 (T, V ) = Φ (Cv ln T + R ln V ) , µ6 (T, V ) = Φ (V )
T 4 T 6

Le cas des formes différentielles à trois variables D = Xdx + Y dy + Zdz qui ne sont pas
des différentielles de fonctions est plus compliqué. Elles ne possèdent de facteurs intégrants
que si et seulement si le champ de vecteurs de composantes X, Y et Z est orthogonal à
son rotationnel. On vérifie que D4 ne satisfait pas cette condition et ne possède donc aucun
facteur intégrant.

xxxxxxxxxxxxxxxxxxxxxxxxxxxx

II. Coefficients thermoélastiques


∂F
1◦ ) Posant FP0 = , etc, et écrivant dF = FP0 dP + FV0 dV + FT0 dT = 0, on en déduit
∂P
1 1
dP = − 0 FV0 dV + FT0 dT , dV = − 0 FT0 dT + FP0 dP
 
FP FV
1
FP0 dP + FV0 dV

dT = −
FT0

d’où

∂P FV0 ∂P FT0
   
=− , =−
∂V T FP0 ∂T V FP0
∂V F0 ∂V FP0
   
= − T0 , =−
∂T P FV ∂P T FV0
∂T F0 ∂T FV0
   
= − P0 , =−
∂P V FT ∂V P FT0

On en déduit aisément les relations mentionnées.


2◦ ) a) α, β et χT sont de caractère intensif ; α, homogène à l’inverse d’une température,
est le coefficient de dilatation isobare ; β, homogène à l’inverse d’une température, est le
coefficient d’augmentation de pression isochore ; χT , homogène à l’inverse d’une pression,
est le coefficient de compressibilité isotherme.
b) La relation α = P βχT se déduit aisément des relations établies au 1◦ ).
1 1
c) Pour le gaz parfait : α = β = et χT = et ce, pour une masse quelconque de gaz.
T P
Pour une mole de gaz de Van der Waals :

1 b 1 1 a
   
α= 1− , β= 1+ ,
T V 2a b T PV 2
1− (1 − )2
RT V V
b
1 1−
χT = V
P a 2ab
1− 2
+
PV PV 3

Christian Carimalo 6 TD de Thermodynamique - L3


A
 noter
 que α et χT ont
 été calculés de façon astucieuse en tenant compte des relations
∂T 1 ∂P 1
= et =− .
∂V P Vα ∂V T V χT

xxxxxxxxxxxxxxxxxxxxxxxxxxxx

III. Détermination d’une équation d’état à partir des coefficients


thermoélastiques
A/ Recherchons par exemple l’équation d’état d’un fluide sous la forme V = F (P, T ).
Celle-ci s’obtient en principe en intégrant la différentielle

∂V ∂V
   
dV = dT + dP = V αdT − V χT dP
∂T P ∂P T

ou encore en intégrant la différentielle d ln V = αdT − χT dP . On voit ainsi que la donnée


des deux coefficients α et χT permet de déterminer l’équation d’état.
a) Dans l’exemple proposé, la dimension de la constante a est celle du produit d’un volume
et du carré d’une température.
b)
 L’intégration
 de la différentielle d ln V ne peut être mené à son terme que si l’on a bien
∂α ∂χT

=− . Pour l’exemple proposé, on a
∂P T ∂T P
∂α 3a ∂V 3a 3a a
     
=− 2 3 = χ T = 1 +
∂P T V T ∂P T V T3 PV T3 V T2
et

∂χT a 2 1 3a a
     
− = 3
+ 2
α = 3
1+
∂T P P VT VT PV T V T2

L’égalité est donc bien vérifiée dans ce cas.


c) Choisissons ici comme point de départ l’intégration de χT . On doit résoudre l’équation

∂V V a
   
=− 1+
∂P T P V T2
qui peut être récrite sous la forme

∂V a ∂
   
V +P =− 2 = (P V )
∂P T T ∂P T

a
d’où P V = − 2 P + ϕ(T ). On pourrait déterminer immédiatement la fonction ϕ(T ) en
T
utilisant la condition que pour les grands volumes l’équation d’état doit se confondre avec
celle des gaz parfaits, P V = RT , d’où ϕ(T ) = RT . Mais continuons plutôt le processus
d’intégration en utilisant l’expression de α qui impose que

∂V 2a ϕ0 V 3a 2a ϕ
 
= 3
+ =Vα= + 3 = 3+
∂T P T P T T T PT

soit ϕ0 /ϕ = 1/T , donc ϕ(T ) = KT où K est une constante que l’on ajuste au moyen de
ladite condition limite. On trouve ainsi K = R.

Christian Carimalo 7 TD de Thermodynamique - L3


d) Si l’on se donne

R(V + a0 )2 R(V + a0 )
α= , β =
P V 2 (V + 2a0 ) PV 2

le mieux est alors de rechercher l’équation d’état sous la forme T = F (P, V ), c’est-à-dire,
intégrer les relations

∂T 1 P V (V + 2a0 ) ∂T 1 V2
   
= = , = =
∂V P Vα R(V + a0 )2 ∂P V Pβ R(V + a0 )

De façon évidente, commençons par la seconde :

PV 2
T = + ϕ(V )
R(V + a0 )

Puis, en utilisant la première :

∂T 1 P V (V + 2a0 ) P V (V + 2a0 )
 
0
= = + ϕ (V ) =
∂V P Vα R(V + a0 )2 R(V + a0 )2

on en déduit ϕ(V ) = constante = K. La condition limite des gaz parfaits donne alors K = 0.
On obtient finalement l’équation

a0
 
P V = RT 1 +
V

∂χT ∂α
   
B/ 1◦ ) = − = −B, d’où χT (T, P ) = −B(T − T0 ) + ϕ(P ). Mais
∂T P ∂P T
χT (T0 , P ) = χ0 = ϕ(P ) est indépendant de P . Ainsi,

χT (T, P ) = χ0 − B(T − T0 )

∂ ln V
 
2◦ ) = −χT , d’où 3 ln V = −χ0 (P − P0 ) + B(P − P0 )(T − T0 ) + ψ(T ). Sans
∂P T
aller plus loin, il suffit de consid’erer la condition ln V (T0 , P0 ) = ln M v0 = ψ(T ), M étant
la masse molaire de l’eau, pour trouver que ψ(T ) est en fait une constante. Finalement,
l’équation d’état recherchée prend la forme

ln V (T, P ) = −χ0 (P − P0 ) + B(P − P0 )(T − T0 ) + ln M v0

1 ∂V R
 
C/ L’intégration de =α= donne V = f (P )(RT + bP ). Utilisant
V ∂T P RT + bP
ensuite l’expression de χT , il vient

∂V RT V RT f
 
= −V χT = − = f 0 (RT + bP ) + bf = −
∂P T P (RT + bP ) P
d’où f 0 /f= −1/P et f (P ) = K/P où K est une constante. Celle-ci est facilement
déterminée par la condition limite préconisée. On trouve K = 1. L’équation d’état recherchée
est donc :
3. Pour simplifier la suite, on a considéré P − P0 comme primitive de 1 dans l’intégration sur P .

Christian Carimalo 8 TD de Thermodynamique - L3


RT + bP
V (T, P ) =
P

A noter que le coefficient β prend ici une expression très simple :


α 1
β= =
P χT T

xxxxxxxxxxxxxxxxxxxxxxxxxxxx

IV. Echelles de températures


1◦ ) La nouvelle échelle θ est définie par

R − R0 ν t
  
θ = 100 =t 1− −1
R100 − R0 100 100

et est donc différente de l’échelle Celsius (θ 6= t, sauf aux points t = 0◦ C et t = 100◦ C).
νt t
 
2◦ ) L’écart θ−t = − − 1 est maximum pour t = 50◦ C et vaut alors ν/4 = 0, 37◦ C.
100 100
θ−t ν t 1
 
3◦ ) | |= − 1 ≤ 10−2 pour t ≤ 100(1 + ), soit t ≤ 167◦ C.
t 100 100 1, 49

xxxxxxxxxxxxxxxxxxxxxxxxxxxx

Christian Carimalo 9 TD de Thermodynamique - L3


TD2

I - Coefficients calorimétriques
Au cours d’une transformation infinitésimale réversible au cours de laquelle sa pression P , son
volume V et sa température T varient respectivement de dP , dV et dT , un gaz reçoit une
quantité de chaleur infinitésimale d̄ Q que l’on peut écrire sous trois formes équivalentes :

d̄ Q = Cv dT + `dV = Cp dT + hdP = λdP + µdV

1◦ ) Donner les définitions de Cv , Cp , `, h, λ et µ en précisant leurs caractères extensif ou


intensif.
2◦ ) Exprimer les rapports λ/Cv , µ/Cp , `/h en fonction des coefficients thermoélastiques α, β
et χT , et montrer que Cp − Cv = αV `.
3◦ ) On suppose le gaz parfait.
a) On a alors ` = P . En déduire dans ce cas l’expression de Cp − Cv .
b) Dans l’hypothèse où le rapport γ = Cp /Cv est constant, déterminer l’équation des isen-
tropiques (adiabatiques réversibles) pour ce gaz.

II - Barre métallique
On considère une barre métallique de longueur `, de section droite Σ, de masse volumique
ρ, de capacité calorifique par unité de masse à force constante (chaleur massique) C, sur
laquelle s’exerce une force de traction F dans la direction de la longueur. L’état d’équilibre
de la barre est décrit par une équation d’état ` = `(F, T ).
1◦ ) Soit une transformation infinitésimale réversible où la force de traction, la température,
la longueur varient respectivement de dF , dT et d`.
a) Quel est le travail élémentaire d̄ W reçu par la barre ?
b) La chaleur reçue par la barre s’écrit d̄ Q = A(F, T )dT + k(F, T )dF . Que représentent A
et k ? Exprimer A en fonction de C, ρ, Σ et `.
∂A
 
c) Exprimer k et à l’aide de T et des dérivées de `.
∂F T

2◦ ) On effectue une traction adiabatique réversible.


a) Quelle est la variation dT1 de température consécutive à une variation infinitésimale dF1
de la force ? Quel est son signe ?
1 ∂`
 
b) On appelle coefficient de dilatation linéaire λ de la barre la grandeur λ = .
` ∂T F
dF1
Exprimer en fonction de T, λ, C, ρ et Σ.
dT1
c) Application : calculer la variation de température résultant d’une traction adiabatique pour
laquelle la traction par unité de surface passe de 0 à 10 kg mm−2 , sur une barre de cuivre à

Christian Carimalo 10 TD de Thermodynamique - L3


27◦ C. On donne λ = 1, 6 10−5 K−1 ; C = 400 J kg−1 ; ρ = 9 103 kg m−3 .
3◦ ) On chauffe réversiblement la barre en maintenant la longueur constante.
a) Quelle force dF2 faut-il exercer si on augmente la température de dT2 ?
1 1 ∂`
 
b) Le module d’élasticité isotherme ET de la barre est défini par =Σ . Exprimer
ET ` ∂F T
dF2
à l’aide de λ, ET , Σ.
dT2
c) Application : pour la barre précédente, ET = 12 1010 N m−2 . Quelle force par unité de
surface faut-il exercer pour élever la température de 1 K en maintenant la longueur constante ?
1 1 ∂`
 
4◦ ) On définit le module d’élasticité ES à entropie S constante par = Σ .
ES ` ∂F S
Montrer que

1 1 T λ2
− =
ET ET ρC

III - Corps solide


Pour un solide soumis à des variations de pression et de température pas trop élevées, on
admet que le coefficient de dilatation linéaire αlin à pression constante ainsi que le coefficient
de compressibilité isotherme χT sont constants.
1◦ ) Montrer que si les propriétés du solide sont uniformes dans toutes les directions, le
1 ∂V
coefficient de dilatation volumique αv = est égal à 3 αlin .
V ∂T P
2◦ ) Montrer que l’on peut alors écrire l’équation d’état sous la forme

V ' V1 + V0 (3 αlin T − χT P )

où V0 est le volume à la température de référence T0 et à faible pression (P → 0). Déterminer


alors V1 .
∂U V − V1 ∂CV
   
3◦ ) Montrer que = puis déterminer .
∂V V V0 χT ∂V T

4◦ ) On suppose que CV est indépendant de T . Exprimer l’énergie interne U en fonction de


T et V . Exprimer de même l’entropie S à l’aide des variables T et V , puis des variables T
et P . Dans chaque cas, on choisira un état de référence.

Christian Carimalo 11 TD de Thermodynamique - L3


Corrigé TD2

I - Coefficients calorimétriques
1◦ ) d̄ Q et V sont des grandeurs extensives, T et P sont des grandeurs intensives.
• Cv , grandeur extensive, est la capacité calorifique à volume constant du gaz.
• Cp , grandeur extensive, est la capacité calorifique du gaz à pression constante.
• `, grandeur intensive homogène à une pression, est le coefficient de chaleur de dilatation
isotherme du gaz.
• h, grandeur extensive homogène à un volume, est le coefficient de chaleur de compression
isotherme.
Les coefficients λ et µ, peu utilisés, n’ont pas de désignation spéciale ; λ, extensif, est ho-
mogène à un volume ; µ, intensif, est homogène à une pression.
1
2◦ ) Dans la dernière expression de d¯Q, faisons le remplacement dP = P βdT − dV ,
V χT
et identifions le résultat obtenu à la première expression de d̄ Q. Il vient

λ 1 λ
Cv = P βλ soit = = αχT , et ` = µ −
Cv Pβ V χT

Toujours dans la troisième expression de d̄Q, faisons le remplacement dV = V αdT −V χT dP


et identifions le résultat obtenu à la seconde expression de d̄ Q. On obtient cette fois
µ 1
Cp = µV α soit = , et h = λ − µV χT
Cp Vα

Enfin, exprimons dV en fonction de dT et dP dans la première expression de d̄ Q :

Cv dT + `V (αdT − χT dP ) = Cp dT + hdP, d0 où


` 1
Cp − Cv = `V α , h = −`V χT soit =−
h V χT

PV
3◦ ) a) Pour le gaz parfait, ` = P , α = 1/T , d’où Cp − Cv = = nR pour n moles. En
T
outre, on a alors χT = 1/P et donc h = −V .
b) Pour une isentropique, d¯Q = 0, ce qui donne Cv dT = −`dV = −P dV et Cp dT =
−hdP = V dP . Dans le plan (P, V ), l’équation différentielle caractérisant une isentropique
dP P
du gaz parfait est donc = −γ . Si γ est constant, elle s’intègre aisément pour donner
dV V
la relation

P V γ = constante

xxxxxxxxxxxxxxxxxxxxxxxxxxxx

Christian Carimalo 12 TD de Thermodynamique - L3


II - Barre métallique
1◦ ) a) d̄ W = F d`.
b) A = C ρ ` Σ est la capacité calorifique à force constante de la barre ; k est le coefficient
de chaleur de traction isotherme (ou encore, chaleur latente de traction).
c) Notons U et S l’énergie interne et l’entropie de la barre, et introduisons les fonctions d’état
H = U − F ` (enthalpie) et G = H − T S (“enthalpie libre”). La différentielle de l’entropie
est

d̄ Q A k
dS(≡ ) = dT + dF
T T T

Celle de l’enthalpie est (dU ≡ d̄ W + d̄ Q)

dH = dU − F d` − `dF = AdT + (k − `)dF

et celle de l’enthalpie libre est

dG = dU − F d` − T dS − `dF − SdT = −`dF − SdT

Exprimons le fait que cette dernière est une différentielle exacte. On obtient

∂S k ∂`
   
= =
∂F T T ∂T F

Puis, exprimant que dH est une différentielle exacte :


!
∂A ∂ ∂2`
   
= (k − `) =T
∂F T ∂T F ∂T 2 F

k ∂`
 
2◦ ) a) On écrit d̄ Q1 = 0, d’où dT1 = − dF1 . Comme et A sont positifs, dT1 est
A ∂T F
négatif pour dF1 positif.
dF1 A C ρ`Σ C ρΣ
b) =− =− =−
dT1 k λT ` λT
∆F1 λT
c) ∆T1 = − = −0, 13 K (1 kg ≡ 9, 81 N)
Σ Cρ
dF2
3◦ ) a) b) = −λ Σ ET
dT2
∆F2
c) = −∆T2 λ ET = −1, 92 N mm−2
Σ
∂` ∂`
   

4 ) De la différentielle d` = dT + dF on tire
∂T F ∂F T
∂` ∂` ∂T ∂`
       
= +
∂F S ∂T F ∂F S ∂F T

∂T k
 
Or, = − . On en déduit
∂T S A

Christian Carimalo 13 TD de Thermodynamique - L3


∂` ∂` ∂` ∂T k`λ T λ2 `
       
− =− = =
∂F T ∂F S ∂T F ∂F S A CρΣ

D’où

1 1 T λ2
− =
ET ET ρC
xxxxxxxxxxxxxxxxxxxxxxxxxxxx

III - Corps solide


∂ ln V
 
1◦ ) αv = . Pour un corps de forme cubique de côtés a, b et c par exemple,
∂T P
1 ∂a
 
V = abc, ln V = ln a + ln b + ln c, αv = + etc = 3 αlin si les propriétés du solide
a ∂T P
sont uniformes dans toutes les directions.
2◦ ) Tant que les variations de température et de pression restent limitées, V reste voisin de
V0 . On peut alors écrire approximativement

∂V ∂V
   
' 3αlin V0 , ' −V0 χT
∂T P ∂P T

d’où, par intégration, V ' V1 + V0 (3 αlin T − χT P ). Comme pour T = T0 et P → 0, on a


V = V0 , il vient V1 = V0 (1 − 3αlin T0 ).
∂U ∂P
   
3◦ ) On a = ` − P avec ` = T comme conséquence des premier et second
∂V T ∂T V
principes. Or, ici,

∂P 3αlin
 
=
∂T V χT

∂U 1 V − V1 ∂CV ∂2U
   
D’où = (3αlin T − χT P ) = . Puis, comme = =
∂V  T χ T V 0 χT ∂V T ∂V ∂T
∂ ∂U ∂CV
   
, on obtient 4 = 0.
∂T ∂V T V ∂V T
1 h i
4◦ ) • U (T, V ) = U (T0 , V0 ) + Cv (T − T0 ) + (V − V1 )2 − (V0 − V1 )2
2V0 χT
T 3αlin
• S(T, V ) = S(T0 , V0 ) + Cv ln + (V − V0 )
T0 χT

xxxxxxxxxxxxxxxxxxxxxxxxxxxx

 
∂CV ∂2P
 
4. A noter que d’une façon générale, =T , formule qui conduit ici à un résultat
∂V T ∂T 2 V
nul.

Christian Carimalo 14 TD de Thermodynamique - L3


TD3

I - Cycle de Carnot d’un gaz parfait


Une masse m d’un gaz parfait monoatomique décrit un cycle constitué par les transformations
réversibles suivantes :
• une transformation adiabatique A(PA , VA , T2 ) → B(PB , VB , T1 ) avec T1 > T2 ;
• une détente à température constante B → C(PC , VC , T1 ) ;
• une transformation adiabatique C → D(PD , VD , T2 ) ;
• une compression à température constante D → A.
On admettra que la capacité calorifique à volume constant du gaz est indépendante de la
température.
1◦ ) a) Représenter le cycle dans le plan (P, V ) (diagramme de Clapeyron).
b) Démontrer les relations PA PC = PB PD et VA VC = VB VD .
2◦ ) a) Déterminer les travaux WAB , WBC , WCD et WDA reçus par le gaz dans chacune des
transformations constituant le cycle, en fonction des coordonnées des états initial et final
correspondants.
b) Quelle est la relation entre WAB et WCD ? Retrouver directement cette relation en ap-
pliquant le premier principe de la Thermodynamique et en tenant compte du fait que le gaz
est parfait.
3◦ ) a) Déterminer, en fonction des coordonnées des sommets du cycle, les quantités de
chaleur QAB , QBC , QCD et QDA reçues par le gaz dans les quatre transformations du cycle
et en préciser les signes.
b) Etablir une relation entre QBC et QDA .
4◦ ) Déterminer le travail total W reçu par le gaz au cours du cycle. Montrer que l’on pouvait
prévoir son signe et le vérifier.
5◦ ) Donner la définition générale du rendement relatif à un cycle et déterminer le rendement
η du cycle considéré ici.
6◦ ) a) Déterminer les coordonnées des états C et D de telle sorte que QBC ait une valeur
Q1 fixée à l’avance.
b) Application numérique :
PA = 105 Pa ; T1 = 300 K ; T2 = 280 K ; Q1 = 200 J ; γ = 5/3 ; R = 8, 31 J K−1 mole−1 ;
nombre de mole : 1/10.

Christian Carimalo 15 TD de Thermodynamique - L3


II - Cycle d’un gaz de Van der Waals
Une mole d’un gaz de Van der Waals a pour équation d’état

a
 
P + 2 (V − b) = RT
V

On fait subir à cette mole de gaz le cycle suivant :


• détente à pression extétieure constante A(P0 , V0 ) → A1 qui double son volume ;
• compression réversible isotherme A1 → A2 qui la ramène à son volume initial ;
• un refroidissement isochore A2 → A0 .
1◦ ) Représenter le cycle dans le diagramme de Clapeyron.
2◦ ) Déterminer en fonction de P0 , V0 , a et b le travail reçu par le gaz dans chacune des
transformations du cycle.
3◦ ) En déduire le travail total et la quantité de chaleur totale reçus au cours du cycle.

III - Transformations adiabatiques et isothermes, réversibles et irréversibles


d’un gaz parfait
On considère un cylindre muni d’un piston de masse négligeable, tous deux imperméables
à la chaleur. On dispose sur le piston une masse M (figure 6). L’ensemble est soumis à la
pression atmosphérique P0 .

P0

He

Figure 1 – Gaz dans un corps de pompe

Au début de l’expérience, un opérateur maintient le piston de telle sorte que la hauteur


de celui-ci au dessus du fond du cylindre soit h0 . Cet espace libre est rempli d’hélium à la
température T0 et à la pression P0 . L’hélium sera considéré comme un gaz parfait monoato-
mique.
Pour les applications numériques, on prendra : section intérieure du cylindre s = 100 cm2 ;
h0 = 1 m ; P0 = 9, 81 104 Pa ; M = 50 kg ; T0 = 273 K ; g = 9, 81 m/s2 ; R = 8, 31 J K−1
mole−1 ; γ = 5/3.
1◦ ) a) Dans une première expérience, l’opérateur laisse descendre très lentement le piston
dans le cylindre jusqu’à ce que celui-ci se trouve en équilibre sur la colonne d’hélium. Quelles
sont, à la fin de l’expérience, la hauteur h et la température T de la colonne d’hélium ?
Calculer le travail W1 reçu par le gaz ainsi que la variation ∆1 S de son entropie.
b) Dans une seconde expérience, les conditions initiales étant les mêmes que précédemment,

Christian Carimalo 16 TD de Thermodynamique - L3


l’opérateur lâche brusquement le piston qui comprime le gaz et s’immobilise après quelques
oscillations. Calculer la hauteur finale h01 et la température finale T10 de l’hélium à l’équilibre.
Calculer le travail W10 reçu par le gaz et le comparer à W1 . Calculer également la variation
∆01 S de l’entropie du gaz pour cette nouvelle transformation.
c) Le piston étant dans la position d’équilibre correspondant à la hauteur h01 , l’opérateur
enlève brusquement la masse M . Calculer la hauteur h001 à laquelle le piston sera remonté
à l’équilibre final et la température finale T100 de l’hélium. Comparer h001 et T100 à h0 et T0
respectivement. Calculer le travail W100 reçu par l’hélium ainsi que la variation ∆001 S de son
entropie. Comparer W100 à W10 .
2◦ ) Les mêmes expériences sont reprises avec cette fois un cylindre à parois diathermes,
c’est-à-dire, conductrices de la chaleur. Le cylindre reste en contact avec un thermostat à la
température T0 .
a) Le piston est abaissé très lentement par l’opérateur jusqu’à ce qu’il soit en équilibre sur
la colonne d’hélium. Calculer la hauteur finale d’équilibre h2 , le travail W2 et la chaleur Q2
reçus par le gaz. Calculer la variation d’entropie ∆2 S du gaz.
b) Le piston est ramené à sa position initiale et est alors lâché brusquement par l’opérateur.
Calculer la hauteur finale d’équilibre h02 , le travail W20 et la chaleur Q02 reçus par le gaz.
Comparer W20 à W2 . Calculer la variation d’entropie ∆02 S du gaz.
c) La colonne d’hélium étant à l’équilibre correspondant à la hauteur h02 , l’opérateur retire
brusquement la masse M . Calculer la hauteur finale d’équilibre h002 , le travail W200 et la chaleur
Q002 reçus par le gaz. Comparer W200 à W20 . Calculer la variation d’entropie ∆002 S du gaz.
d) Calculer les variations d’entropie ∆SΣ , ∆0 SΣ , ∆00 SΣ correspondant aux expériences précé-
dentes, pour le système thermodynamique isolé Σ constitué par le cylindre et le thermostat.
Commenter.

IV - Etude d’un cycle


On considère n moles d’un gaz parfait subissant le cycle de transformations suivant :
• transformation adiabatique réversible A(PA , TA ) → B(PB , TB ) avec TB < TA ;
• transformation isochore B(PB , TB ) → C(PC , TC = TA ) au cours de laquelle le gaz reçoit
une quantité de chaleur positive d’une source de chaleur à température constante TA ;
• transformation isotherme réversible C → A pendant laquelle le gaz est en contact avec la
source de chaleur de température TA .
On note γ le rapport, supposé constant, des chaleurs molaires à pression et à volume constant
du gaz et R la constante des gaz parfaits.
1◦ ) Représenter le cycle ABCA dans le diagramme de Clapeyron et préciser la nature,
réversible ou irréversiblle, de ce cycle.
2◦ ) Déterminer, exclusivement en fonction de TA , PA , TB , n, R et γ :
a) la capacité calorifique à volume constant Cv du gaz ;
b) les volumes VA et VB du gaz dans les états A et B respectivement ;
c) la variation d’énergie interne ∆AB U du gaz entre les états A et B ;

Christian Carimalo 17 TD de Thermodynamique - L3


d) la variation d’entropie ∆BC S du gaz entre les états B et C ;
e) le travail WCA reçu par le gaz dans la transformation CA.
3◦ ) Disposer dans un tableau, pour les trois transformations, les travaux et chaleurs reçus
par le gaz, ainsi que les variations d’énergie interne et d’entropie du gaz.
4◦ ) Déterminer le travail total W et la quantité de chaleur totale Q reçus par le gaz au cours
du cycle. Quels sont leurs signes respectifs ?
5◦ ) Calculer la variation d’entropie ∆Ssource de la source de température TA au cours du
cycle. Commenter à l’aide du principe d’évolution.
6◦ ) a) On se propose d’étudier la possibilité d’inverser le sens du cycle. Montrer, en le
justifiant, qu’il est alors nécessaire de mettre en jeu une seconde source de chaleur. Celle-ci
sera choisie de température constante. Quelle condition doit remplir cette température ? Au
cours de quelle transformation le gaz doit-il être mis en contact avec la seconde source ?
b) Pour ce nouveau cycle, calculer la variation d’entropie de l’ensemble des deux sources.
Conclusion ?

V - Modélisation du fonctionnement d’un moteur de voiture

C adiabatiques

D
B

Figure 2 – Cycle de Beau de Rochas

On modélise le fonctionnement d’un moteur de voiture à l’aide du cycle de Beau de Rochas


schématisé à la figure 8. Il est constitué de
• deux adiabatiques réversibles AB et CD d’équations P V n = constante où n est un nombre
supérieur à 1.
• deux transformations réversibles BC et DA à volume constant.
Le mélange combustible est assimilé, avant comme après combustion, à un gaz parfait dont
la capacité calorifique à volume constant Cv est constante.
1◦ ) Dans quel sens est décrit le cycle ? Justifier la réponse.
2◦ ) Calculer, en fonction de Cv , TA , TB , TC et TD les quantités de chaleur reçues par le gaz

Christian Carimalo 18 TD de Thermodynamique - L3


dans chacune des quatre transformations du cycle, ainsi que le travail total W reçu par le
gaz au cours du cycle.
3◦ ) Soit Q1 la quantité de chaleur produite par la combustion du mélange dans la transfor-
mation CD. Exprimer le rendement η = −W/Q1 en fonction de TA , TB , TC et TD .
4◦ ) On appelle rapport de compression le nombre a = VA /VB = VD /VC . Exprimer TC et
TB en fonction de TA , TD , a et n. En déduire une expression simple de η en fonction de a et
n. Application numérique : PA = 1 atm., PB = 12 atm., a = 8 ; calculer an et en déduire η.
5◦ ) a) La voiture a une puissance mécanique de 20 kW. Quelle quantité de chaleur par
seconde doit fournir la combustion AB ?
b) L’essence a un pouvoir calorifique de 11500 kcal/kg (1 cal = 4,18 J). Quelle est la masse
d’essence brulée en 1 seconde ?
c) En déduire la consommation en litres aux 100 km. L’essence a pour masse volumique 0,7
kg/litre et la voiture roule à 90 km/h.

VI -
A/ - On rappelle que la quantité de chaleur d̄ Q reçue par une mole de gaz au cours d’une
transformation infinitésimale réversible peut s’exprimer en fonction des variables P et V par
la relation d̄ Q = λdP + µdV où λ et µ sont des coefficients calorimétriques.
1◦ ) Exprimer λ et µ en fonction de Cp et Cv , chaleurs molaires  à pressionconstante et à
∂T ∂T
 
volume constant respectivement, et des dérivées partielles et .
∂V P ∂P V
Dans la suite du problème, le gaz est considéré comme un gaz parfait pour lequel le rapport
γ = CP /CV est une fonction linéaire de la température : γ = aT + b où a et b sont deux
constantes positives.
2◦ ) Exprimer λ et µ en fonction de P, V et γ.
3◦ ) Déterminer l’énergie interne molaire U (T ) de ce gaz.
T V b−1
4◦ ) Démontrer que l’équation des isentropiques s’écrit = constante
aT + b − 1
B/ Le gaz parfait considéré précédemment décrit un cycle constitué des quatre transforma-
tions suivantes :
• une compression isotherme A(V1 , T1 ) → B(V2 , T1 ) pendant laquelle le gaz est en contact
avec une source de chaleur à la température T1 ;
• un échauffement à volume constant B → C(V2 , T2 ) pendant lequel le gaz est en contact
avec une source de chaleur de température T2 ;
• une détente isotherme C → D(V1 , T2 ) pendant laquelle le gaz reste en contact avec la
source de température T2 ;
• un refroidissement isochore D → A pendant lequel le gaz est en contact avec la source de
température T1 .
1◦ ) Quelle est la nature de la machine thermique ditherme dans laquelle circule le gaz ?
2◦ ) a) Pour chacune IJ des transformations du cycle, calculer le travail WIJ et la chaleur

Christian Carimalo 19 TD de Thermodynamique - L3


QIJ reçus par le gaz, en précisant leurs signes. Toutes les grandeurs calculées seront exprimées
uniquement en fonction de T1 , T2 , V1 , V2 , a, b et R.
b) Quelles sont les quantités de chaleur QF et QC fournies au gaz respectivement par la
source froide et par la source chaude ?
3◦ ) Exprimer le rendement η de cette machine en fonction de T1 , T2 , V1 , V2 , a, b et R.
4◦ ) On considère maintenant un moteur thermique fonctionnant réversiblement suivant un
cycle de Carnot entre les deux sources de chaleur de températures respectives T1 et T2 > T1 .
a) Rappeler ce qu’est un cycle de Carnot. Etablir l’expression du rendement ηc de ce moteur.
b) Comparer l’expression de η à celle de ηc . Le résultat est-il conforme à ce que vous savez
du fonctionnement des machines thermiques ? Justifier la réponse.

VII - Cycle de Brayton


Le cycle de Brayton décrit par un gaz est constitué des transformations réversibles suivantes :
• compression isentropique A1 (P1 , T1 ) → A2 (P2 , T2 ) ;
• détente isobare A2 (P2 , T2 ) → A3 (P2 , T3 ) ;
• détente isentropique A3 (P2 , T3 ) → A4 (P1 , T4 ) ;
• compression isobare A4 (P1 , T4 ) → A1 (P1 , T1 ) ;
P et T étant la pression et la température du gaz, respectivement.
Ce cycle sert de modèle théorique pour décrire le fonctionnement thermodynamique idéal de
certains moteurs thermiques ou de certaines machines frigorifiques et de climatiseurs. Le gaz
utilisé est de l’air que l’on considèrera comme un gaz parfait de masse molaire M et dont le
rapport γ = Cp /Cv est constant.

A/ Moteur thermique
1◦ ) Dans le diagramme de Clapeyron, représenter le cycle de Brayton, parcouru dans le sens
indiqué plus haut, puis dans le diagramme entropique où l’entropie est portée en abscisse et
la température en ordonnée.
2◦ ) Dans ce diagramme entropique, que représente l’aire du cycle ? Montrer, en justifiant la
réponse, que ce cycle correspond bien à celui d’un moteur thermique.
3◦ ) Préciser les signes des quantités de chaleur Q12 , Q23 , Q34 et Q41 reçues par le gaz au
cours de chacune des quatre transformations du cycle.
4◦ ) Définir le rendement η du cycle et l’exprimer en fonction des températures T1 , T2 , T3 et
T4 .
5◦ ) a) Soit r = P2 /P1 le rapport de compression pour ce cycle. Exprimer les températures
T2 et T3 en fonction de T1 , T4 , r et γ.
c) En déduire l’expression de η en fonction de r et γ.
6◦ ) Exprimer les températures T2 et T3 en fonction de T1 , T4 et η.

Christian Carimalo 20 TD de Thermodynamique - L3


B/ Machine frigorifique
Le cycle de Brayton de sommets A1 , A2 , A3 et A4 est maintenant parcouru en sens inverse
du précédent.
1◦ ) En précisant les signes des quantités de chaleur Q14 , Q43 , Q32 et Q21 , reçues par le gaz
au cours des quatre transformations de ce nouveau cycle, montrer que l’on peut décrire le
fonctionnement d’une machine frigorifique.
2◦ ) a) Définir l’efficacité e de la machine frigorifique ainsi modélisée et l’exprimer en fonction
de T1 , T2 , T3 et T4 .
b) Exprimer e en fonction du rapport de compression r de ce cycle et de γ.
3◦ ) Exprimer T2 et T3 en fonction de T1 , T4 et e.

C/ Etude du compresseur intervenant dans la transformation adiabatique


Le rôle du compresseur est d’aspirer l’air se trouvant initialement dans une conduite de
section droite Si à la pression Pi et à la température Ti , et de le refouler dans une conduite
de section droite Sf dans les conditions de pression et de température (Pf , Tf ) (figure 3
(a)). Poussé par les couches d’air en amont, l’air pénètre dans le compresseur et en sort en
repoussant les couches d’air en aval. Dans le corps du compresseur, des aubes mues par un
moteur électrique brassent l’air en lui fournissant un travail Wc par mole d’air traversant le
compresseur. Les parois de la tuyauterie et du compresseur sont supposées adiabatiques. Cet
écoulement adiabatique de l’air est considéré en régime permanent.
1◦ ) Considérer une masse d’air Σ se trouvant à la date t entre une section Si (t) de la conduite
en amont et une section Sf (t) de la conduite en aval et qui, à la date t + dt se retrouve entre
les sections Si (t + dt) et Sf (t + dt) (figure 3 (b)). En appliquant le premier principe de la
Thermodynamique à Σ entre t et t + dt et en igorant l’énergie cinétique macroscopique de
l’air dans cet écoulement, démontrer que

Wc = H(Tf , Pf ) − H(Ti , Pi )

où H(T, P ) est l’enthalpie molaire de l’air. Ce résultat pourra être admis sans démonstration
pour la suite du problème.
2◦ ) L’air étant encore considéré comme un gaz parfait, en déduire l’expression de Wc en
fonction des données.

D/ Application : machine frigorifique


Une machine frigorifique a les caractéristiques suivantes :
• débit massique d’air à l’entrée du compresseur ègal à D = 15, 8 g s−1 ;
• puissance de réfrigération (ou chaleur retirée par unité de temps) égale à q = 1 kW ;
• efficacité e = 1, 9 ;
• P1 = 1 atm., T2 = 290 K ;
Pour l’air, on a M = 29 g mole−1 , γ = 1, 4.
1◦ ) Donner les valeurs numériques de T1 , T3 , T4 et P2 .

Christian Carimalo 21 TD de Thermodynamique - L3


2◦ ) Quelle est la puissance du compresseur ?

S i(t) S f (t)
Wc

Si Sf

S i (t+dt) S f (t+dt)

(a) (b)

Figure 3 – Ecoulement de l’air dans le compresseur

Christian Carimalo 22 TD de Thermodynamique - L3


Corrigé TD3

I - Cycle de Carnot d’un gaz parfait


1◦ ) a)
P
B

D V

Figure 4 – Cycle de Carnot

b) Pour les transformations isentropiques AB et CD, on peut écrire

PA VAγ = PB VBγ et PC VCγ = PD VDγ

avec γ = Cp /Cv = 5/3 (gaz parfait monoatomique), d’où PA PC (VA VC )γ = PB PD (VB VD )γ .


Pour les transformations isothermes DA et BC on a les relations

PA VA = PD VD et PC V C = PB V B

d’où PA PC VA VC = PB PD VB VD . De ces relations on déduit aisément

PA PC = PB PD , VA VC = VB VD

3
2◦ ) a) • WAB = ∆AB U = nR(TB − TA )
2
VC
• WBC = −nRTB ln
VB
3 3
• WCD = ∆CD U = nR(TD − TC ) = nR(TA − TB )
2 2
VA
• WDA = −nRTA ln
VD
b) WAB = −WCD . Cette égalité résulte des circonstances suivantes. D’une part, les deux
transformations AB et CD étant des adiabatiques, QAB = QCD = 0 et par conséquent
WAB = UB − UA , WCD = UD − UC . D’autre part, les deux transformations BC et DA
sont isothermes. Or, l’énergie interne d’un gaz parfait ne dépend que de la température et
par suite UB = UC , UD = UA . D’où l’égalité mentionnée.

Christian Carimalo 23 TD de Thermodynamique - L3


3◦ ) a) D’après ce qui précède,
• QAB = QCD = 0
VC VA
• QBC = −WBC = nRTB ln > 0, QDA = −WDA = nRTA ln <0
VB VD
b) De l’égalité VA VC = VB VD on tire VA /VD = VB /VC et par suite

QBC QDA
+ =0
TB TA

On retrouve ici l’égalité de Clausius qui résulte du bilan entropique de ce cycle réversible.
VD
4◦ ) W = nR(TA − TB ) ln . Ce travail doit être négatif car dans le diagramme (P, V ) il
VA
est parcouru dans le sens trigonométrique. Il en est bien ainsi car VD > VA (DA est une
compression) et TA = T2 < TB = T1 .
5◦ ) Un rendement est défini comme le rapport de ce qui est récupéré sur ce qui est fourni. Ici,
on récupère le travail |W | alors que seule la quantité de chaleur QBC est réellement fournie.
On définit donc le rendement du cycle de Carnot comme η = −W/QBC . On trouve ainsi

TA T2
η =1− =1− = 0, 07
TB T1

6◦ ) a) b) On doit exprimer les coordonnées du cycle en fonction des données de l’application


numérique qui sont Q1 , PA , T1 , T2 , n et R. Le volume VA est donné par l’équation d’état :
Q1
VA = nRT2 /PA = 2, 3 10−3 m3 . Le volume VD s’en déduit par VD = VA exp =
nRT1
2, 23 VA = 5, 13 10−3 m3 . Le volume VC s’obtient de la même manière à partir de VB et ce
dernier volume est connu si l’on connait PB , puisque VB = nRT1 /PB . Or, en fonction des
variables P et T , l’équation d’une isentropique du gaz parfait prend la forme
γ
P T γ − 1 = constante

5
T1 2

d’où l’on tire PB = PA = 1, 19 PA . Ainsi, VB = 2, 1 10−3 m3 et VC = 4, 68 10−3 m3 .
T2

xxxxxxxxxxxxxxxxxxxxxxxxxxxx

II - Cycle d’un gaz de Van der Waals


1◦ ) On admet ici que toutes les transformations sont réversibles. Voir figure ci-après.
2◦ ) • Transformation A0 A1 : W01 = −P0 V0 .
2V0 − b a a
• Transformation A1 A2 : W12 = RT1 ln − avec T1 = (P0 + )(2V0 − b).
V0 − b 2V0 4V02
• Transformation A2 A0 : W20 = 0.
3◦ ) W = −Q = W01 + W12 > 0.

Christian Carimalo 24 TD de Thermodynamique - L3


P

A2

P A1
0 A0

V0 2V0 V

Figure 5 – Cycle du gaz de Van der Waals

xxxxxxxxxxxxxxxxxxxxxxxxxxxx

III - Transformations adiabatiques et isothermes, réversibles et irréversibles


d’un gaz parfait

P0

He

Figure 6 – Gaz dans un corps de pompe

1◦ ) a) La première transformation est une adiabatique réversible au cours de laquelle la


pression et le volume du gaz sont liés par la relation P V γ = constante. La pression finale
Mg Mg
est Pf = P0 + . Or, = 0, 5 × 9, 81 × 104 Pa = P0 /2, donc Pf = 3P0 /2. On a ainsi
s s
 3
2 5
Pf hγ = P0 hγ0 soit h = h0 = 78, 4 cm
3

Appliquant l’équation d’état P h = nRT /s aux états initial et final, on obtient nR/s =
Pf h 3 h
Pf h/Tf = P0 h0 /T0 , soit Tf = T0 = T0 = 321 K.
P 0 h0 2 h0
∆(P V )
• Puisque Q1 = 0, on a W1 = ∆1 U = , soit
γ−1
3 3 Tf
 
W1 = (Pf h − P0 h0 ) = sP0 h0 − 1 = 259 J
2 2 T0

• ∆1 S = 0 (adiabatique réversible, donc isentropique).


b) Cette seconde transformation est manifestement irréversible et l’on ne peut plus appliquer
la relation P V γ = constante pour déterminer la hauteur finale. En revanche, comme elle

Christian Carimalo 25 TD de Thermodynamique - L3


est aussi adiabatique, on peut encore écrire la relation bilan : W10 = ∆01 U . Comme c’est la
Pf h0f − P0 h0
pression Pf qui fournit le travail, on a donc W10 = −Pf s (h0f −h0 ) = ∆01 U = s
γ−1
d’où l’on tire !
0 P0 4
hf = γ h0 + γ − 1 = h0 = 80 cm
Pf 5

Pf h0f
On en déduit aussi Tf0 = T0 = 1, 2 T0 = 328 K, puis W10 = 294 J.
P 0 h0
• Calculons la variation correspondante de l’entropie. Son expression générale en fonction des
Pf Vfγ nR
variables P et V à l’état initial et à l’état final est ∆S = cv ln γ , avec cv = =
Pi V i γ−1
1 P0 sh0 3 5 4
 
. On obtient ∆01 S = 5 × 0, 981 × ln + ln = 0, 164 J K−1 . Elle est
γ − 1 T0 2 3 5
positive, témoignant du caractère irréversible de cette transformation adiabatique.
c) Il s’agit encore d’une transformation adiabatique irréversible au cours de laquelle c’est la
pression finale P100 = P0 qui travaille. On a maintenant

s
W100 = −P0 s(h001 − h01 ) = ∆001 U = (P0 h001 − P10 h01 )
γ−1
d’où l’on tire finalement
 0
h0 P P0 26
 
h001 = 1
+γ−1 0 + γ − 1 = h0 = 1, 04 m
γ2 P0 P1 25

puis T100 = 278 K, W100 = −235 J.


• La variation d’entropie est maintenant
 00 γ !
P0 h 3 5 26
 
∆001 S = 5 × 0, 981 × ln 1
= 5 × 0, 981 × − ln + ln = 0, 156 JK−1
P100 h01 2 3 20

Elle est aussi positive, comme il se doit pour cette transformation adiabatique irréversible.
2◦ ) Dans cette question, les transformations envisagées sont monotherme (contact avec un
seul thermostat).
a) Ici, il s’agit d’une transformation réversible monotherme, donc isotherme pour le gaz. On
3 2
a T2 = T0 , donc Pf Vf = P0 V0 avec Pf = P0 . D’où h2 = h0 = 67 cm.
2 3
• Le gaz étant parfait, son énergie interne ne dépend que de sa température et par conséquent
∆2 U = 0. On a ainsi

h2
W2 = −Q2 = −P0 V0 ln = 398 J
h0

• La transformation étant réversible et isotherme, on a

Q2
∆2 S = = −1, 46 J K−1
T0

Christian Carimalo 26 TD de Thermodynamique - L3


3
b) La transformation est irréversible, mais Tf = T0 , Pf = P0 . L’état final est donc le même
2
que celui de l’expérience précédente et h02 = h2
P0 V 0
• W20 = −Q02 = −P1 s(h02 − h0 ) = = 490 J.
2
• ∆02 S = ∆2 S (mêmes états initiaux et finals).
c) • P200 = P0 , h002 = h0
P0 V 0
• W200 = −Q002 = −P0 s(h0 − h2 ) = − = −327 J
3
• ∆002 S = −∆02 S = −∆2 S = 1, 46 J K−1
d) • ∆SΣ = 0 (adiabatique réversible pour Σ) ;
Q0 P0 V 0 P0 V 0 h2 P0 V 0 1 3
 
• ∆0 SΣ = ∆02 S − 2 = + ln = − ln = 0, 34 J K−1
T0 2T0 T0 h0 T0 2 2
Cette variation est positive car pour Σ il s’agit d’une transformation adiabatique irréversible.
Q002
• ∆00 SΣ = ∆002 S − = 0, 26 J K−1
T0
Cette variation est positive car pour Σ il s’agit ici encore d’une transformation adiabatique
irréversible.
xxxxxxxxxxxxxxxxxxxxxxxxxxxx

IV - Etude d’un cycle

P
A

B
V

Figure 7 – Cycle du problème 3-IV

1◦ ) Dans la transformation isochore BC, la température du gaz ne peut rester constante.


L’équilibre de température avec celle, constante, du thermostat avec lequel le gaz est alors en
contact ne peut être réalisé en cours de transformation. Cette transformation est donc cer-
tainement irréversible. Pour cette raison, elle est représentée en pointillés dans le diagramme
de Clapeyron.
2◦ ) Déterminer, exclusivement en fonction de TA , PA , TB , n, R et γ :
nR
a) Cv =
γ−1

Christian Carimalo 27 TD de Thermodynamique - L3


nRTA
b) • VA =
PA
• Dans la transformation adiabatique réversible (isentropique) AB la pression et la température
 γ  γ
T γ−1 TB γ − 1
 
du gaz satisfont la relation P = PA , d’où PB = PA , puis VB =
TA TA
nRTB
.
PB
nR
c) ∆AB U = (TB − TA )
γ−1
nR TA
d) ∆BC S = ln (TC = TA )
γ − 1 TB
VA VA nRTA TA
e) WCA = −nRTA ln = −nRTA ln = ln
VC VB γ−1 TB
3◦ ) Pour chacune des transformations on a les relations suivantes.
Pour AB : QAB = 0, ∆AB S = 0, WAB = ∆AB U
nR
Pour BC : WBC = 0, QBC = ∆BC U = (TA − TB )
γ−1
Pour CA : ∆CA U = 0, WCA = −QCA = −TA ∆CA S.
Comme ∆U = 0 et ∆S = 0 pour tout le cycle, on en déduit ∆AB U = −∆BC U , ∆BC S =
−∆CA S et notamment WCA = TA ∆BC S.

W Q ∆U ∆S

nR
AB −∆AB U 0 (TB − TA ) 0
γ−1

nR TA
BC 0 −∆AB U −∆AB U ln
γ − 1 TB

CA TA ∆BC S −TA ∆BC S 0 −∆BC S

nRTA TB TB
 
4◦ ) W = −Q = − 1 − ln . Or, la fonction x − 1 − ln x (avec ici x =
γ − 1 TA TA
TB /TA > 0) est toujours positive et donc W > 0, ce qui est normal car dans le plan (P, V ),
le cycle est parcouru dans le sens inverse du sens trigonométrique.
5◦ ) ∆Ssource − Q/TA = W/TA > 0. A la fin du cycle, la variation d’entropie de l’ensemble
Σ comprenant le gaz et la source est ∆SΣ = ∆Ssource . Elle est positive, ce qui est normal
car pour cet ensemble la suite de transformations est une adiabatique irréversible.
6◦ ) a) L’inverse du cycle serait un cycle au cours duquel le gaz devrait produire un travail
W 0 = −W < 0 alors qu’il ne serait en contact qu’avec un seul thermostat. Mais cela est

Christian Carimalo 28 TD de Thermodynamique - L3


impossible d’après l’énoncé de Kelvin du second principe de la Thermodynamique. Un cycle
moteur ne peut être réalisé que si le gaz échange de la chaleur avec au moins deux sources
de chaleur au cours de ce cycle. Pour le cycle considéré, l’une des sources est le thermostat
dont la température TA est la plus élevée. Le gaz est en contact diatherme avec lui dans la
VC
transformation réversible isotherme AC et en reçoit la chaleur positive QAC = nRTA ln .
VA
La température du gaz diminue de TA à TB dans la transformation isochore CB. C’est
dans cette transformation que le gaz doit être mis en contact avec une source de chaleur de
température T < TA . En fait, ce contact doit être effectif dans toute cette transformation
S’il n’en était pas ainsi, étant donné que l’on suppose la présence de deux sources de chaleur
seulement, cela signifierait que dans une phase de la transformation AC le gaz doit subir une
transformation non seulement isochore mais aussi adiabatique. Pendant cette phase, l’énergie
interne du gaz ne pourrait évoluer, sa température devrait rester constante, son volume aussi
et donc aussi sa pression. Le gaz ne pourrait donc jamais atteindre l’état B. Il faut donc
choisir T = TB .

b) La source chaude de température TA reçoit la quantité de chaleur −QAC . Sa variation


nR TA
d’entropie au cours du cycle est donc ∆SA = −QAC /TA = − ln ; la source froide
γ − 1 TB
de température TB reçoit au cours du cycle la quantité de chaleur −QCB et la variation
nR TA
consécutive de son entropie est ∆SB = −QCB /TB = − (1 − ) ; la variation d’entro-
γ−1 TB
pie du gaz à la fin du cycle est nulle. Ainsi, au cours du cycle, l’ensemble comprenant les deux
nR TA TA
 
sources et le gaz voit son entropie varier de ∆S = ∆SA + ∆SB = − 1 − ln
γ − 1 TB TB
dont la positivité (x − 1 − ln x > 0), rend bien compte de l’irréversibilité du cycle considéré
due, ici encore, à l’irréversibilité de la transformation isochore CB.

xxxxxxxxxxxxxxxxxxxxxxxxxxxx

V - Modélisation du fonctionnement d’un moteur de voiture

C adiabatiques

D
B

V
Figure 8 – Cycle de Beau de Rochas

1◦ ) Dans le plan (P, V ), la surface du cycle donne la valeur absolue du travail reçu au cours
de ce cycle. Le cycle est moteur, donc W < 0, s’il est parcouru dans le sens trigonométrique.
La suite de transformation est donc dans le sens ABCD.

2◦ ) QAB = QCD = 0 ; QBC ∆BC U = Cv (TC − TB ), QDA = ∆DA U = Cv (TA − TD ) ;


W = −QBC − QDA = Cv (TB + TD − TA − TC ).

Christian Carimalo 29 TD de Thermodynamique - L3


TD − TA
3◦ ) η = 1 + QDA /Q1 = 1 −
TC − TB
1
4◦ ) TB = TA an−1 ; TC = TD an−1 , η = 1 − .
an−1
an = PB /PA = 12, η = 1 − 8/12 = 1/3.
|W |
5◦ ) a) Par unité de temps : QBC = = 60 kW.
η
60
b) M = g/s= 1, 25 g/s.
4, 18 × 115
1, 25 10−3
c) Volume par seconde = `/s ' 1, 8 10−3 `/s .
0, 7
100
Volume pour 100 km = 1, 8 × × 3600 ' 7, 2 `
90
xxxxxxxxxxxxxxxxxxxxxxxxxxxx

- VI - A -

1◦ ) On peut écrire indifféremment d̄ Q comme d̄ Q =


 λdP +µdV , d̄ Q = Cv dT + `dV ou
∂T ∂T
d̄ Q = Cp dT + hdP , et écrire dT comme dT = dV + dP . Par identification,
∂V P ∂P V
on en tire
∂T ∂T
   
λ = Cv , µ = Cp
∂P V ∂V P

Dans la suite du problème, le gaz est considéré comme un gaz parfait pour lequel le rapport
γ = CP /CV est une fonction linéaire de la température : γ = aT + b où a et b sont deux
constantes positives.
R
2◦ ) Pour ce gaz, la relation de Mayer est encore applicable et l’on a Cv = , Cp =
 γ− 1
Rγ ∂T
. A l’aide de l’équation d’état P V = RT (pour 1 mole), on obtient = P/R,
γ − 1 ∂V P
∂T V Pγ
 
= V /R, et par suite λ = ,µ= .
∂P V γ−1 γ−1
dU R R
3◦ ) = Cv = , et par intégartion : U = U (T ) = ln [aT + b − 1] + U0 où
dT aT + b − 1 a
U0 est une constante.
4◦ ) Une isentropique peut être définie par l’équation d¯Q = 0, ce qui donne ici Cv dT =
dV dV dT 1 dz
−`dV = −P dV = −RT , soit = − × = en posant
V V T aT + b − 1 a + z(b − 1)
1
z = 1/T . L’intégration donne ln V = ln [a + z(b − 1)] + constante1 , soit finalement la
b−1
T V b−1
relation = constante.
aT + b − 1

Christian Carimalo 30 TD de Thermodynamique - L3


- VI - B -

1◦ ) Il s’agit d’une machine thermique ditherme irréversible, car les deux transformations BC
et CD au cours desquelles le gaz est en contact soit avec l’une, soit avec l’autre des deux
sources de chaleur sont isochores et ne sont donc pas isothermes. La machine est motrice
car dans le plan (P, V ), le cycle est parcouru en sens inverse du sens trigonométrique.
V1
2◦ ) a) • AB : ∆AB U = 0 (isotherme), donc WAB = −QAB = RT1 ln > 0.
V2
R aT2 + b − 1
• WBC = 0, donc QBC = ∆BC U = ln > 0 (T2 > T1 ).
a aT1 + b − 1
V1
• ∆CD U = 0, WCD = −QCD = −RT2 ln < 0.
V2
R aT2 + b − 1
• WDA = 0, donc QDA = ∆DA U = − ln < 0.
a aT1 + b − 1
b) QF = QAB + QDA , QC = QBC + QCD .
V1
ln
W T1
 
V2
3◦ ) η = − = 1− X avec X = .
QC T2 V1 R aT2 + b − 1
ln + ln
V2 a aT1 + b − 1
4◦ ) a) Un cycle de Carnot est consitué de quatre transformations : deux adiabatiques et
deux isothermes. Dans ces dernières, le système décrivant le cycle est en contact soit avec
la source chaude soit avec la source froide, recevant de la première la chaleur Q2 , et de la
seconde la chaleur Q1 . Lorsque le cycle est réversible, ces deux grandeurs vérifient la relation
Q2 Q1
de Clausius + = 0. Le travail W est quant à lui donné par W = −Q2 −Q1 . Si le cycle
T2 T1
T1
est moteur, on a W < 0, Q2 > 0, Q1 < 0, et son rendement est ηc = −W/Q2 = 1 − .
T2
b) η/ηc = X < 1. Le résultat est conforme au théorème de Carnot selon lequel le rendement
d’une machine ditherme est inférieur au rendement d’une machine de Carnot fonctionnant
(réversiblement) entre les mêmes températures, et n’est égal à ce rendement que si et seule-
ment si le cycle est réversible. Or, dans le cas présent, le cycle est irréversible, comme expliqué
au 1◦ ).
xxxxxxxxxxxxxxxxxxxxxxxxxxxx
VII - Cycle de Brayton
- A - Moteur thermique
 −1
∂T ∂S
  
1◦ ) Voir figure ci-après. A noter que = = T /Cp > 0.
∂S P ∂T P

2◦ ) La forme différentielle T dS est la quantité de chaleur reçue par le système étudié, lors
d’une transformation infinitésimale réversible. La valeur absolue de l’intégrale de cette forme
le long d’une courbe est la surface de la région du plan (T, S) se trouvant entre l’axe des
abcisses et ladite courbe et limitée par les deux parallèles à l’axe des ordonnées passant
respectivement par le point initial et par le point final de la courbe. L’aire d’un cycle de ce
diagramme représente donc, au signe près, la chaleur totale Q reçue par le système au cours
du cycle. Si ce dernier est parcouru dans le sens trigonométrique du diagramme entropique,

Christian Carimalo 31 TD de Thermodynamique - L3


on a Q = −W > 0 et le cycle est moteur, ce qui est le cas ici.

P T
2 3 3
P2

P1 1
1 4

V S

3◦ ) D’apr‘es ce qui vient d’être dit, Q12 = 0, Q23 > 0, Q34 = 0, Q41 < 0.
4◦ ) η = −W/Q23 avec −W = Q23 + Q41 , Q23 = ∆23 H = Cp (T3 − T2 ), Q41 = ∆41 H =
T4 − T1
Cp (T1 − T4 ). On en déduit η = 1 + Q14 /Q23 = 1 − .
T3 − T2
1− γ1
5◦ ) a) Dans les deux transformations isentropiques, on a la relation T = constante × P ,
1− 1
ce qui conduit aux relations T2 = T1 x, T3 = T4 x où x = r γ > 1.
1
c) η = 1 − .
x
1 T1 T4
6◦ ) x = , d’où T2 = , T3 = .
1−η 1−η 1−η

- B - Machine frigorifique
1◦ ) Les échanges énergétiques et entropiques de ce cycle sont les opposés de ceux du cycle
moteur précédent. On a donc Q14 > 0, Q43 = 0, Q32 < 0, Q21 = 0. Dans ce cycle, de
la chaleur (Q14 ) est extraite de la source froide, tandis qu’une quantité plus importante de
chaleur (|Q32 | = x Q14 > Q14 ) passe dans la source chaude. Ce transfert de chaleur ne
peut être réalisé qu’en fournissant le travail W 0 = −W > 0. C’est ainsi que fonctionne une
machine frigorifique ditherme.
1
2◦ ) a) et b) e = Q14 /W 0 = 1/ [−1 − Q32 /Q14 ], soit e = .
x−1
3◦ ) x = 1 + 1/e, d’où T2 = T1 (1 + 1/e), T3 = T4 (1 + 1/e).

- C - Etude du compresseur intervenant dans la transformation adiabatique

S i(t) S f (t)
Wc

Si Sf

S i (t+dt) S f (t+dt)

(a) (b)

1◦ ) Entre les dates t et t + dt, Σ a reçu le travail Wtot = nWc − Pi ∆Vi + Pf ∆Vf , où

Christian Carimalo 32 TD de Thermodynamique - L3


∆Vi est le volume compris entre les sections Si (t) et Si (t + dt), et ∆Vf le volume compris
entre les sections Sf (t) et Sf (t + dt), n le nombre de moles traversant le compresseur. Les
parois étant adiabatiques, son énergie interne a varié de ∆U = Wtot . Cette variation est la
différence entre l’énergie interne de la masse d’air comprise entre les sections Sf (t + dt) et
Si (t + dt), et l’énergie interne de la (même) masse d’air comprise entre les sections Sf (t) et
Si (t). Ces deux masses d’air ont en commun la partie située entre les sections Si (t + dt) et
Sf (t). En régime permanent, l’énergie interne de cette dernière n’évolue pas. On en déduit
que ∆U est la différence entre l’énergie interne de l’air Σi compris entre les sections Si (t)
et Si (t + dt), et l’énergie interne de l’air Σf compris entre les sections Sf (t) et Sf (t + dt),
la première occupant le volume ∆Vi , la seconde le volume ∆Vf . Les masses de Σi et Σf
sont manifestement égales, et correspondent exactement au nombre n de moles traversant
le compresseur. Σi étant sous les conditions (Pi , Ti ) et Σf sous les conditions (Pf , Tf ), on
a donc UΣi = nU (Pi , Ti ), UΣf = nU (Pf , Tf ) où U (P, T ) est l’énergie interne molaire de
l’air sous les conditions (P, T ). Notant V (P, T ) le volume molaire de l’air sous les conditions
(P, T ), on a ∆Vi = nV (Pi , Ti ), ∆Vf = nV (Pf , Tf ). On obtient ainsi

∆U = n [U (Pf , Tf ) − U (Pi , Ti )] = nWc − Pi nV (Pi , Ti ) + Pf nV (Pf , Tf ), soit

Wc = H(Pf , Tf ) − H(Pi , Ti )

où H(T, P ) est l’enthalpie molaire de l’air sous les conditions (P, T ).
2◦ ) Wc = Cp (Tf − Ti ).

- D - Application : machine frigorifique


e D
1◦ ) T1 = T2 = 190 K ; nombre de moles débitées par seconde : n = ; Q14 =
1+e M
M Mq
q = Cp (T4 − T1 ), d’où T4 = T1 + = 253, 1 K ; T3 = T4 (1 + 1/e) = 386, 3 K ;
D Cp D
1− γ1
r = 1 + 1/e, d’où r ' 4, 4 et P2 ' 4, 4 atm.
Pc Wc T3 − T4
2◦ ) = = ' 2, 11, donc Pc ' 2, 11 kW.
q Q14 T4 − T1

xxxxxxxxxxxxxxxxxxxxxxxxxxxx

Christian Carimalo 33 TD de Thermodynamique - L3


TD4

I - Etude thermodynamique d’un fil élastique (1)


On considère un fil élastique dont le comportement mécanique est analogue à celui d’un
ressort. Un état thermodynamique de ce fil est défini par la donnée de sa température T et
de son allongement ` ou de la force de tension f s’exerçant sur lui. Ces variables sont liées
par l’équation d’état f = −a`T 2 où a est une constante positive.
Lors d’une transformation infinitésimale réversible, le travail et la quantité de chaleur élémen-
taires reçus par le fil s’écrivent respectivement

d̄ W = −f d` et d̄ Q = C` dT + λd`

A - Détermination des coefficients calorimétriques et de l’entropie


1◦ ) En utilisant les expressions
 différentielles des premier et second principe de la Thermo-
∂C`

dynamique, déterminer λ et en fonction des variables d’état et de leurs dérivées
∂` T
partielles.
2◦ ) En déduire λ en fonction de T , ` et a.
3◦ ) Déterminer C` (`, T ) sachant qu’à allongement nul la capacité calorifique est donnée par
C` (0, T ) = bT 2 où b est une constante positive.
4◦ ) Déterminer la fonction d’état entropie et montrer qu’elle s’exprime selon :

bT 2
S(`, T ) = − a`2 T + S0
2

S0 étant une constante.

B - Transformation isotherme
On suspend une masse m à l’élastique dans le champ de gravitation terrestre de valeur
g = 9, 81 m s−2 . L’expérimentateur accompagne doucement la masse depuis la position
d’allongement nul (élastique au repos) jusqu’à la position d’équilibre `1 du système. La
transformation est donc supposée réversible et se fait en contact avec un thermostat de
température T0 (température de la pièce). Dans les questions qui suivent, on exprimera les
résultats en fonctions de m, g, T0 et a.
1◦ ) Déterminer `1 .
2◦ ) Calculer le travail W1 reçu par l’élastique.
3◦ ) Calculer la variation consécutive d’entropie ∆1 S de l’élastique. En déduire, en le justifiant,
la chaleur Q1 reçue par l’élastique.
(mg)2
4◦ ) Montrer que la variation d’énergie interne de l’élastique est ∆1 U = − .
2aT02

Christian Carimalo 34 TD de Thermodynamique - L3


C - Transformation monotherme irréversible
Cette fois, toujours dans une pièce à la température T0 , l’expérimentateur accroche la même
masse m à l’élastique initialement au repos (allongement nul) et la lâche sans la retenir.
Après quelques oscillations, la masse finit par s’immobiliser.
1◦ ) Montrer qu’une fois à l’équilibre, l’allongement du fil est la même qu’en B. Que peut-on
dire de la force de tension de l’élastique au cours de cette transformation ?
2◦ ) A l’aide du B, déterminer les variations d’énergie interne et d’entropie correspondantes
de l’élastique.
3◦ ) Déterminer le travail W2 , puis la quantité de chaleur Q2 reçus par l’élastique.
4◦ ) Les résultats obtenus sont-ils en accord avec le principe d’évolution ?

II - Etude d’un fil métallique (2)


A - Equation d’état
On considère un fil élastique de section constante s, dont la longueur est `0 à la température
T0et lorsqu’il n’est soumis à aucune force de traction (f = 0). Son module d’Young  E =
` ∂f 1 ∂`
 
et son coefficient de dilatation linéaire à force constante λ = sont
s ∂` T ` ∂T f
constants et positifs. Montrer que l’équation d’état du fil s’exprime comme

` = `0 exp [af + b(T − T0 )]

où a et b sont des constantes que l’on calculera.


B - Calculs des coefficients calorimétriques
Lors d’une transformation infinitésimale réversible où T , f et ` varient respectivement de dT ,
df et d`, la chaleur reçue par le fil est d̄ Q = C` dT + Ld` = Cf dT + kdf .
1◦ ) Définir les coefficients calorimétriques.
2◦ ) Par application du premier et du second principes de la Thermodynamique, calculer L
et k.
3◦ ) De quelles variables dépendent C` et Cf ? En admettant que dans le domaine de
température étudié on a C` = C0 = constante, déterminer Cf en fonction de C0 et des
données du problème.
C - Transformations
C1 - Transformation réversible isotherme
Elle s’effectue à la température atmosphérique constante T0 .
La force appliquée au fil varie de f = 0 à f = f0 et sa longueur devient `1 . Calculer :
a) le travail W0 et la chaleur Q0 reçus par le fil ;
b) la variation d’énergie interne du fil et montrer qu’elle se met sous la forme

(`1 − `0 )
∆0 U = f0 `1 − (1 − bT0 )
a

Christian Carimalo 35 TD de Thermodynamique - L3


c) la variation d’entropie ∆0 S du fil ;
d) la variation d’entropie ∆0 SΣ du système Σ formé par le fil et l’atmosphère. Conclure.

C2 - Transformation monotherme (température extérieure T0 )


Le fil étant à la température T0 et non soumis à une force de traction, la force f0 lui
est appliquée brutalement, tandis qu’il reste toujours en contact avec l’atmosphère. Après
établissement de l’équilibre, quels ont été
a) le travail W1 et la chaleur Q1 reçus par le fil ;
b) la variation d’énergie interne ∆1 U du fil ;
c) la variation d’entropie ∆1 S du fil ;
d) la variation d’entropie ∆1 SΣ du système isolé Σ comprenant le fil et l’atmosphère. Conclure

C3 - Transformations à forces constantes


1◦ ) Le fil étant soumis à la force constante f0 et se trouvant à la température T0 est isolé de
l’atmosphère. Il est ensuite chauffé jusqu’à ce que sa température devienne T1 . Sa longueur
devient égale à `2 . Calculer :
a) le travail W2 et la chaleur Q2 reçus par le fil ;
b) la variation d’énergie interne ∆2 U et d’entropie ∆2 S du fil.
2◦ ) Le fil se trouve à la température T1 et est soumis à la force de traction f0 . On supprime
brutalement cette force et on remet le fil en contact avec l’amosphère à la température T0 .
Après équilibre final, quels ont été
a) le travail W3 et la chaleur Q3 reçus par le fil ;
b) la variation d’énergie interne ∆3 U et d’entropie ∆3 S du fil.

III - Etude d’un échantillon de caoutchouc


Une tige de caoutchouc de longueur ` et à la température T est soumis à une force de
contrainte colinéaire à la tige. Les dimensions transversales de cet échantillon sont supposées
très petites devant la longueur `. Dans ces conditions, on ne s’intéressera qu’aux variations
de longueur de la tige. La force f est donnée par la relation
!
` `2
f = AT − 02
`0 `

où A est une constante positive et `0 la longueur de l’échantillon en l’absence de contrainte,


à la température T .
1 d`0
Soit α0 = le coefficient de dilatation thermique de l’échantillon, non soumis à
`0 dT
contrainte. Il est indépendant de la température, positif, et tel que α0 T  1.
1◦ ) La chaleur infinitésimale d¯Q reçue par l’échantillon au cours d’une transformation
infinitésimale réversible pour laquelle T et ` varient respectivement de dT et d` s’écrit d̄ Q =
C` dT + λd`.

Christian Carimalo 36 TD de Thermodynamique - L3


où C` est la capacité calorifique à longueur constante et λ un coefficient calorimétrique.
a) Déterminer λ.
b) Montrer qu’on peut en déduire l’expression de l’entropie S(`, T ) à une fonction de T près.
Pour quelle valeur `m de ` la fonction S présente-t-elle un extremum, la température étant
fixée ? Donner une valeur approchée de `m .
∂S
 
c) Quelle est la signification physique de ? Interpréter son signe.
∂` T

d) Déterminer l’expression de l’énergie interne U (`, T ), à une fonction de la température près.


2◦ ) A température constante T0 , on allonge réversiblement le fil depuis la longueur `0 jusque
la longueur 2`0 .
a) Calculer le travail W reçu par le fil.
b) Quelle a été la variation d’énergie interne ∆U du fil ?
c) En déduire la chaleur Q reçue par le fil. Retrouver ce résultat à partir de l’expression de
l’entropie.
3◦ ) On considère maintenant l’allongement du fil de la longueur `0 à la longueur 2`0 ef-
fectué de façon irréversible en appliquant une force de contrainte constante pendant toute la
transformation. Le fil est en contact avec l’atmosphère de température T0 , jouant le rôle de
thermostat.
a) Déterminer le travail W 0 et la chaleur Q0 reçus par le fil.
b) Comparer Q0 à Q. Conclure en utilisant le second principe de la Thermodynamique.
4◦ ) Etablir,
en fonction des données, l’expression du coefficient de dilatation thermique
∂`

αf = de l’échantillon soumis à une force constante. On pourra utiliser la variable
∂T f
sans dimension x = `/`0 .
a) Pour quelle valeur de ` a-t-on αf = 0 ? La comparer à `m .
∂f ∂S
   
b) Montrer que a le signe de −αf et comparer alors le signe de à celui de
∂T ` ∂` T
αf .
5◦ ) L’ensemble des informations obtenues sur la tige correspond-il à ce que l’on peut attendre
de l’application des principes de la Thermodynamique ?

N.B. Contrainte signifie une compression ou une traction.

IV - Feuille de caoutchouc
A/ Une feuille de caoutchouc homogène et dont on néglige les variations d’épaisseur est
soumise à des efforts uniformément répartis dans toutes les directions de sa surface. Ceux-ci
provoquent une extension de la surface A de la feuille.
On admettra que lorsque sa surface varie de dA, le travail élémentaire reçu par la feuille
est donné par d¯W = σ dA, où σ, qui représente le travail par unité de surface des efforts
s’exerçant sur elle, dépend de A et de la température T par la relation σ = a T /A, a étant
une constante positive.

Christian Carimalo 37 TD de Thermodynamique - L3


La chaleur infinitésimale reçue par la feuille dans une transformation infinitésimale réversible
où la température et la surface varient respectivement de dT et dA sera exprimée sous la
forme d̄ Q = CA dT + λ dA, CA et λ étant deux coefficients calorimétriques.
1◦ ) Nommer le coefficient CA et en donner l’interpétation physique.
2◦ ) Déterminer λ. Montrer que CA ne dépend pas de la surface A. On supposera dans la suite
que CA ne dépend pas de la température dans le domaine exploré et l’on posera simplement
CA = C0 .
3◦ ) Déterminer :
a) l’énergie interne U (T, A) de la feuille. Quelle propriété possède cette fonction ?
b) l’entropie S(T, A) de la feuille.
B/ Un ballon shérique est constitué par une feuille de caoutchouc identique à celle étudiée en
A/. Le ballon est gonflé avec de l’hélium considéré comme un gaz parfait. A la température
T et sous la pression Pa de l’atmosphère, le ballon a pour rayon r. L’épaisseur de la feuille de
caoutchouc est négligeable à la fois devant r et les variations de r. On note respectivement
n, P, et Cv le nombre de mole, la pression et la capacité calorifique à volume constant de
l’hélium.
1◦ ) Exprimer la quantité de chaleur d̄ Qh reçue par l’hélium au cours d’une transformation
infinitésimale réversible où la température et le rayon du ballon varient respectivement de dT
et de dr. On l’exprimera en fonction de Cv , P , r, dT et dr.
2◦ ) On considère le système Σ comprenant à la fois le ballon et l’hélium qu’il contient. Pour
la transformation envisagée à la question précédente, exprimer :
a) le travail élémentaire d̄WΣ reçu par le système Σ soumis à la pression extérieure constante
Pa ;
b) la chaleur élémentaire d̄ QΣ = d̄ Q + d̄ Qh reçue par Σ.
3◦ ) a) Déterminer la différence de pression P − Pa en exprimant de deux manières la
différentielle dUΣ de l’énergie interne du système global Σ.
b) Exprimer P en fonction de n, T et r et, remarquant que l’on doit avoir Pa ≥ 0, déduire
du résultat du 3◦ ) a) qu’un équilibre ne peut être réalisé que si le ballon contient un nombre
minimum de mole d’hélium, nombre que l’on exprimera en fonction de a et R, constante des
gaz parfaits.
4◦ ) L’atmosphère est considérée comme une source de chaleur de température constante T0 .
On suppose quà cette température, la pression extérieure diminue lentement de la valeur P1
à la valeur P2 . Le rayon du ballon qui contient N > nm moles d’hélium passe alors de la
valeur r1 à la valeur r2 .
a) Donner les expressions de r1 et de r2 en fonction de P1 , P2 et des données.
b) Déterminer le travail W et la chaleur Q reçus par Σ en fonction de N , T0 , r1 et r2 .
Préciser leurs signes.
5◦ ) On suppose maintenant qu’à la même température T0 , la pression extérieure diminue de
façon quasi instantanée de P1 à P2 . Le rayon du ballon évolue alors irréversiblement de r1 à
r2 . Déterminer le travail W 0 et la chaleur Q0 reçus par Σ dans cette nouvelle transformation,
tout en précisant leurs signes.

Christian Carimalo 38 TD de Thermodynamique - L3


6◦ ) Comparer W 0 à W . Montrer que ce résultat est compatible avec l’énoncé de Kelvin du
second principe de la Thermodynamique.

V - Pile hydroélectrique
On considère une pile hydroélectrique qui n’échange avec le monde extérieur que de l’énergie
électrique et de la chaleur. Le volume V et la pression P sont constants.
On rappelle que le travail électrique reçu par la pile dans une transformation infinitésimale
où la charge varie de dq est d̄ W = E dq, E étant la fem de la pile. Lorsqu’on charge la pile,
dq est positif ; lorsque la pile débite, dq est négatif.
On note Cq (q, T ) la capacité calorifique à charge constante de la pile et a(q, T ) sa chaleur
de charge isotherme.
On suppose que la fem E(q, T ) de la pile est donnée par

E(q, T ) = E(T ) = E0 [1 + α(T − T0 )]

où E0 et α sont des constantes positives.


A/ 1◦ ) Calculer a(q, T ) en fonction de E0 , α et T .
2◦ ) Montrer que Cq ne dépend que de T .
3◦ ) En déduire l’expresssion des différentielles de l’énergie interne et de l’entropie de la pile.
B/ On charge la pile de façon réversible et isotherme à la température T0 . Sa charge varie
de ∆q. Calculer :
1◦ ) le travail électrique W et la chaleur Q reçus par la pile ;
2◦ ) les variations d’énergie interne ∆1 U et d’entropie ∆1 S correspondantes.
3◦ ) Calculer numériquement W, Q, ∆1 U et ∆1 S. On donne E0 = 0, 5 V ; α = 2 10−4 K−1 ;
T0 = 300 K ; ∆q = 100 C.
C/ On suppose Cq constant. La pile est isolée thermiquement. Elle se trouve initialement dans
l’état (q0 , T0 ). On fait alors débiter la pile de façon irréversible à travers une résistance R.
Un courant d’intensité I traverse cette résistance pendant une durée ∆t. La charge débitée
est alors ∆q = −I∆t. On note r la résistance interne de la pile.
1◦ ) Quelle énergie la pile a-t-elle fournie à la résistance R ?
2◦ ) a) En déduire la variation ∆2 U d’énergie interne de la pile.
I
b) Montrer que la variation ∆T de sa température s’écrit ∆T = ∆t [rI − αE0 T0 ].
Cq
c) Calculer numériquement ∆T . On donne r = 10 Ω, Cq = 4, 18 J K−1 , I = 2 A, ∆t = 50 s.
3◦ ) Exprimer la variation d’entropie ∆2 S de la pile et en préciser le signe. On tiendra compte
du fait que ∆T /T0  1. Calculer numériquement ∆2 S.

VI - Etude thermodynamique d’un composé paramagnétique


Les paramètres extensifs d’un système paramagnétique sont l’énergie interne U et l’aimanta-
tion par unité de volume M .

Christian Carimalo 39 TD de Thermodynamique - L3


1◦ ) Donner les paramètres intensifs conjugués de M et S, l’entropie.
On définit des coefficients calorimétriques au moyen des différentielles

T dS = CM dT + `dM = CB dT + h dB

B étant le champ magnétique appliqué.


2◦ ) Calculer :
∂CM
 
a) ` et en fonction de T et des dérivées partielles de B(T, M ) par rapport à T .
∂M T
∂CB
 
b) h et en fonction de T et des dérivées partielles de M (T, B) par rapport à T .
∂B T

3◦ ) Montrer que :
∂U ∂M
   
a) =h+B ;
∂B T ∂B T
∂T h
 
b) =− ;
∂B S CB
 2
∂B ∂M
  
c) CB − CM = T .
∂M T ∂T B
C B
4◦ ) Application. Le composé a pour équation d’état M = . Dans cette expression, µ0
µ0 T
est la perméabilité du vide et C une constante
 positive appelée
  constante
  deCurie. Calculer,
∂CM ∂CB ∂U ∂T
 
en fonction de T, M, C, et CB : `, h, , , , et CB − CM .
∂M T ∂B T ∂B T ∂B S
Quelles propriétés simples de U et de CB − CM peut-on déduire ?
5◦ ) En admettant que CB > 0, comment varie la température du composé lorsqu’on diminue
B à entropie constante ?

Christian Carimalo 40 TD de Thermodynamique - L3


Corrigé TD4

I - Etude thermodynamique d’un fil élastique (1)

A - Détermination des coefficients calorimétriques et de l’entropie


1◦ ) et 2◦ ) S étant l’entropie, U l’énergie interne, définissons la fonction d’étatF =U − T
S
∂f ∂S

dont la différentielle dF = −f d` − SdT est une d.t.e. Par conséquent : = .
 ∂T ` ∂` T
C` λ ∂S λ ∂f
 
Comme dS = d̄ Q/T = dT + d`, il vient = , d’où λ = T = −2a`T 2 .
T T ∂` T T ∂T `
∂C` ∂λ/T
  
Exprimant que dS est une d.t.e., on trouve =T = −2a`T
∂` T ∂T `
∂C`
 
3◦ ) L’intégration de = −2a`T donne C` (`, T ) = −a`2 T + φ(T ). La condition
∂` T
C` (0, T ) = bT 2 , on trouve φ(T ) = bT 2 et finalement C` (`, T ) = −a`2 T + bT 2 .
" #
bT 2 bT 2
4◦ ) −a`2 − a`2 T , d’où S = − a`2 T + S0 où S0
 
dS = + bT dT − 2a`T d` = d
2 2
est une constante.

B - Transformation isotherme
mg
1◦ ) mg + f1 = 0, soit `1 = .
aT02
`21 m2 g 2
Z `1
2◦ ) W1 = Wopérateur = − f d` = aT02 = .
0 2 2aT02
m2 g 2
3◦ ) ∆1 S = −aT0 `21 = − . La transformation est isotherme (donc réversible), on peut
aT03
m2 g 2
donc écrire Q1 = T0 ∆1 S = − .
aT02
m2 g 2
4◦ ) ∆1 U = W1 + Q1 = − .
2aT02
C - Transformation monotherme irréversible
1◦ ) L’équilibre final est le même que celui de la transformation précédente, car tant qu’on
ne sort pas du domaine d’élasticité du fil, l’équation d’état f = −a`T 2 reste valable et
l’allongement du fil, défini par mg + f = 0, est le même. Par contre, la force de tension,
qui n’est bien définie que dans les états d’équilibre du fil, ou dans une succession de tels
états (transformation réversible), ne l’est pas dans une transformation irréversible non quasi-
statique comme celle-ci.
m2 g 2 m2 g 2
2◦ ) ∆2 U = ∆1 U = − ; ∆ 2 S = ∆1 S = − (même état initial et même état final).
2aT02 aT03
m2 g 2 3m2 g 2
3◦ ) W2 = mg`1 = , Q2 = ∆2 U − W 2 = − .
aT02 2aT02

Christian Carimalo 41 TD de Thermodynamique - L3


4◦ ) La variation d’entropie du système, isolé thermiquement, comprenant le fil et le thermo-
Q2 m2 g 2
stat est ∆2 S − = > 0 : elle est strictement positive, en accord avec le principe
T0 2aT03
d’évolution.
xxxxxxxxxxxxxxxxxxxxxxxxxxxx

II - Etude d’un fil métallique (2)

A - Equation d’état
1 ∂` 1 ∂` 1
   
d ln ` = df + dT = df + λdT , d’où, en intégrant :
` ∂f T ` ∂T f Es
` f 1
 
ln = + λ(T − T0 ), soit ` = `0 exp [af + b(T − T0 )] où a = et b = λ.
`0 Es Es
B - Calculs des coefficients calorimétriques
1◦ )
1
2◦ ) dU = C` dT +(L+f )d`, dH = d(U −f `) = Cf dT +(k −`)df , dS = [C` dT + Ld`] =
T
1
[Cf dT + kdf ], dF = d(U − T S) = −SdT + f d`, dG = d(F − f `) = −SdT − `df , sont
T
des d.t.e.
∂S L ∂f b
   
• A partir de dF et dS : = =− ; d’où L = T .
∂` T T ∂T ` a
∂S k ∂`
   
• A partir de dG et dS : = = ; d’où k = bT `.
∂f T T ∂T f

1 ∂C` ∂L/T 1 ∂Cf


     
3◦ )A partir de dS, = = 0 : C` ne dépend que de T . Puis =
T ∂` T ∂T ` T ∂f T
∂k/T
 
= b2 ` 6= 0 : Cf dépend de T et de f ou `. Exprimons ensuite la différence Cf −C` :
∂T f
b2
d̄ Q = Cf dT + kdf = C` dT + L` [adf + bdT ] , d0 où Cf − C` = b`L = ` T
a
b2
soit, dans le domaine de température étudié, Cf = C0 + ` T .
a
C - Transformations
C1 - Transformation réversible isotherme
1 ` `1 Z
` 0 x1
Z
`0
a) Pour T = T0 , f = ln , d’où W0 = f d` = dx ln x = [x ln x − x]x1 1 où
a `0 `0 a 1  a
`0 1 `1

x1 = `1 /`0 . On obtient : W0 = [x1 ln x1 + 1 − x1 ] = `1 ln + `0 − `1 = `1 f0 −
Z `1
a a `0
`1 − `0 b
. Puis : Q0 = Ld` = T0 [`1 − `0 ],
a `0 a
(`1 − `0 )
b) ∆0 U = W0 + Q0 = f0 `1 − (1 − bT0 )
a
Q0 b
c) ∆0 S = = [`1 − `0 ].
T0 a

Christian Carimalo 42 TD de Thermodynamique - L3


Q0
d) ∆0 SΣ = ∆0 S − = 0 : cette variation est nulle car pour le système Σ, il s’agit d’une
T0
transformation adiabatique réversible (conforme au principe d’évolution).

C2 - Transformation monotherme (température extérieure T0 )


a) b) et c) Les états initial et final étant les mêmes que les précedents, ∆1 U = ∆0 U ,
(`1 − `0 )
∆1 S = ∆0 S. Comme W1 = f0 (`1 − `0 ), on a Q1 = ∆1 U − W1 = f0 `0 − (1 − bT0 ).
a
Q1 Q0 − Q1 W1 − W0 `0
d) ∆1 SΣ = ∆1 S − = = = [x1 − 1 − ln x1 ]. Or, φ(x1 ) =
T0 T0 T0 aT0
x1 − 1 − ln x1 est positif pour tout x1 > 0 et nul si et seulement si x1 = 1. Ainsi, ∆1 SΣ > 0,
ce qui est conforme au principe d’évolution, s’agissant pour Σ d’une adiabatique irréversible.

C3 - Transformations à forces constantes


Z T1 " #
b2
Z T1
1◦ )a) W2 = f0 (`2 − `1 ) ; Q2 = Cf dT = C0 + ` T dT . Pour f = f0 , on a
T0 T0 a
1 ` d` b `2 1 `
Z  
T = T0 + ln , `dT = . D’où Q2 = C0 (T1 − T0 ) + T0 + ln d`, soit
b `1 b a `1 b `1
bT0 `1 ` /` bT0
Q2 = C0 (T1 − T0 ) + (`2 − `1 ) + [x ln x − x]12 1 = C0 (T1 − T0 ) + (`2 − `1 ) +
 a a a
1 `2 1 `2

`2 ln − `2 + `1 . Notant que T1 − T0 = ln , il vient finalement
a `1 b `1
`2 `1
Q2 = C0 (T1 − T0 ) + [ bT1 − 1 ] − [ bT0 − 1 ]
a a
.
`2 `1
b) ∆2 U = W2 + Q2 = C0 (T1 − T0 ) + [ bT1 − 1 ] − [ bT0 − 1 ] + f0 (`2 − `1 ) ;
Z T1 a a
dT T1 b
∆2 S = Cf = C0 ln + (`2 − `1 ).
T0 T T0 a
2◦ ) Le fil se trouve à la température T1 et est soumis à la force de traction f0 . On supprime
brutalement cette force et on remet le fil en contact avec l’amosphère à la température T0 .
Après équilibre final, quels ont été
a) W3 = 0 ; Q3 = ∆3 U .
b) Comme on revient à l’état (f = 0, T = T0 ), on a ∆3 U = −∆2 U − ∆0 U , ∆3 S =
−∆2 S − ∆0 S. D’où
`2 `0
∆3 U = Q3 = −C0 (T1 − T0 ) − [ bT1 − 1 ] + (bT0 − 1) − f0 `2 ;
a a
b T1
∆3 S = − [`2 − `0 ] − C0 ln .
a T0

xxxxxxxxxxxxxxxxxxxxxxxxxxxx

Christian Carimalo 43 TD de Thermodynamique - L3


III - Etude d’un échantillon de caoutchouc

1
1◦ ) a) d¯W = f d`, dU = d¯Q + d¯W = C` dT + [λ + f ] d`, dS = [C` dT + λd`],
 T 
∂S ∂f λ
 
dF = d(U −T S) = −SdT +f d` ; dF étant une d.t.e., on doit avoir =− = ,
" # ∂` T ∂T ` T

∂f

` `2
soit λ = −T = −f + Aα0 T 2 + 2 02 .
∂T ` `0 `
∂S 1
   
b) = −A x [1 − α0 T ] − 2 [1 + 2α0 T ] où x = `/`0 . Par intégration de cette
∂` T x
relation par rapport à ` (T étant fixé), on obtient
" #
x2 1
S(`, T ) = −A`0 [1 − α0 T ] + [1 + 2α0 T ] + Φs (T )
2 x

où Φs (T ) est une fonction de T . Considérée comme une fonction de `, T étant fixé, cette
expression présente un extremum (en fait un maximum, puisque A > 0) pour
1
1 + 2α0 T

3
x = xm = ' 1 + α0 T
1 − α0 T
`0
Notons ici que, α0 étant constant, on a ln `0 = α0 T +constante, soit ln = 1+α0 (T −T0 ),
L0
où L0 = `(f = 0, T = T0 ). Comme α0 T  1, on peut utiliser l’approximation `0 '
L0 [1 + α0 (T − T0 )]. On obtient ainsi `m = `0 xm ' L0 [1 + α0 (2T − T0 )].
∂S λ
 
c) = , où λ est la chaleur reçue par l’échantillon dans une transformation isotherme,
∂` T T
lorsque sa longueur a augmenté d’une unité de longueur. Puisque A > 0, lorsque ` > `m on
a λ < 0 (l’échantillon fournit de la chaleur au thermostat), tandis que λ > 0 pour ` < `m
(l’échantillon absorbe de la chaleur).
" # " #
∂U ` `2 x2 2
 
d) = λ + f = Aα0 T 2 + 2 02 , d’où U (`, T ) = Aα0 T 2 `0 − + Φu (T ),
∂` T `0 ` 2 x
Φu (T ) étant une fonction de la température.
2◦ ) a) W = AT0 `0 .
5A`0 α0 T02
b) ∆U = .
2
5A`0 α0 T02
c) Q = ∆U −W = −AT0 `0 . La transformation étant isotherme, on a Q = T0 ∆S.
2
7AT0 7AT0
3◦ ) a) La force appliquée est f 0 = f (` = 2`0 , T = T0 ) = . D’où W 0 = `0 . Les
4 4
états initial et final de cette transformation sont les mêmes que ceux de la transformation
précédente, donc ∆0 U = ∆U et ∆0 S = ∆S, et Q0 = ∆U − W 0 .
b) La variation d’entropie de l’ensemble, isolé thermiquement, comprenant l’échantillon et le
Q0 Q − Q0 W0 − W 3A`0
thermostat est ∆0 Stot = ∆0 S − = = = > 0 : elle est strictement
T0 T0 T0 4
positive, conformément au principe d’évolution.

Christian Carimalo 44 TD de Thermodynamique - L3


 −1
∂` ∂f ∂f ∂f AT 2
        
4◦ ) On a =− , avec = 1 + 3 > 0. D’où
∂T f ∂T ` ∂` T ∂` T `0 x
`0 x3
− x3m
αf = x(1 − α0 T )
T 1 + 2x3

a) αf = 0 pour ` = `m .
∂S
   
b) Evident d’après ce qui précède. On a αf > 0.
∂` T

5◦ ) Réponse laissée au lecteur.

xxxxxxxxxxxxxxxxxxxxxxxxxxxx

IV - Feuille de caoutchouc
-A-
1◦ ) CA est la capacité calorifique à aire constante.
1
2◦ ) dU = CA dT + (σ + λ)dA, dS = [CA dT + λ dA] ; dF = d(U − T S) = −SdT + σdA
 T 
∂S ∂σ λ
 
est une d.t.e., donc : =− = , soit λ = −σ ; dU = CA dT + (σ + λ)dA ≡
∂A T ∂T A T
CA dT est une d.t.e., donc CA ne peut dépendre que de la température.
3◦ ) a) L’énergie interne ne dépend que de la température : U = U (T ) = C0 T + constanteu .
b) S(T, A) = C0 ln T − a ln A + constantes .

-B-

1◦ ) d̄ Qh = Cv dT + P dV = Cv dT + 4πr2 P dr.
2◦ ) a) d̄ WΣ = −Pa dV (dV = 4πr2 dr) ;
b) d̄ QΣ = d̄ Q + d̄ Qh = (C0 + Cv )dT + P dV − σdA (dA = 8πrdr).
3◦ ) a) dUΣ = (C0 + Cv )dT = d¯WΣ + d¯QΣ = (C0 + Cv )dT + (P − Pa )dV − σdA, d’où

P − Pa = .
r
b) P = nRT /V ; de la condition Pa ≥ 0 on déduit qu’un équilibre où le ballon a un rayon r

à la température T ne peut être réalisé que si P ≥ , ce qui impose que le ballon contienne
r
2a
un nombre minimum de mole d’hélium égal à nm = , lequel est trouvé ici, dans le cadre
3R
du modèle proposé, indépendant à la fois de T et de r.
4◦ ) L’atmosphère est considérée comme une source de chaleur de température constante T0 .
On suppose quà cette température, la pression extérieure diminue lentement de la valeur P1
à la valeur P2 . Le rayon du ballon qui contient N > nm moles d’hélium passe alors de la
valeur r1 à la valeur r2 .
1
3RT0

3
a) ri = (N − nm ) , (i = 1, 2), N > nm . On a r2 > r1 .
4πPi

Christian Carimalo 45 TD de Thermodynamique - L3


Z V2
r2 3RT0
b) W = − Pext dV = −4πP1 r13 ln
< 0, avec P1 r13 = P2 r23 = (N − nm ). La
V1 r1 4π
transformation étant telle que Tf = Ti = T0 , on a ∆UΣ = 0 et Q = −W > 0.
5◦ ) W 0 = −P2 (V2 − V1 ) = (P2 − P1 )V1 < 0. On a encore ∆UΣ = 0 et Q0 = −W 0 > 0.
P1 P2 P2
6◦ ) On a W = −P1 V1 ln , et W 0 − W = (P2 − P1 )V1 − P1 V1 ln = P1 V1 φ( ), avec
P2 P1 P1
φ(x) = x − 1 − ln x. Or, φ(x) est positif pour tout x > 0 et nul si et seulement si x = 1. Par
conséquent, W 0 − W > 0. Ce résultat est compatible avec l’énoncé de Kelvin. En effet, la
différence W 0 −W représente le travail total w reçu par Σ dans le cycle constitué de la seconde
transformation irréversible du 5◦ ), suivi de la transformation réversible inverse de celle du
4◦ ). Alors qu’il n’est en contact qu’avec la seule source de chaleur qu’est l’atmosphère, le
système Σ décrivant un cycle de transformations ne peut que recevoir du travail : w > 0.

xxxxxxxxxxxxxxxxxxxxxxxxxxxx
V - Pile hydroélectrique

-A-
1
1◦ ) dU = Cq dT + (a + E)dq, dS = [Cq dT + adq] ; dF = d(U − T S) = −SdT + Edq
T
∂S ∂E a
   
est une d.t.e., d’où =− = = −αE0 , soit a = −αE0 T .
∂q T ∂T q T
∂Cq ∂[a + E]
   
2◦ ) dU étant une d.t.e., on doit avoir = . Mais a+E = E0 (1−αT0 ) =
∂q T ∂q T
∂Cq
 
constante. Donc = 0 et Cq est fonction de T uniquement.
∂q T

Cq (T )
3◦ ) dU = Cq (T )dT + E0 (1 − αT0 )dq, dS = dT − αE0 dq.
T
-B-
1◦ ) W = E0 ∆q ; Q = T0 ∆S = −αE0 T0 ∆q.
2◦ ) ∆1 U = E0 (1 − αT0 )∆q ; ∆1 S = −αE0 ∆q.
3◦ ) W = 50 J ; Q = −3 J ; ∆1 U = 47 J ; ∆1 S = −10−2 J/K.

-C-
C/ On suppose Cq constant. La pile est isolée thermiquement. Elle se trouve initialement dans
l’état (q0 , T0 ). On fait alors débiter la pile de façon irréversible à travers une résistance R.
Un courant d’intensité I traverse cette résistance pendant une durée ∆t. La charge débitée
est alors ∆q = −I∆t. On note r la résistance interne de la pile.
1◦ ) W 0 = RI 2 ∆t.
2◦ ) a) ∆2 U = −W 0 = RI∆q = Cq ∆T + E0 (1 − αT0 )∆q.
b) A intensité de courant constante (loi des mailles) : E = (R + r)I = constante = E0 , et
∆q I
E0 − RI = rI, d’où ∆T = [RI − E0 + αE0 T0 ] = ∆t [rI − αE0 T0 ].
Cq Cq
c) ∆T = 4, 8 K.

Christian Carimalo 46 TD de Thermodynamique - L3


T0 + ∆T 1 rI 2 ∆t 20
3◦ ) ∆2 S = Cq ln − αE0 ∆q ' [Cq ∆T − αE0 T0 ∆q] = = J/K > 0.
T0 T0 T0 3
xxxxxxxxxxxxxxxxxxxxxxxxxxxx

VI - Etude thermodynamique d’un composé paramagnétique

1◦ ) B et T , respectivement.
2◦ ) a) dU = CM dT + (` + B)dM ; dF = d(U − T S) = BdM − SdT et dS!sont des d.t.e.,
∂S ∂B ` ∂CM ∂`/T ∂2B
       
d’où =− = , =T = −T .
∂M T ∂T M T ∂M T ∂T M ∂T 2 M
∂S ∂M
   
b) dG = d(F − M B)) = −SdT − M dB est une d.t.e., d’où = =
! ∂B T ∂T B
h ∂CB ∂h/T ∂2M
   
, =T =T .
T ∂B T ∂T M ∂T 2 B
∂U ∂S ∂M ∂M
       
3◦ ) a) dU = T dS + BdM , d’où =T +B =h+B .
∂B T ∂B T ∂B T ∂B T
∂T h
 
b) =− : évident !
∂B S CB
∂M ∂M
   
c) Utilisant l’égalité CM dT +`dM = CB dT +hdB, écrivant dM = dB, dT +
B ∂B
 T ∂T
∂M
puis identifiant les facteurs des différentielles, on obtient notamment CB − CM = ` .
∂T B
L’existence d’une équation d’état Φ(B, T, M ) = 0 permet aussi d’obtenir la relation
∂B ∂T ∂M ∂B ∂M ∂B
           
= −1 ou =− . Compte tenu de l’ex-
∂T M ∂M B ∂B T ∂T M ∂T B ∂M T
pression de ` obtenue au 2◦ ) a), on en déduit ainsi
 2
∂B ∂M
  
CB − CM = T
∂M T ∂T B

4◦ ) ` = −B, donc CM ainsi que U ne sont  fonctions que de la température ; h = −M et


∂T M µ0 2

CB n’est fonction que de la température ; = ; CB − CM = M ne dépend
∂B S CB C
que de M .
∂T M
 
5◦ ) = > 0 : T diminue lorsque B diminue (refroidissement par désaimantation
∂B S CB
adiabatique).

xxxxxxxxxxxxxxxxxxxxxxxxxxxx

Christian Carimalo 47 TD de Thermodynamique - L3


TD5

Détente de Joule-Gay-Lussac, détente de Joule-Thomson

I - Gaz parfait, gaz de Van der Waals


On considère, d’une part, une mole d’un gaz parfait G0 d’équation d’état P V = RT et,
d’autre part, une mole d’un gaz de Van der Waals G d’équation d’état

a a b
 
P+ (V − b) = RT avec  1 et 1
V2 PV 2 V

où P est la pression, V le volume molaire, T la température, R la constante des gaz parfais,
a et b des constantes ne dépendant que de la nature du gaz.
1◦ ) Déterminer les coefficients thermoélastiques α, β et χT pour chacun des deux gaz G0 et
G.
2◦ ) Pour les deux gaz, déduire le coefficient ` correspondant et montrer que dans chaque
cas, Cv ne dépend pas du volume. Dans la suite, on admettra que le Cv du gaz G ne dépend
pas de la température dans le domaine exploré et a la même expression que celui d’un gaz
parfait diatomique.
3◦ ) Déterminer, pour les deux gaz et en fonction de V et T leurs fonctions d’état : énergie
interne U et entropie S.
4◦ ) Déterminer pour chacun des deux gaz sa fonction d’état enthalpie H. Pour le gaz G, on
donnera une expression approchée de H en fonction des variables indépendantes P et T en
utilisant un développement limité à l’ordre 1.
5◦ ) Pour les deux gaz, déterminer l’équation des isentropiques liant P et V .

II - Détente de Joule-Gay-Lussac
Un récipient à parois adiabatiques, de volume 2V , est divisé en deux compartiments de
volume V pouvant communiquer par l’ouverture d’un robinet. Au début de l’expérience, l’un
des compartiments est vide et l’autre contient une mole d’un gaz à la température Ti . On
ouvre le robinet (sa manœuvre ne fournit aucun travail) et on attend que l’équilibre thermique
soit établi, la température d’équilibre du gaz étant alors Tf . Le gaz peut être l’un ou l’autre
des deux gaz considérés en I.
1◦ ) Pour chacun des deux gaz, donner la variation de température ∆T = Tf − Ti correspon-
dante en admettant que Cv soit constant. Quel est le signe de cette variation ?
2◦ ) Application numérique. Calculer ∆T pour l’azote à l’aide des données suivantes : R =
8, 31 J K−1 mole−1 ; V = 1 ` ; Ti = 298 K ; Cv = 20 J K−1 mole−1 ; a = 0, 14 J m3 mole−2 ;
b = 3, 9 10−5 m3 mole−1 .

Christian Carimalo 48 TD de Thermodynamique - L3


III - Détente de Joule-Thomson
Un tube dont les parois extérieures sont adiabatiques est équipé d’un bouchon poreux. En
amont de ce bouchon, on introduit un gaz de Van der Waals, sous une pression maintenue
constante P1 et à température constante T1 . En aval, la pression et la température sont
maintenues constantes et égales respectivement à P2 et T2
1◦ ) Calculer la variation d’énergie interne d’une mole de gaz traversant le bouchon poreux.
Quelle est la fonction d’état qui n’est pas affectée par cette transformation ?
2◦ ) Le coefficient d’élévation de température, ou coefficient de Joule-Thomson, est défini
par

∂T
 
τ=
∂P H

Exprimer τ en fonction de Cp et le coefficient de dilatation isobare α. Quelle est la valeur de


τ pour un gaz parfait ? Discuter pour un gaz quelconque.
3◦ ) Pour le gaz de Van der Waals, exprimer τ en fonction de V, a, b, R, Cp et T .
4◦ ) Quelle est la région du plan (P, T ) pour laquelle une détente s’accompagne d’un refroi-
dissement ?
5◦ ) Calculer la température au dessus de laquelle il est impossible de refroidir le gaz par
détente isenthalpique.
6◦ ) Application numérique. Calculer cette température limite pour
• l’azote (a = 0, 14 J m3 mole−2 ; b = 3, 9 10−5 m3 mole−1 ) ;
• l’hydrogène (a = 2, 48 10−2 J m3 mole−2 ; b = 2, 66 10−5 m3 mole−1 ) ;
• l’hélium (a = 3, 45 10−3 J m3 mole−2 ; b = 2, 4 10−5 m3 mole−1 ) .
7◦ ) Que se passe-t-il pour une fuite de chacun de ces gaz à la température ordinaire (' 298
K) ?

Christian Carimalo 49 TD de Thermodynamique - L3


Corrigé TD5

I - Gaz parfait, gaz de Van der Waals

1 ∂V 1 ∂P 1 ∂V
     
1◦ ) α= ;β= ; χT = − ;
V ∂T P P ∂T V V ∂P T
1 1
• Gaz parfait : α = β =, χT = ;
T P
• Gaz de Van der Waals :
 −1 " 2 #−1
1 ∂T 1 b 2a b
  
α= = 1− 1− 1−
V ∂V P T V RT V V
1 a 1α
 
β= 1+ 2
, χT =
T PV pβ

∂P
 
2◦ ) En appliquant les deux principes de la Thermodynamique : ` = T = T P β.
∂T V
• Gaz parfait : ` = P ;
a
• Gaz de Van der Waals : ` = P + ;
V2
∂Cv ∂(` − P )
   
dU étant une d.t.e., = . Dans un cas comme dans l’autre, cette
∂V T ∂T V
dérivée partielle est nulle, ce qui signifie que Cv n’est fonction que de la température ; on
5R
admet Cv = constante = .
2
5R 5R T V
3◦ ) • Gaz parfait : U = T + U0 , S = ln + R ln + S0 ;
2 2 T0 V0
5R a 5R T V −b
• Gaz de Van der Waals : U = T − + U00 , S = ln + R ln + S00 .
2 V 2 T0 V0 − b
7R
4◦ ) • Gaz parfait : H = T + H0 ;
2
b −1 a b a
   
• Gaz de Van der Waals : P V = RT 1 − − ' RT 1 + − ,
V V V RT V
7R RT a 7R a
   
d’où H ' T + b− + H00 ' T + P b− + H00 (à des termes du
2 V RT 2 RT
second ordre près).
5◦ ) On obtient cette relation en écrivant S = constante et en utilisant l’équation d’état.
2 7
• Gaz parfait : T V 5 = cste, soit P V 5 = cste0 ;
2 7 a b 7
• Gaz de Van der Waals : T (V − b) 5 = cste, soit P V 5 (1 + 2
)(1 − ) 5 = cste0 .
PV V
xxxxxxxxxxxxxxxxxxxxxxxxxxxx

Christian Carimalo 50 TD de Thermodynamique - L3


II - Détente de Joule-Gay-Lussac
1◦ ) Q = 0 (parois adiabatiques), W = 0, donc ∆U = 0.
• Gaz parfait : ∆T = 0 ;
1 1 a
• Gaz de Van der Waals : ∆T = ∆ =− < 0.
Cv V 2Cv V
2◦ ) ∆T = −3, 5 K.
xxxxxxxxxxxxxxxxxxxxxxxxxxxx

III - Détente de Joule-Thomson


1◦ ) Détente isenthalpique, ∆H = 0 (voir cours).
TV 1
 
2◦ ) τ = α− ; τ = 0 pour un gaz parfait. Pour un gaz quelconque, α dépend
Cp T
généralement des deux variables P et V . La courbe α = 1/T sépare alors le plan (P, V )
en deux régions : celle où α > 1/T , pour laquelle une détente isenthalpique provoque un
refroidissement (τ > 0), et celle où α < 1/T , pour laquelle une telle détente provoque un
échauffement (τ < 0).
1 x b 2a
3◦ ) Pour le gaz de Van der Waals, α = , où x = 1 − , TM = .
T 2
TM V Rb
1 − x (1 − x)
T
b 1−x TM
 
D’où : τ = x2 − 1 . Admettant que α est toujours positif, le
Cp T M T
1 − x2 (1 − x)
T
TM
dénominateur de τ est positif et le signe de τ est donc déterminé par celui de x2 − 1.
T
TM
4◦ ) τ est positif si x2 > 1, soit T < TM x2 . La “courbe d’inversion” séparant les deux
T
T
régions du plan (P, T ) correspondant à l’un ou l’autre signe de τ , est définie par t = = x2 .
TM
b
Déterminons la pression correspondante. Remplçant dans l’équation d’état V par puis
1−x
√ P √ a
x par t, on trouve p = = −3x2 + 4x − 1 = −3t + 4 t − 1, où P0 = 2 .
P0 b

La courbe p(t) est représentée à la figure ci-dessus. C’est à l’intérieur de cette courbe que
l’on a τ > 0, et à l’extérieur que τ < 0 ; p(t) est nul pour t = 1 et t = 1/9, et présente un
maximum pM = 1/3 pour t = 4/9.

Christian Carimalo 51 TD de Thermodynamique - L3


5◦ ) D’après l’expression de τ donnée au 3◦ ), il est impossible d’avoir τ > 0 dès lors que
T > TM , puisque x et toujours inférieur à 1 : si la température est supérieure à TM , il est
impossible de refroidir le gaz par détente isenthalpique.
6◦ ) Application numérique :
• azote : TM = 864 K ;
• hydrogène : TM = 224 K ;
• hélium TM = 35 K.
7◦ ) A la température ordinaire, l’azote peut se refroidir si la pression est dans le bon intervalle
de valeurs donné par la courbe d’inversion, tandis que l’hydrogène et l’hélium se réchauffent
(d’où la nécessité de refroidir préalablement ces gaz et de les liquéfier...).

xxxxxxxxxxxxxxxxxxxxxxxxxxxx

Christian Carimalo 52 TD de Thermodynamique - L3


TD6

I - Enoncés du second principe de la Thermodynamique

1◦ ) Donner l’énoncé du second principe de la Thermodynamique sous la forme d’un principe


d’évolution

2◦ ) a) Donner l’énoncé du 2nd principe sous la forme donnée par Kelvin. Montrer que
l’application du principe d’évolution peut conduire à l’énoncé de Kelvin.

b) Enoncer le 2nd principe sous la forme donnée par Clausius. Montrer que l’application du
principe d’évolution peut conduire à l’énoncé de Clausius.

II - Une masse d’eau m de chaleur massique isobare constante cp , soumise à la pression


atmosphérique constante, se trouve à la température T1 supérieure à la température at-
mosphérique T2 . L’atmosphère est utilisée comme source de chaleur.

Quel est le travail maximum que peut produire une machine thermique ditherme fonctionnant
entre cette masse d’eau et l’atmosphère ?

III - Deux masses d’eau identiques m se trouvent initialement à la même température T1 . Un


réfrigérateur fonctionnant entre ces deux masses d’eau, jouant le rôle de sources de chaleur,
effectue un certain nombre de cycles jusqu’à ce que la température finale de l’une de ces
sources soit égale à T2 < T1 . Quel est le travail minimum pour effectuer cette opération ?

IV - On veut extraire d’une source froide dont la température constante est T2 une quantité
de chaleur Q2 . A cet effet, un réfrigérateur effectue un grand nombre de cycles entre cette
source et une source chaude de température constante T1 . Quel est le travail minimum que
doit fournir le moteur du réfrigérateur pour effectuer cette opération ?

V - On veut extraire d’une masse m d’une certaine substance une quantité de chaleur
Q2 . A cet effet, un réfrigérateur effectue un certain nombre de cycles utilisant cette sub-
stance comme source froide de température variable et comme source chaude un thermostat
de température constante T1 . Calculer le travail minimum nécessaire pour effectuer cette
opération, en fonction de Q2 , de T1 et de la variation d’entropie ∆S2 de la substance. Mon-
trer que l’expression du travail minimum ainsi obtenue admet comme cas particulier le résultat
du IV.

Dans le cas où la substance reste soumise à pression constante, exprimer le travail minimum
en fonction de T1 , ∆S2 et de la variation d’enthalpie ∆H2 de la substance.

VI - On dispose de deux masses égales d’eau, de chaleur massique isobare constante cp , aux
températures respectives T1 et T2 < T1 . On les réunit par une machine réversible, à laquelle
elles servent de sources de chaleur, et qui fournit du travail jusqu’à ce que les deux sources
soient à la même température T0 . Calculer T0 .

Christian Carimalo 53 TD de Thermodynamique - L3


VII - Etude d’une bulle de savon
L’objectif de ce problème est de calculer la surpression ∆P , par rapport à l’atmosphère qui
l’entoure, qui existe à l’intérieur d’une bulle de savon à l’équilibre.
La bulle de savon dans l’air est modélisée comme une masse d’air m enfermée dans une
membrane élastique, de forme sphérique de rayon r. L’air de la bulle est assimilé à un gaz
parfait.
On suppose qu’on décrit bien la membrane avec deux variables indépendantes : la température
T d’une part et son rayon r ou sa surface A = 4πr2 , d’autre part. Quand on fait varier
réversiblement ces paramètres de dT et dA, on doit fournir le travail d¯W = K dA et la
chaleur d̄ Q = Cm dT + a dA où Cm et a sont des coefficients calorimétriques. On suppose
que K = K0 − αT où K0 et α sont des constantes.
1◦ ) Donner les différentielles de l’énergie interne Um et de l’entropie Sm de la membrane en
fonction de T et A et de leurs différentielles.
2◦ ) Appliquer les principes de la Thermodynamique pour en déduire a et une dérivée partielle
de Cm .
3◦ ) Montrer que l’énergie libre Fm = Um − T Sm vaut K A à une fonction de la température
T près.
4◦ ) Montrer que l’énergie libre F du système global “bulle de savon” comprenant la mem-
brane et l’air qu’elle contient peut s’écrire sous la forme

V
 
F = KA − nRT ln + Φ(T )
V0

où V est le volume de la bulle et Φ une fonction de la température.


5◦ ) Donner une expression du travail W fourni par l’atmosphère (à T0 et P0 ) à la bulle quand
le rayon de celle-ci varie.
6◦ ) Quelle est la relation entre ce travail et la variation consécutive de l’énergie libre ? Quelle
est la fonction qui est minimum lorsque la bulle est à l’équilibre ?
7◦ ) A l’équilibre, en déduire, pour la température T0 et en fonction du rayon r de la bulle,
la pression P de l’air à l’intérieur de celle-ci.

VIII - Modèle de goutte d’eau


On considère une goutte d’eau de masse M dans l’atmosphère terrestre. On la supposera
incompressible mais déformable. Comme elle est incompressible, son volume V est constant
pour une température T donnée, et l’on négligera même sa dilatation thermique de telle
sorte que V = M/ρ, avec ρ = 103 kg m−3 . En conséquence, le travail des forces de pression
atmosphérique sera considéré comme nul.
En revanche, l’aire A de l’interface de la goutte avec l’atmosphère peut varier lorsque la goutte
se déforme. Pour faire varier réversiblement sa surface de dA et modifier sa température de
dT il faut fournir à la goutte le travail élémentaire d¯W = σ dA et la chaleur élémentaire
d¯Q = CA dT + ` dA, CA étant la capacité calorifique de la goutte à aire constante. On
suppose que σ varie selon la loi : σ = σ0 − αT .

Christian Carimalo 54 TD de Thermodynamique - L3


1◦ ) Calculer ` et montrer que CA est fonction de la température uniquement. Exprimer CA
en fonction de la masse M de la goutte et de la chaleur massique de l’eau que l’on supposera
constante.
2◦ ) a) Exprimer dU et dS.
b) En déduire U, S et F .
On considère maintenant deux états possibles de la masse M d’eau : un état (A) ou “grosse
goutte” constitué par une seule goutte sphérique, un état (B) constitué par deux gouttes
identiques séparées, chacune de masse M/2. Ces deux configurations sont considérées comme
deux états d’équilibre thermodynamique dans les conditions du problème.
Pour calculer la variation des fonctions thermodynamiques entre ces deux états, on considère
une transformation dans laquelle on déforme la goutte unique jusqu’à ce qu’elle se sépare en
deux gouttes identiques.
4◦ ) Exprimer la variation de surface ∆A dans cette transformation.
5◦ ) a) Calculer la variation d’énergie interne ∆U entre un état “grosse goutte” à la température
TA et un état “deux gouttes identiques” à la température TB .
b) Calculer la variation d’entropie ∆S de façon approchée pour |TA − TB |/TA  1
6◦ ) On se demande si deux gouttes peuvent se réunir spontanément en une seule et de façon
adiabatique.
a) Montrer que la variation d’énergie interne d’une telle transformation serait nulle. En déduire
la variation de température ∆T .
b) Calculer la variation d’entropie ∆S de façon approchée.
c) Est-ce bien cette transformation qui est spontanée ou la transformation inverse ?
7◦ ) On considère maintenant que la réunion spontanée des deux gouttes s’est faite de façon
isotherme grâce à la chaleur Q reçue de l’atmosphère.
a) Exprimer Q en fonction de ∆U puis de ∆A. Quel est son signe ?
b) Exprimer ∆S. Quel est son signe ? Qu’en pensez-vous ?
c) Exprimer ∆F . Quel est son signe ? Qu’en pensez-vous ?
8◦ ) La pluie. Un nuage est initialement constitué de gouttes de rayon 10 µm, à 0◦ C qui ont
tendance à se rassembler. La pluie tombe quand la taille des gouttes atteint 2 mm. Quelle est
la température des gouttes à ce moment, le nuage étant isolé thermiquement et contenant
1 kg d’eau pour 50 kg d’air ?
Application numérique. On donne : la chaleur massique de l’eau 4,18 J K−1 g−1 ; la chaleur
massique de l’air 0,17 J K−1 g−1 ; σ0 = 0, 124 J m−2 .

Christian Carimalo 55 TD de Thermodynamique - L3


Corrigé TD6

- II -
A la fin de l’opération, la température de la masse d’eau est T2 et son entropie a varié de
T2
∆Seau = mcp ln , alors qu’elle aura cédé la chaleur Qeau = mcp (T2 − T1 ). La machine di-
T1
therme utilisée aura effectué un certain nombre de cycles et de ce fait, son entropie n’aura pas
varié. Quant à l’atmosphère, immense réservoir de chaleur dont la température reste constante
Qatm.
et égale à T2 , recevant la chaleur Qatm. , son entropie aura varié de ∆Satm. = .
T2
Considérant le système global Σ comprenant la masse d’eau, la machine et l’atmosphère, son
énergie interne aura varié de ∆UΣ = ∆Ueau + ∆Umachine + ∆Uatm. ≡ ∆Ueau + ∆Uatm. =
W = Qeau + Qatm. ; d’où Qatm. = W − Qeau . Ce système étant isolé thermiquement, la
variation de son entropie est nécessairement positive, ce qui conduit à l’inégalité

T2 T2 − T1 W
mcp ln − mcp + ≥0
T1 T2 T2

T1
 
Le travail −W que l’on peut ainsi récupérer est donc tel que −W ≤ mcp T2 φ , où
T2
φ(x) = x − 1 − ln x. Comme on sait, φ(x) est toujours positif et est nul si et seulement si
T1

x = 1. Il en ressort que le travail maximum récupérable est Wmax = mcp T2 φ .
T2
- III -
Lorsque la température de la source S1 aura atteint la valeur T2 , celle de l’autre source S2
sera devenue T3 . La première aura reçu la chaleur Q1 = mcp (T2 −T1 ), tandis que l’autre aura
reçu la chaleur Q2 = mcp (T3 − T1 ). L’ensemble Σ comprenant la machine et les deux sources
est isolé thermiquement. Son énergie interne aura varié de ∆UΣ = W = mcp (T3 + T2 − 2T1 )
T2 T3
et son entropie aura varié de ∆SΣ = mcp ln 2 , la machine effectuant des cycles. On doit
T1 " #
T12 T12
avoir ∆SΣ ≥ 0, soit T3 ≥ . On en déduit l’inégalité W ≥ mcp + T2 − 2T1 . Le
T2 T2
(T1 − T2 )2
travail minimum pour effectuer ladite opération est donc Wmin = mcp .
T2
- IV -
Q2 W − Q2
∆UΣ = W = Q1 + Q2 , d’où Q1 = W − Q2 ; ∆SΣ = + ≥ 0, et W ≥
T2 T1
T1
 
|Q2 | − 1 = Wmin .
T2
-V-
W − Q2
∆UΣ = W = Q1 + Q2 , d’où Q1 = W − Q2 ; ∆SΣ = ∆S2 + ≥ 0, et W ≥
T1
Q2 − T1 ∆S2 = Wmin . Si T2 = cste, ∆S2 = Q2 /T2 .
Si la substance évolue à pression extérieure constante, Q2 = ∆H2 et Wmin = ∆H2 − T1 ∆S2 .

Christian Carimalo 56 TD de Thermodynamique - L3


- VI -
T02 √
∆SΣ = mcp ln = 0 (machine réversible). Donc T0 = T1 T2 .
T1 T2
xxxxxxxxxxxxxxxxxxxxxxxxxxxx

VII - Etude d’une bulle de savon -

1
1◦ ) et 2◦ ) dUm = Cm dT + (K + a)dA, dSm = [Cm dT + a dA] ; dFm = d(Um − T Sm ) =
T
∂Sm ∂K a
  
−Sm dT + KdA est une d.t.e., d’où = − = , soit a = −αT , et
∂A T  ∂T
 A  T
∂Cm ∂(a + K)

a + K = K0 = constante. On a aussi (par dUm ) = = 0, et Cm
∂A T ∂T A
n’est fonction que de la température.
∂Fm
 
3◦ ) = K(T ), d’où, par intégration, Fm (T, A) = K(T ) A + ψm (T ).
∂A T

∂Fg nRT
 
4◦ ) Pour le gaz (parfait) à l’intérieur de la bulle : = −P = − , d’où Fg (T, V ) =
∂V T V
V
−nRT ln + ψg (T ). L’énergie libre totale de la bulle, F = Fm + Fg est donc de la forme
V0
V
 
F = KA − nRT ln + Φ(T )
V0

5◦ ) W = −P0 ∆V .
6◦ ) Pour le système “bulle + atmosphère”, ∆Stot = ∆Sbulle + ∆Satm. = ∆Sbulle −
∆Ubulle − W
≥ 0, soit W ≤ −∆ [Ubulle − T0 Sbulle ] = −∆Fbulle . Compte tenu de l’ex-
T0
pression de W , cette inégalité peut être récrite comme ∆Gbulle ≤ 0, où Gbulle = Ubulle −
T0 Sbulle +P0 Vbulle est l’enthalpie libre de la bulle soumise aux conditions (T0 , P0 ). C’est cette
fonction qui est minimum à l’équilibre.
∂Gbulle
 
7◦ ) A l’équilibre, = 0, ce qui conduit à l’équation
∂r T0 ,P0
2K(T0 ) 2K(T0 )
 
4πr2 P0 − P + = 0, soit, pour r 6= 0, P = P0 + .
r r
Le nombre n de moles d’air dans la bulle étant fixé, cette dernière équation permet en fait
4πr3
d’obtenir le rayon r à l’équilibre, puisque P = nRT0 /Vbulle , et Vbulle = .
3
xxxxxxxxxxxxxxxxxxxxxxxxxxxx

VIII - Modèle de goutte d’eau -

1
1◦ ) dU = CA dT + (` + σ)dA, dS = [CA dT + `dA] ; dF = d(U − T S) = −SdT + σdA
T
∂S ∂σ `
   
est une d.t.e., d’où =− = = α, soit ` = αT ; ` + σ = σ0 = constante,
∂A T  ∂T A T
∂CA ∂(` + σ)
  
d’où = = 0, et CA n’est fonction que de la température ; CA = M c
∂A T ∂T A
(avec c ' constante).
2◦ ) a) et b) U = M cT + σ0 A + M u0 ; S = M c ln T + αA + M s0 . On notera que

Christian Carimalo 57 TD de Thermodynamique - L3


pour préserver les propriétés d’extensivité des fonctions d’état, on a introduit des constantes
d’intégration “massiques” u0 et s0 .
1
4◦ ) ∆A = A2p −AG ; le volume d’une
h 1petiteigoutte est Vp = V /2 et a pour rayon rp = r/2 ,
3

d’où ∆A = 2[4πrp2 ] − 4πr2 = 4πr2 2 3 − 1 > 0.


M M
5◦ ) a) et b) UG = M cTA + σ0 AG + M u0 , U2p = 2 cTB + σ0 A2p + 2 u0 , d’où
2 2
∆U = U2p − UG = M c(TB − TA ) + σ0 ∆A ;
TB TB − TA
∆S = M c ln + α∆A ' M c + α∆A.
TA TA
6◦ ) Q = 0 (adiabatique), W = 0 (pas de travail), donc ∆U = 0. On en déduit ∆0 T =
σ0
TA − TB = ∆A (∆A précédent).
Mc
∆0 T σ(TA )∆A
b) ∆0 S ' M c − α∆A ' > 0.
TA TA
c) Comme ∆0 S > 0, c’est bien cette transformation qui est spontanée.
7◦ ) a) Q = ∆U = M c∆T − σ0 ∆A. Comme ∆T = 0, on obtient Q = −σ0 ∆A < 0.
b) ∆S = −α∆A < 0. Il s’agit d’une transformation monotherme, la variation d’entropie
peut être négative.
c) ∆F = −σ(T0 )∆A < 0. Ce signe est conforme au principe d’évolution : lors d’une trans-
formation monotherme sans échange de travail, l’énergie libre décroı̂t.
4πr3 4πr03
8◦ ) Dans le passage de N gouttes de volume V = à N 0 gouttes de volume V 0 = ,
3 0
0
3
NA r meau
le volume total est conservé : N 0 V 0 = N V , d’où = 0 ; on a N = .
NA r ρeau V
Ecrivant ∆Unuage = [Mair cair + meau ceau ] (Tf −Ti )+[N 0 A0 − N A] σ0 = 0, on trouve, toutes
simplifications faites,
r
3σ0

meau
 1− 0
Tf − Ti = r ' 3K
rρeau Mair meau
cair + ceau
Mair

xxxxxxxxxxxxxxxxxxxxxxxxxxxx

Christian Carimalo 58 TD de Thermodynamique - L3


TD7

I - Equilibre liquide-vapeur d’un corps pur


1◦ ) Rappeler la règle des phases de Gibbs. L’appliquer au cas de l’équilibre liquide-vapeur
d’un corps pur. Tracer et commenter le réseau d’isothermes d’un mélange liquide-vapeur dans
le diagramme de Clapeyron.

Soit x le taux massique de vapeur dans le mélange. On considèrera dans la suite un mélange
de masse unité. On notera :
• cpv la chaleur massique à pression constante de la vapeur à proximité de la courbe de rosée
(s)
et cpv la chaleur massique de la vapeur saturante ;
• c` la chaleur massique à pression constante du liquide à proximité de la courbe d’ébullition
(s)
et c` la chaleur massique du liquide saturant ;
• L(T ) la chaleur latente massique de vaporisation à la température T .

2◦ ) Exprimer x en fonction de Vv (T ) et V` (T ), volumes massiques respectifs de la vapeur et


du liquide à la température T , et du volume total V .
3◦ ) Le mélange est caractérisé par sa température T et son titre x. Au cours d’une transfor-
mation infinitésimale où ses paramètres varient de dT et dx respectivement, déterminer la
chaleur d̄ Q reçue par le système. En déduire la variation dS d’entropie du système.
4◦ ) Considérant deux isothermes voisins de température respectives T et T + dT et leurs
intersections respectives B et B 0 avec la courbe de rosée, montrer que

dP
 
c(s)
pv = cpv − α T vv
dT BB 0

où α est le coefficient de dilatation à pression constante. Dans toute la suite, on assimilera
la vapeur à un gaz parfait.
5◦ ) Les deux isothermes précédents coupent la courbe d’ébullition en A et A0 respectivement.
(s)
Montrer que c` ≈ c` .
(s)
6◦ ) Calculer cv − c(s) en fonction de L, T et dL/dT . Pour cela, on exprimera de deux
façons différentes la variation d’entropie lorsque le système passe d’un liquide saturé à la
température T à la vapeur saturante à la température T + dT .
7◦ ) Déduire de 3◦ ) et 6◦ ) l’équation des isentropiques du mélange liquide-vapeur.
8◦ ) Calculer, en fonction de la température, le titre x0 pour lequel une détente isentropique
du mélange ne modifie pas son titre.
Application numérique. Calculer, pour l’eau, les valeurs de x0 pour une température comprise
entre 0◦ C et 30◦ C. On donne : L(T ) = L0 −aT , avec L0 = 3335 J g−1 ; a = 2, 915 J K−1 g−1 ;
c` ' 1 cal g−1 .

Christian Carimalo 59 TD de Thermodynamique - L3


II - Relation de Clapeyron, point triple
On considère l’équilibre de deux phases, notées 1 et 2 respectivement, d’une mole d’un corps
pur à la température T et à la pression P . On note x la fraction molaire de la phase 2, g1 et
g2 les enthalpies libres molaires, v1 et v2 les volumes molaires, s1 et s2 les entropies molaires
des phases 1 et 2, respectivement.
1◦ ) Exprimer l’enthalpie libre g(T, P, x) du système des deux phases en fonction de x, g1 et
g2 . En déduire la condition d’équilibre des deux phases en fonction de g1 et g2 .
2◦ ) Cette condition étant satisfaite à une température T et une pression P , déterminer la
relation entre v1 , v2 , s1 , s2 , dP/dT pour qu’elle soit encore réalisée à la température T + dT
et à la pression P + dP .
3◦ ) Montrer que la chaleur latente molaire `(T ) de la transformation phase 1 → phase 2
s’écrit

dP
`(T ) = T (v2 − v1 )
dT

4◦ ) Au point triple d’un corps pur, celui-ci existe sous les trois phases solide, liquide et vapeur.
Dans le plan (ln P, T ), et au voisinage du point triple, les courbes d’équilibre liquide-vapeur
et solide-vapeur ont pour équations respectives

bv bs
ln Pvap = av − , ln Psub = as −
T T

où av , bv , as et bs sont des constantes et ln désigne le logarithme néperien.


a) Déterminer la température Tt et la pression Pt du point triple.
b) En assimilant la vapeur à un gaz parfait et en négligeant les volumes molaires du liquide et
du solide devant celui de la vapeur, montrer que les chaleurs latentes molaires de vaporisation
`vap et de sublimation `sub sont indépendantes de la température. En donner les expressions.
c) Exprimer `vap et `sub en fonction des enthalpies molaires à l’état solide , liquide et vapeur.
En déduire la chaleur latente molaire de fusion `fus au voisinage du point triple.
5◦ ) Pour le corps pur considéré. on donne av = 15, as = 25, bv = 6000 K, bs = 16000 K,
lorsque la pression est exprimée en mm Hg. Calculer Tt , Pt , `vap , `sub et `fus .

III - Point triple de l’eau


Les caractéristiques du point triple de l’eau sont Pt = 6, 03 10−3 atm, Tt = 273, 14 K.
On sait que la température de vaporisation de l’eau varie de ∆T = 0, 2 K pour un accrois-
sement de pression de ∆P = 8, 9 10−5 atm au voisinage du point triple.
1◦ ) En assimilant la vapeur à un gaz parfait, calculer la chaleur latente de vaporisation Lv
au point triple.
2◦ ) Etablir une relation entre les chaleurs latentes de vaporisation Lv , de sublimation Ls et
de fusion Lf . On envisagera un cycle infiniment petit autour du point triple.
3◦ ) Connaissant Lf , déterminer la pente dPs /dT de la courbe de sublimation au point triple.
4◦ ) Justifier la pente de la courbe de fusion.

Christian Carimalo 60 TD de Thermodynamique - L3


On donne Lf = 333, 66 J g−1 et le volume massique du solide vs = 1, 097 cm3 g−1 .

IV - Loin des conditions critiques, la chaleur latente molaire de vaporisation d’un liquide
peut être considérée comme une fonction sensiblement affine de la température

`(T ) = `0 − β T

`0 , β étant des constantes.


1◦ ) Donner les expressions de la pression de vapeur saturante et du volume molaire de
la vapeur saturante en fonction de T . On supposera que le volume molaire du liquide est
négligeable devant celui de la vapeur et que la vapeur est assimilable à un gaz parfait.
2◦ ) Quelles sont les variations de la pression et du volume molaire de la vapeur d’eau saturante
à 100◦ C sous 1 atmosphère lorsqu’on élève la température de 1◦ C ? On donne : la masse
molaire de l’eau M = 18 ; la chaleur latente massique de vaporisation de l’eau, en cal/g, à
la température T : L(T ) = 607 − 0, 7 (T − 273).

V - Etude de l’équilibre vapeur-solide du cadmium


On se propose d’étudier la sublimation du cadmium pour des températures comprises entre
300 K et 600 K.
A/ Etude thermodynamique du cadmium solide
Dans le domaine de températures T exploré, l’énergie libre Fs de ns moles du solide s’écrit
 
Fs (T, ns ) = −ns `0 + 3ns RT ln 1 − e−θ/T (1)

où `0 , énergie de cohésion, et θ, température d’Einstein, sont des constantes positives ; R est
la constante des gaz parfaits. Le volume n’intervient pas dans le modèle défini par (1). On
donne `0 = 113 103 MKSA et θ = 104 K.
1◦ ) L’équation (1) est-elle acceptable du point de vue du caractère extensif/intensif des
grandeurs qui y figurent ?
2◦ ) Déterminer l’entropie Ss , l’énergie interne Us , la chaleur molaire à volume constant Cv
et le potentiel chimique µs du solide.

B/ Etude thermodynamique de la vapeur de cadmium


L’énergie libre Fv de nv moles de vapeur de cadmium occupant le volume V à la température
T est donnée par
" ! #
a nv T −3/2
Fv (T, V, nv ) = nv RT ln −1 (2)
V

a étant une constante dont on précisera la dimension.


1◦ ) Déterminer l’entropie Sv , l’énergie interne Uv et le potentiel chimique µv de la vapeur.
2◦ ) A partir de (2), trouver l’équation d’état de la vapeur.

Christian Carimalo 61 TD de Thermodynamique - L3


3◦ ) Vérifier que l’expression obtenue pour Sv est acceptable du point de vue du caractère
extensif/intensif des grandeurs qui y figurent.

C/ Etude de l’équilibre vapeur-solide d’une masse donnée de cadmium


On considère un système Σ de n moles de cadmium contenant nv moles de vapeur occupant
un volume fixé V à la température fixée T , en équilibre avec ns moles de solide. Le volume
occupé par le solide est négligeable devant celui de la vapeur. Les phases solide et vapeur
sont décrites par les modèles définis par (1) et (2), respectivement.
1◦ ) Démontrer qu’à l’équilibre le potentiel chimique du cadmium est le même dans les deux
phases. En déduire une relation entre nv , T et V traduisant cet équilibre.
2◦ ) Montrer que la pression d’équilibre solide-vapeur est donnée par
1  3
P = R T 5/2 1 − e−θ/T e−`0 /(RT )
a

3◦ ) En déduire la différence sv − ss entre les entropies molaires de la vapeur et du solide.


Retrouver ce résultat à partir de A/2 et B/1.
4◦ ) Déterminer la chaleur latente molaire de sublimation `(T ). Calculer `(T ) pour T = 500 K.

VI - Retard à la vaporisation
Un fluide est initialement à l’état de liquide à la température T0 inférieure à sa température
critique. Sa pression P0 est progressivemenr amenée puis fixée à une valeur inférieure à la
pression de vapeur saturante Ps (T0 ) du fluide pour la température T0 . On observe alors un
retard à la vaporisation : le fluide se trouve dans un état où n` moles en phase liquide à la
pression P0 coexistent avec nv moles en phase vapeur, ces dernières se présentant sous forme
de petites bulles à l’intérieur desquelles la pression est Pv . Ce retard est dû à un phénomène
de tension superficielle à l’interface entre la phase liquide et la phase vapeur. Pour simplifier,
on supposera qu’il n’y a qu’une seule bulle de vapeur, de forme sphérique. On note V, A et
r le volume, la surface et le rayon de la bulle, respectivement.
On admettra que l’enthalpie libre totale G?t de l’ensemble “liquide-bulle de vapeur avec
l’interface” a pour expression

G?t = G` (T0 , P0 , n` ) + Fv (T0 , V, nv ) + P0 V + σA

où G` est l’enthalpie libre de la phase liquide, Fv est l’énergie libre de la phase vapeur à
l’intérieur de la bulle et σ le coefficient de tension superficielle supposé ne dépendre que de
la température.
1◦ ) T0 et P0 étant fixés, trouver l’expression de la différentielle G?t en fonction des différentielles
de nv et de r.
2◦ ) a) Ecrire les deux conditions d’équilibre de la bulle de vapeur avec la phase liquide, dont
l’une sera exprimée à l’aide des enthalpies libres molaires des deux phases.

b) Montrer que l’autre condition conduit à la relation Pv = P0 + .
r
3◦ ) a) Quelle relation existe-t-il entre les enthalpies libres molaires des deux phases lorsque
celles-ci coexistent à la température T0 et à la pression Ps (T0 ) ?

Christian Carimalo 62 TD de Thermodynamique - L3


b) On assimile dorénavant la phase vapeur à un gaz parfait et l’on admet que que le volume
molaire v` du liquide est pratiquement invariable. Montrer alors, à partir d’une relation établie
au 2◦ ) et de la relation rappelée au 3◦ ) a) que la pression Pv de la vapeur dans la bulle est
aussi donnée par

RT0 Pv
 
P0 − Ps (T0 ) = ln
v` Ps (T0 )

4◦ ) Application numérique. Le fluide est de l’eau à la température T0 = 373 K et à la pression


P0 = 0, 5 105 Pa. On donne : Ps (T0 ) = 105 Pa ; σ = 58, 85 10−3 kg s−2 ; le volume massique
de l’eau liquide est de 1 cm3 g−1 ; la masse molaire de l’eau est M = 18 g. Calculer :
a) La pression Pv de la vapeur dans la bulle ;
b) le rayon r de la bulle ;
c) le nombre nv de moles d’eau en phase vapeur dans la bulle.

VII - Fusion du soufre monoclinique


1◦ ) Rappeler la condition d’équilibre de deux phases d’un corps pur.
2◦ ) Démontrer la relation de Clapeyron pour le changement d’état solide-liquide d’un corps
pur.
3◦ ) A la pression ordinaire P1 = 105 Pa, la température de fusion du soufre monoclinique
est T1 = 400 K. La variation de volume consécutive à la fusion est alors de 40 cm3 kg−1 . La
masse molaire du soufre est de 32 g/mole.
En assimilant à une droite la courbe de fusion dans le plan (P, T ) et en supposant la chaleur
latente de fusion constante et égale à 1800 J/mole dans le domaine de température considéré,
calculer :
a) la température de fusion du soufre monoclinique à la pression P2 = 1, 8 108 Pa ;
b) la variation de volume consécutive à la fusion à la température T2 .

VIII - Pompe à chaleur - Cycle de Rankine

Le chauffage d’un appartement est assuré à l’aide d’une pompe à chaleur. Celle-ci extrait
de la chaleur de l’atmosphère extérieure, constituant une source froide de températute T1 ,
en ne recevant qu’un faible travail et transmet une certaine quantité de chaleur à l’air de
l’appartement constituant la source chaude de température T2 . Le fluide utilisé est du fréon
(difluoromonochlorométhane) qui accomplit un cycle de transformations au cours duquel il est
successivement détendu dans un détendeur (D), évaporé dans un vaporisateur (V), comprimé
dans un compresseur (C), refroidi puis liquéfié dans un liquéfacteur (L) (voir figure 9).
A/ Expliquer pourquoi le liquéfacteur et le vaporisateur doivent être respectivement reliés à
l’air de l’appartement et à l’atmosphère extérieure.

B/ On suppose que le cycle est parfaitement réversible. On note W le travail de compression


reçu par le fréon pendant 1 heure, −Q1 et −Q2 les chaleur reçues respectivement par la
source froide et par la source chaude, pendant la même durée. On donne T1 = 273 K,
T2 = 305 K ; la puissance du moteur électrique actionnant le compresseur est 400 W.

Christian Carimalo 63 TD de Thermodynamique - L3


source chaude

Q2 < 0 refroidissement
T2
L
T’1

D C

V
T1 T1
Q1 > 0

source froide

Figure 9 – Cycle d’une pompe à chaleur

1◦ ) Calculer la quantité de chaleur fournie à l’appartement pendant 1 heure.


2◦ ) Quelle serait la quantité de chaleur fournie à l’appartement si la même puissance électrique
était dispensée par effet Joule dans un radiateur.
3◦ ) Quelle est l’origine du “gain énergétique” ? Définir et calculer le coefficient de perfor-
mance η de la pompe à chaleur.

C/ En fait, le cycle n’est pas réversible. Une bonne représentation du fonctionnement de la


pompe à chaleur est donnée par le cycle de Rankine comprenant les cinq transformations
suivantes.
• (D) [a → b] : détente irréversible à enthalpie constante, adiabatique ;
• (V) [b → c] : vaporisateur isotherme, isobare et réversible ;
• (C) [c → d] : compression adiabatique réversible ;
• (L) [d → e] : refroidissement isobare réversible, jusqu’à obtention de la vapeur saturante ;
• (L) [e → a] : liquéfaction isotherme, isobare et réversible.
Trois courbes isothermes du fréon interviennent dans le cycle : celui de température T1 ,
température du vaporisateur ; celui dont la température T10 est celle atteinte à la fin de la
vaporisation ; celui dont la température T2 est celle du liquéfacteur.
1◦ ) Représenter le cycle dans le diagramme de Clapeyron.
2◦ ) Représenter le cycle dans le diagramme (T, S).
3◦ ) Représenter le cycle dans le diagramme de Mollier (H, S). Montrer que dans ce dia-
gramme, les transformations b → c et e → a sont des segments de droite. Quelles sont leurs
pentes ?
4◦ ) Montrer que le coefficient de performance η peut être calculé si l’on connaı̂t les enthalpies
ha , hc et hd des états a, c et d.

Christian Carimalo 64 TD de Thermodynamique - L3


IX - Transition phase normale → phase supraconductrice d’une substance
magnétique
A/ On considère une mole de substance magnétique de volume V invariable, à la température
T et placée dans un champ magnétique B. On admet que son aimantation par unité de volume
B
M est liée à B par la relation M = χ où χ est une constante et µ0 la perméabilité du
µ0
vide.
Cette substance peut exister sous deux phases distinctes : la phase normale (n), définie par
χn = 0, et la phase supraconductrice (s), pour laquelle χs = −1. Cette dernière existe
seulement à basse température et à faible champ magnétique.
1◦ ) En utilisant la règle de la variance, montrer que lorsque les deux phases coexistent à
pression fixe, il existe une relation entre B et T définissant la courbe de transition entre les
deux phases, soit B(T ).
On définit l’état d’équilibre entre les phases (n) et (s) par la température T et la fraction
molaire x de la partie de substance se trouvant dans la phase (s). On note L la chaleur
latente de changement d’état pour une mole passant de l’état (s) à l’état (n), CBs et C n les
B
chaleurs molaires à champ magnétique constant des phases (s) et (n), respectivement.
2◦ ) Pour une transformation réversible élémentaire (x, T ) → (x + dx, T + dT ), donner les
expressions du travail d̄ W et de la chaleur d̄ Q reçus par le système. Montrer que le travail
chimique est nul.
3◦ ) En appliquant les deux principes de la Thermodynamique, déterminer L et CB
s − C n en
B
supposant B(T ) connu.

B/ La détermination de la courbe de transition B(T ) nécessite l’introduction de modèles


permettant de caractériser les deux états (n) et (s).

♣ Etude d’un modèle simple


1◦ ) Ce modèle sera défini par la donnée de l’enthalpie libre généralisée

B2
G? = N a + N bP + Ψ(T, N ) (1)
2

N étant le nombre de moles du corps magnétique, T et P la température et la pression de


celui-ci, a et b deux constantes positives. Montrer que

Ψ(T, N ) = N ψ(T ) (2)

Quelle est la nature de l’équation (1) ?


2◦ ) Montrer que le volume molaire d’un échantillon décrit par (1) est invariable et que
l’aimantation totale doit être de la forme M = N ϕ(B, T ).
3◦ ) Calculer M et vérifier que le résultat est en accord avec 2◦ ).
4◦ ) Calculer l’entropie S du système. Le modèle peut-il être valable à basse température ?
5◦ ) Déterminer la fonction d’état énergie interne U du système.

Christian Carimalo 65 TD de Thermodynamique - L3


♣ Description des phases (n) et (s) à l’aide du modèle (1)
1◦ ) Montrer que l’expression (1) convient pour les deux phases si l’on fixe de façon appropriée
le paramètre a pour chacun des cas. On remarquera que le volume molaire est constant pour
les deux phases.
Soient gn? et gs? les enthalpies libres molaires généralisées des phases (n) et (s), respec-
tivement. On note ψn (T ) et ψs (T ) la fonction arbitraire figurant dans (2), selon le cas.
Déterminer gn? et gs? .
2◦ ) Ecrire la condition d’équilibre entre les deux phases à pression fixée et en déduire la
relation B(T ).
3◦ ) Déterminer les entropies molaires sn et ss des deux phases. Exprimer sn − ss sur la
courbe de transition. Retrouver alors la chaleur latente molaire L et le saut de chaleur
molaire CB s − C n à la transition (résultats A/ 3◦ ))
B

4◦ ) On rend!bien compte des résultats expérimentaux pour T ≤ T0 en prenant B(T ) =


T2
B0 1 − 2 où B0 est une constante que l’on supposera positive.
T0
a) Tracer la courbe B(T ) et préciser les régions correspondant à l’une ou à l’autre phase.
s − C n . Tracer leurs courbes de variation en fonction de T .
b) Expliciter L et CB B

5◦ ) Application numérique pour l’aluminium. On donne : B0 = 1, 05 10−2 Tesla ; T0 =


1, 18 K ; masse volumique 2, 7 103 kg m−3 ; masse atomique 27 ; µ0 = 4π 10−7 S.I. Calculer
CBs − C n à T .
B 0

X - L’atmosphère isotherme
Du fait de l’attraction gravitationnelle, la pression atmosphérique varie avec l’altitude z. Pour
rendre compte de ce phénomène, on utilise le modèle suivant :
• La température T de l’atmosphère est supposée uniforme, c’est-à-dire, indépendante de z ;
• l’atmosphère est envisagée comme une succession de couches gazeuses de même épaisseur
infinitésimale δz. Chaque couche est considérée comme une “phase” particulière du gaz
constituant l’atmosphère, chaque phase étant ainsi associée à une valeur particulière de z et
donc de l’énergie potentielle de pesanteur ;
• le gaz constituant l’atmosphère, de masse molaire M , est supposé parfait.
Dans ce modèle, l’atmosphère constitue un système thermodynamique comportant une infi-
nité de phases, en équilibre les unes avec les autres, à la même température T .
1◦ ) Définir le “potentiel chimique” de la phase constituée de la tranche de gaz située entre
les altitudes z et z + δz. On définira pour cela l’énergie interne molaire généralisée Ũ tenant
compte de l’énergie potentielle de pesanteur, et l’enthalpie libre molaire généralisée G̃ de
cette tranche.
2◦ ) Quelle est la condition d’équilibre de l’atmosphère à la température T ?
3◦ ) a) En déduire l’expression P (z, T ) de la pression de l’atmosphère à l’altitude z.
b) retrouver ce résultat à partir de la condition d’équilibre mécanique d’une masse δm de gaz
d’épaisseur δz, de section horizontale δS, située à l’altitude z.

Christian Carimalo 66 TD de Thermodynamique - L3


XI - Machine frigorifique

Figure 10 – Cycle d’une machine frigorifique

Un groupe frigorifique prélève de la chaleur à l’intérieur d’une chambre froide et en restitue


à l’atmosphère extérieure. Le fluide réfrigérant F utilisé par cette pompe décrit le cycle
réversible suivant (figure 10)
• le fluide, à l’état de liquide saturé, pénètre dans un détendeur (qui ne contient pas de
pièce mobile) où il subit une détente isenthalpique et passe de l’état A(T1 , P1 ) représenté
par un point A sur la courde d’ébullition, à l’état B(T2 , P2 ) où il est partiellement à l’état
de vapeur ; P2 est la pression de vapeur du fluide à la température T2 ;
• il pénètre ensuite dans l’évaporateur (radiateur échangeur) où il achève sa vaporisation
et quitte l’évaporateur à la température T2 et sous la pression P2 (état C sur la courbe de
rosée) ;
• Le fluide gazeux subit ensuite une transformation adiabatique dans un compresseur où il
reçoit de l’énergie grâce à des pales en rotation entraı̂nées par un moteur électrique ; le fluide
en sort à la température T10 et sous la pression P1 (état D) ;
• le fluide pénètre alors dans le condenseur (radiateur échangeur) où il est maintenu à pression
constante P1 ; il s’y refroidit, s’y liquéfie entièrement à la température T1 jusqu’à l’état initial
A(T1 , P1 ).
On fait les hypothèses suivantes :
- le groupe frigorifique fonctionne en régime permanent ; l’énergie cinétique du fluide et les
effets de pesanteur sont ignorés ;
- le fluide gazeux est supposé parfait ; son rapport γ = Cp /Cv est supposé constant ;
- la chaleur massique du liquide, notée c` , est supposée constante.
On note M la masse molaire du fluide et Lv (T ) sa chaleur latente massique de vaporisation.

Christian Carimalo 67 TD de Thermodynamique - L3


♠ Tous les calculs devront être effectués pour une masse m = 1 kg de fluide F. On donne :
T1 = 305 K ; P1 = 12, 65 105 Pa ; T2 = 273 K ; P2 = 5 105 Pa ; Lv (T1 ) = 175 103 J kg−1 ;
Lv (T2 ) = 205 103 J kg−1 ; c` = 1, 38 103 J kg−1 K−1 ; M = 86, 5 10−3 kg mole−1 ; γ = 1, 2 ;
R = 8, 31 J mole−1 K−1 .

A/ Etude du cycle du fluide réfrigérant.


1◦ ) Dans le plan (P, V ), représenter l’allure du cycle décrit par l’unité de masse du fluide F.
On y fera figurer sa coube de saturation et les isothermes concernés.
2◦ ) Détente isenthalpique A → B.
Dans le plan (P, V ), soit A0 le point représentatif du fluide à l’état de liquide saturant à la
température T2 .
a) Exprimer les différences d’enthalpie massiques ∆HAA0 et ∆HA0 B du fluide en fonction de
la fraction massique x de fluide vaporisé à l’état B, et des données.
b) En déduire x. Application numérique.
c) Calculer la variation d’entropie massique ∆SAB du fluide dans la transformation A → B.
Application numérique.
3◦ ) Evaporation B → C.
Pour cette transformation, calculer la quantité de chaleur QBC reçue par l’unité de masse
du fluide et la variation d’entropie massique ∆SBC correspondante. Application numérique.
4◦ ) Compression isentropique C → D.
a) calculer la température T10 de l’état D. Application numérique.
b) Calculer le travail WCD reçu par l’unité de masse du fluide. Application numérique.
c) Calculer la variation d’enthalpie massique ∆HCD . Application numérique.
5◦ ) Condensation D → A.
Pour cette transformation, calculer la chaleur QDA reçue par l’unité de masse du fluide et la
variation d’entropie massique correspondante ∆SDA . Application numérique.
6◦ ) Définir et calculer l’efficacité (ou coefficient de performance)  de ce groupe frigorifique.
Application numérique.

B/ Refroidissement de la chambre froide.


Dans cette partie, on fait l’hypothèse simplificatrice que tout se passe comme si la pompe
thermique était une machine ditherme réversible fonctionnant uniquement entre les deux
sources de chaleur constituées l’une par l’air de la chambre froide de température variable
T (t) et l’autre par l’atmosphère extérieure à la température constante T0 . On note P la
puissance mécanique fournie par le compressuer et C la capacité calorifique de la chambre
froide. On néglige toute fuite thermique.
La chambre froide étant à la température initiale T0 , on met la pompe thermique en marche.
7◦ ) Entre les dates t et t + dt, la température de la chambre froide passe de T (t) à T (t + dt)
tandis que la machine effectue un certain nombre de cycles. En faisant les bilans d’énergie et

Christian Carimalo 68 TD de Thermodynamique - L3


d’entropie de la machine pour ces cycles, trouver l’équation différentielle liant T à t.
8◦ ) Par intégration de cette équation, déterminer la fonction t(T ).
9◦ ) Calculer le temps τ au bout duquel la température de la chambre froide est TF = 277 K,
à l’aide des données : T0 = 293 K ; P = 54 W ; C = 106 J K−1 .

Christian Carimalo 69 TD de Thermodynamique - L3


Corrigé TD7

I - Equilibre liquide-vapeur d’un corps pur

1◦ ) Voir le cours : v = c + 2 − ϕ.
2◦ ) On considère un mélange de masse unité, le volume total V est donc le volume massique
V − V`
total : V = xV` + (1 − x)Vv , d’où x = .
Vv − V`
3◦ ) d¯Q = d¯Qv + d¯Q` + dxL avec d¯Qv = cpv dT + hv dP , dbQ` = c` dT + h` dP ;
d̄ Q
dS = = · · ·. Remarque : cette expression de dS peut être obtenue en exprimant
T
directement l’entropie totale à l’aide des entropies massiques Sv et S` : S = xSv + (1 − x)S` ,
L
d’où dS = xdSv + (1 − x)dS` + dx [Sv − S` ], Sv − S` = .
T
dP
 
◦ (
4 ) d¯Qv = c s)pv dT = cpv dT + hv dP , avec dP = dT , hv = −T Vv α (α =
dT rosée
1 ∂Vv dP
  
(s)
), d’où, après division par dT , cpv = cpv − α T Vv .
Vv ∂T P dT rosée
dP
 
(s)
5◦ ) On a aussi bien c` = c` − α` T V` , mais comme α` est petit, on peut écrire
dT ébull.
(s)
c` ≈ c` .
(s)
6◦ ) On a dS = dx(Sv − S` ) + xdSv + (1 − x)dS` , avec T dSv = cpv dT + hv dP = dT cpv ,
(s) 1h (s)
i
T dS` = cp` dT , d’où dS = dxL + xc(s)
pv dT + (1 − x)cp` dT . Cette différentielle étant
T
une d.t.e. vis-à-vis de x et de T , on a

d L ∂ h (s)
  i
(s) (s)
= xcpv + (1 − x)cp` = c(s)
pv − cp`
dT T ∂x

L (s) dT d
L (s) dT xL
   
7◦ ) dS = dx + c` + xdT = c` +d . Une isentropique est définie
T T dT
T T  T
(s) dT xL
par dS = 0, ce qui donne ici l’équation c` = −d .
T T
T d L L0
 
8◦ ) [x0 ]−1 =− = ; 0, 34 ≤ x0 ≤ 0, 38 (1 cal = 4,18 J).
c` dT T T c`

xxxxxxxxxxxxxxxxxxxxxxxxxxxx

II - Relation de Clapeyron, point triple


1  P, x) = (1 − x)g1 (T, P ) + xg2 (T, P ). A T et P fixés, la condition d’équilibre est
 ) g(T,
∂g
= 0 = g2 (T, P ) − g1 (T, P ).
∂x T,P
2◦ ) g1,2 (T + dT, P + dP ) = g1,2 (T, P ) + dg1,2 . Pour que la condition d’équilibre soit encore
réalisée dans les conditions (T + dT, P + dP ), il faut que dg1 = dg2 , et comme dg1,2 =

Christian Carimalo 70 TD de Thermodynamique - L3


dP s2 − s1
−s1,2 dT + v1,2 dP , la relation suivante doit donc être satisfaite : = .
dT v2 − v1
dP
3◦ ) `(T ) = T (s2 − s1 ), d’où `(T ) = T (v2 − v1 ) .
dT
bs − bv bs av − bv as
4◦ ) a) Au point triple, Pvap = Psub , d’où Tt = , puis ln Pt = .
as − av bs − bv
RT 2 dP
 
b) `vap ' = Rbv ; de même, `sub ' Rbs .
P dT vap

c) Dans le domaine de températures où les deux phases liquide et vapeur coexistent : `vap =
`vap (T ) = hv (T, Pvap (T )) − h` (T, Pvap (T )) = hv − h` (h : enthalpie molaire) ; dans le
domaine de températures où les deux phases solide et vapeur coexistent : `sub = `sub (T ) =
hv (T, Psub (T )) − hs (T, Psub (T )) = hv − hs . Ecrivant qu’au voisinage du point triple, T ' Tt ,
Pvap ' Psub , on a `sub −`vap ' hv (Tt )−hs (Tt )−hv (Tt )+h` (Tt ) = h` (Tt )−hs (Tt ) = `fus (Tt ).
On trouve ici `fus = R(bs − bv ).
5◦ ) Tt = 1000 K ; ln P = 9, d’où P = 403 mm Hg (e3 ' 20, e9 ' 400) ; en prenant
R = 25/3 : `vap = 5 104 J/mole, `sub = 1, 3 105 J/mole, `fus = 8 104 J/mole.

xxxxxxxxxxxxxxxxxxxxxxxxxxxx

III - Point triple de l’eau

dP RT 2
RT dP
   
1◦ ) Lv = T (vv − v` ) '  v` (vv ' 2 105 cm3 /g,
car vv '
dT vap vap P P dT
RTt2 ∆P
v` ' 1 cm3 /g). D’où, au voisinage du point triple, Lv ' = 46 103 J/mole = 610
Pt ∆T
cal/g.
2◦ ) Ls = Lf + Lv .
dP dP P Lf
   
3◦ ) = + = 51 Pa/K.
dT sub dT vap RT 2
dP
 
4◦ ) < 0, car vs > v` . Avec v` = 1 cm3 /g, vs = 1, 097 cm3 g−1 , Lf = 333, 66 J
dT fus
dP
 
g−1 , on trouve = −1, 26 107 Pa/K = −124 atm/K.
dT fus

xxxxxxxxxxxxxxxxxxxxxxxxxxxx

- IV -
RT d Lv `0 β
1◦ ) v v = ; [ln Pv ] = , d’où ln Pv = − − ln T +C, C étant une constante.
Pv dT RT 2 RT R
∆P L(T ) 537 × 4, 18 × 18
2◦ ) Variation relative : = 2
∆T = × 1 = 3, 5 10−2 .
P RT 8, 32 × (373)2

xxxxxxxxxxxxxxxxxxxxxxxxxxxx

Christian Carimalo 71 TD de Thermodynamique - L3


V - Etude de l’équilibre vapeur-solide du cadmium

- A - Etude thermodynamique du cadmium solide

1◦ ) θ est une température, le rapport θ/T , sans dimension, est intensif, T est intensive, `0
doit être intensif. Il en résulte que Fs /ns est intensif et F est extensif.
∂Fs 1 ∂Us ∂Fs
     
2◦ ) Ss = − , Us = Fs + T Ss , Cv = , µs = ; d’où
∂T nv ,V ns ∂T nv ,V ∂ns T,V

θ/T 3ns Rθ 3Reθ/T θ2


 h i 
Ss = 3ns R θ/T − ln 1 − e−θ/T ; Us = −ns `0 + θ/T , Cv = 2 2 ,
e −1 e −1 eθ/T − 1 T
h i
µs = −`0 + 3RT ln 1 − e−θ/T .

- B - Etude thermodynamique de la vapeur de cadmium

V T 3/2 5 anv −3/2


  
1◦ )[a] = [ ] : a s’exprime en m3 K3/2 mole−1 ; Sv = nv R − ln T ,
nv 2 V
3 anv −3/2
 
Uv = nv RT , µv = RT ln T .
2 V
∂Fv nv RT
 
2◦ ) P = − = .
∂V nv ,T V
3◦ ) ···

- C - Etude de l’équilibre vapeur-solide d’une masse donnée de cadmium

anv −3/2
h i  
1◦ ) µs = µv : cours. D’où −`0 + 3RT ln 1 − e−θ/T = RT ln T .
V
nv P 1  3
2◦ ) = d’où P = R T 5/2 1 − e−θ/T e−`0 /(RT ) .
V RT a
dP d 5R `0 3Rθ/T
3◦ ) sv − ss = (vv − vs ) ' RT [ln P ] = + − θ/T (trouvé en faisant
dT dT 2 T e −1
directement la différence des expressions de Ss /ns et Sv /nv .
5 3Rθ
4◦ ) `(T ) = T (sv − ss ) = `0 + RT − θ/T . `(500K) = 112173 J/mole.
2 e −1

xxxxxxxxxxxxxxxxxxxxxxxxxxxx

VI - Retard à la vaporisation

   
1◦ ) dG?t= g` (T0 , P0 ) − gv (T0 , Pv ) dnv + P0 − Pv + 4πr2 dr, g` et gv étant les
r
enthalpies libres molaires respectives du liquide et de la vapeur, Pv la pression de la vapeur.
∂G?t ∂G?t 2σ
   
2◦ ) a) et b) = 0 = g` (T0 , P0 ) − gv (T0 , Pv ), = 0 = P0 − Pv + .
∂nv r ∂r nv r
3◦ ) a) g` (T0 , Ps (T0 )) = gv (T0 , Ps (T0 )).
b) On utilise la relation générale dg = −sdT +vdP , s et v étant l’entropie molaire et le volume

Christian Carimalo 72 TD de Thermodynamique - L3


molaire. Pour le liquide, supposant v` invariable, on obtient g` (T0 , P0 ) = g` (T0 , Ps (T0 )) +
RT
v` [P0 − Ps (T0 )] ; supposant la vapeur assimilable à un gaz parfait, de sorte que vv = , on
Pv
Pv
 
obtient gv (T0 , P0 ) = gv (T0 , Ps (T0 ))+RT0 ln . Les conditions d’équilibre conduisent
 Ps (T0 )
RT0 Pv
alors à l’équation P0 − Ps (T0 ) = ln .
v` Ps (T0 )
Pv 2σ
4◦ ) = 0, 9997 ! r = = 2, 35 10−6 m ; nv = 1, 76 10−15 (Nv = nv NA '
Ps (T0 ) Pv − P0
109 ! !).

xxxxxxxxxxxxxxxxxxxxxxxxxxxx

VII - Fusion du soufre monoclinique


1◦ ) et 2◦ ) : voir le cours.
dP L
 
3◦ ) On utilise la relation de Clapeyron : = x = , tout en supposant cette
dT T ∆V
pente constante. On obtient ainsi ∆T = ∆P/x. A la température de fusion : ∆T =
1, 8 108 × 400 × 40 10−6 × 32 10−3
= 51 K, donc T2 = 451 K ; écrivant ensuite l’égalité
1800
L L T1
   
des pentes aux deux points : = , on trouve ∆2 V = ∆1 V = 51
T ∆V 1 T ∆V 2 T2
cm3 /kg.

xxxxxxxxxxxxxxxxxxxxxxxxxxxx

VIII - Pompe à chaleur - Cycle de Rankine


A/ Laissé au lecteur...
Q1 Q2
B/ 1◦ ) Rappels : W + Q1 + Q2 = 0, ∆S = 0 = + , d’où Q2 = −W η où η =
T1 T2
T2
' 9, 5 est l’efficacité. On obtient |Q2 | = 3, 8 kWh.
T2 − T1
2◦ ) 0, 4 kWh seulement.
3◦ ) On puise dans l’atmosphère ; η = 9, 5.
∂T
 
C/ 1◦ ) et 2◦ ) Note : Dans la transformation D on a ∆S > 0 ; = T /Cp est positif et
∂S P
pratiquement constant à T constant si la vapeur est assimilable à un gaz parfait : le graphe
de R dans le diagramme (T, S) est presque un segment de droite.
∂H
 
3◦ ) Comme dH = V dP + T dS, on a = T . Les transformations bc (V ) et ea (L)
∂S P
s’effectuant à températures constantes, leurs représentations dans le plan (H, S) sont des
segments de droite. A noter aussi que les pentes de de et ea sont les mêmes au point e.
|Qde + Qea | Qde + Qea Qbc
4◦ ) η = = , soit 1/η = 1 + . Les transformations
W Qde + Qea + Qbc Qde + Qea
bc et da s’effectuant à pression constante, on a Qbc = hc − hb , Qda = ha − hd ; en outre,
ha − hd
ha = hb . On en déduit η = .
hc − hd

Christian Carimalo 73 TD de Thermodynamique - L3


T d
P
R
a L
e
C

a L R D
d
e
D C c
b V
V c
b

V S

Figure 11 – Représentations du cycle de Rankine dans les deux plans (P, V ) et (T, S)

H
d

C
e

L
c

a b

Figure 12 – Représentation du cycle de Rankine dans le plan (H, S)

xxxxxxxxxxxxxxxxxxxxxxxxxxxx

IX - Transition phase normale → phase supraconductrice


d’une substance magnétique

A/ 1◦ ) Les trois variables B, P, T sont intensives et nous avons un seul constituant sous
deux phases : v = c + 3 − ϕ = 2. Si P est fixé, il reste une seule variable indépendante, ce
qui fait que B et T doivent être liés par une relation B(T ).
2◦ ) • d̄Wchimique = dns µs +dnn µn = N dx [µs − µn ]. A l’équilibre, les potentiels chimiques
des deux phases sont égaux, µs = µn , et le “travail chimique” est nul. Il reste d̄W = BdM−
P dV , où M est l’aimantation totale 5 ; comme le volume total V reste constant, il n’y a pas
non plus de travail mécanique. Lorsque les deux phases coexistent, M = xMs + (1 − x)Mn .
B
Comme χn = 0, Mn = 0, et Ms = V Ms = − V . On aboutit à d¯W = Bd [xMs ] et
µ 0
BV dB
 
finalement à l’expression d̄ W = − Bdx + x dT .
µ0 dT
5. Lecture conseillée : H. Callen, “Thermodynamics and an introduction to Thermostatistics”, John
Wiley and sons, 1985, p 479.

Christian Carimalo 74 TD de Thermodynamique - L3


• S = N xss +N (1−x)sn où ss et sn sont les entropies molaires respectives de la phase (s) et
s dT + hs dB, ds = C n dT + hn dB (P et V sont fixés). On en
de la phase (n), avec dss =CB n B 
s n
déduit d¯Q = T dS = N dx[ss − sn ] + [xCB + (1 − x)CB ]dT + [xhs + (1 − x)hn ]dB ;
s dB
s + hs n n + hn dB (définis sur la courbe de transition),
en définissant C B = CB , C B = CB
dT dT
et en tenant compte de L = T (sn − ss ), on obtient
s n
h i
d̄ Q = T dS = −N Ldx + dT xC B + (1 − x)C B

d L
 
s n
3◦ ) En exprimant que dS est une d.t.e., il vient −N T = C B −C B . Puis, en exprimant
dT T
i BV  dB

s n
h
que dU = d¯W + d¯Q = −N Ldx + dT xC B + (1 − x)C B − Bdx + x dT est
µ0 dT
s n BV dB dL BV dB
aussi une d.t.e., on obtient C B − C B − = −N −2 . Combinant les deux
µ0 dT dT µ0 dT
relations ainsi trouvées, il vient
NL BV dB
=−
T µ0 dT
c’est-à-dire une relation analogue à la relation de Clapeyron, que l’on peut obtenir plus
simplement en exploitant l’égalité des potentiels chimiques. Ceux-ci, assimilables aux en-
thalpies libres molaires, ont pour différentielles respectives dgs = −mn dB − sn dT , dgs =
−ms dB − ss dT . De l’égalité de ces différentielles
 le long de la courbe de transition, on tire
dB V B dB
ms dB = (sn − ss )dT , soit L = T ms =T − .
dT N µ0 dT

B/ ♣ Etude d’un modèle simple


1◦ ) G? doit être extensif, d’où Ψ(T, N ) = N ψ(T ), nécessairement. L’équation (1) est une
équation fondamentale G? = G? (T, P, B, N ).
∂G?
 
2◦ ) dG? = V dP − SdT − MdB + µdN ; V = = N b, d’où le volume molaire
∂P T,B,N,P
v = b (puisque N = NA ) = constante.
∂V ∂M
   
=0= : M est donc indépendant de P et n’est fonction que de
∂B T,P,N ∂P T,B,N
B, T et N . Comme M est extensif, il doit être de la forme M = N ϕ(B, T ).
∂G?
 
3◦ ) M=− = −N aB, en accord avec 2◦ ).
∂B T,P,N

∂G?
 
4◦ ) S = − = −N ψ 0 (T ). Ce modèle serait éventuellement valable à basse
∂T B,P,N
température si ψ 0 (T ) → 0 lorsque T → 0, en conformité avec le 3ème principe de la Ther-
modynamique.
B2
 
5◦ ) U= G? − P V + T S + B M = −N a + N ψ − T ψ0 .
2
♣ Description des phases (n) et (s) à l’aide du modèle (1)
B V
1◦ ) M = −N aB, Mn = 0, Ms = V Ms = −V : on doit prendre a = pour la
µ0 N µ0
phase (s), a = 0 pour la phase (n).

Christian Carimalo 75 TD de Thermodynamique - L3


B2
gn? = P V + ψn (T ), gs? = V + P V + ψs (T ).
2µ0
B2
2◦ ) gn? = gs? , d’où ψn (T ) − ψs (T ) = V .
2µ0
V dB
3◦ ) sn = −ψn0 , ss = −ψs0 , d’où L = T [sn − ss ] = T [ψs0 − ψn0 ] = − B , etc.
µ0 dT
4◦ ) a)

(n)
B0

(s)

T
T0

" # " #
2V 2 T 2 T2 s n 2V B02 T T2
b) L = B0 2 1 − 2 ; ∆C = C B − C B = 3 2 −1 .
N µ0 T0 T0 N µ0 T02 T0

L ∆C
LM ∆M

T0 /3 T
T0
T
TM T0

5◦ ) ∆M = 3 10−3 J K−1 mole−1 .

xxxxxxxxxxxxxxxxxxxxxxxxxxxx

X - L’atmosphère isotherme

1◦ ) Soit δU l’énergie interne usuelle d’une masse δm du gaz atmosphérique G, dans les
conditions (T, P (z)). L’énergie totale δ Ũ de cette masse dans le champ de pesanteur terrestre
δm
est δU + gzδm. Le nombre de moles contenues dans cette masse étant δn = , écrivons
M
δ Ũ = δn [u + M gz] où u est l’énergie interne molaire usuelle de G dans les conditions
(T, P (z)). A partir de là, on définit l’enthalpie libre généralisée δ G̃ = δ Ũ + P δV − T δS =

Christian Carimalo 76 TD de Thermodynamique - L3


δn [g + M gz], où g est l’enthalpie libre molaire usuelle. Le potentiel chimique d’une tranche
δ G̃
δz de G est ainsi µ = = g(T, P (z)) + M gz.
δn
2◦ ) Chaque “phase” de l’atmosphère est indexée par son altitude z. Comme l’atmosphère
est en équilibre, les potentiels chimiques de toutes ces phases doivent être égaux. Autrement
dit, le potentiel chimique défini au 1◦ ) est en fait indépendant de z.
P (z)
3◦ ) a) Considérant G comme un gaz parfait, on a g(T, P (z)) = RT ln + ψ(T ) où ψ(T )
P1
est une fonction de la température uniquement et P1 une constante homogène à une pression.
P (z)
Ecrivant g(T, P (z)) = g(T, P (0)), on aboutit à une relation de la forme RT ln =
P0 (T )
M gz
 
−M gz, soit P (z, T ) = P0 (T ) exp − .
RT
b) Ecrivons la condition d’équilibre mécanique d’une masse dm de G occupant le volume de
gaz dV = dzdS entre les altitudes z et z + dz. Les forces de pressions s’exerçant sur les
côtés parallèles à l’axe des z s’équilibrant, il reste gdm − [P (z + dz) − P (z)] dS = 0, soit
dm MP 1 dP M
P (z + dz) − P (z) = − g = −dzg ou = −g , équation dont l’intégration
dS   RT P dz RT
M gz
donne P (z, T ) = P0 (T ) exp − .
RT

P(z + dz)

dS
dz

dS

P(z)

xxxxxxxxxxxxxxxxxxxxxxxxxxxx

XI - Machine frigorifique

- A - Etude du cycle du fluide réfrigérant

1◦ ) Voir figure (13).


2◦ ) a) ∆AA0 H = c` (T2 − T1 ) ; ∆A0 B H = xLv (T2 ).
T1 − T2
b) Or, ∆AB H = 0, donc x = c` = 0, 215.
Lv (T2 )
T2 Lv (T2 ) T2 T1
 
c) ∆AB S = ∆AA0 + ∆A0 B S = c` ln +x = c` ln −1+ = 8, 8 J K−1
T1 T2 T1 T2
kg−1 .

Christian Carimalo 77 TD de Thermodynamique - L3


P

D
A
D’
T’1
A’ B C T1
T2

Figure 13 – Cycle d’une machine frigorifique

QBC
3◦ ) QBC = (1 − x)Lv (T2 ) = 161000 J kg−1 ; ∆BC S = = 589, 5 J K−1 kg−1 .
T2
 γ−1
P1

γ
4◦ ) a) T10 = T2 = 318, 7 K.
P2
R
b) QCD = 0, donc WCD = ∆CD U = ncv ((T10 − T2 ) = (T 0 − T2 ) = 22000 J kg−1 .
M (γ − 1) 1
c) ∆CD H = ncp (T10 − T2 ) = γWCD = 26000 J kg−1 .

5◦ ) QDA = QDD0 + DD0 A = (T1 − T10 ) − Lv (T1 ) = −182900 J kg−1 ; ∆DA S =
M (γ − 1)
Rγ T1 Lv (T1 )
ln 0 − = −598 J K−1 kg−1 .
M (γ − 1) T1 T1
QBC
6◦ )  = = 7, 33.
WC D
- B - Refroidissement de la chambre froide
.
dQ1 dQ2
7◦ ) dU = dQ1 + dQ2 + dW = 0, dS = + = 0, dW = P dt, dQ2 = −CdT , d’où
T0 T
dt C T0
 
= 1− .
dT P T
C T
 
8◦ ) t(T ) = T − T0 − T0 ln .
P T0
9◦ ) τ = t(TF ) = 8397 s = 2h 20 mn.

xxxxxxxxxxxxxxxxxxxxxxxxxxxx

Christian Carimalo 78 TD de Thermodynamique - L3


TD8

Statistique de Boltzmann, entropie statistique

I - Loi barométrique
A/ Dans certaines conditions, on modélise l’atmosphère terrestre par un gaz parfait constitué
de particules de masse m à la température T constante. L’accélération de la pesanteur g sera
considérée comme indépendante de l’altitude.
1◦ ) Déterminer, en fonction de l’altitude z, la pression P (z) du gaz atmosphérique ainsi que
sa densité moléculaire n(z), nombre par unité de volume de particules à cette altitude. On
notera P0 et n0 leurs valeurs respectives à l’altitude z = 0.
2◦ ) Quelle “hauteur caractéristique” H intervient dans l’expression de n(z) ? Montrer que
H définit la valeur moyenne < z > de l’altitude des particules. Calculer < z > pour des
particules sphériques de rayon r = 0, 2 µm et de densité ρ = 1, 2 g cm−3 . Est-il matériellement
possible d’observer la répartition de ces particules suivant z ?
3◦ ) Expérience de Jean Perrin.
a) Comment est modifiée la répartition n(z) des particules si elles se trouvent en suspension
dans l’eau à la température T ? On donne la densité de l’eau : ρe = 1 g cm−3 .
b) En observant au microscope une suspension de petites sphères de gomme-gutte (rayon
r = 0, 2 µm, densité ρ = 1, 2 g cm−3 ) dans de l’eau à T = 293 K, Jean Perrin a compté 100
de ces particules à l’altitude de référence et environ 17 de ces particules à z = 90 µm. En
déduire une valeur de la constante de Boltzmann, puis de la constante d’Avogadro.

B/ Pour modéliser la troposphère, couche atmosphérique se trouvant entre les altitudes 0 et


dT
11 km, on ne considère plus la température constante mais qu’elle varie selon la loi = −a
dz
où a est une constante positive. On suppose encore l’air comme un gaz parfait de masse
molaire M = 29 g. A l’altitude z = 0, la pression et la température sont respectivement
P0 = 1 atm et T0 = 290 K.
1◦ ) Etablir les répartitions T (z), n(z), P (z).
2◦ ) A z = 200 m, la température a diminué de 1, 4 K par rapport à T0 . Déterminer l’altitude
z1 pour laquelle la pression a diminué de moitié. Que vaut alors T (z1 ) ?

II - Oscillateur harmonique
On considère N oscillateurs harmoniques à une dimension, sans interaction, tous de même
masse m et de même pulsation propre ω, en contact avec un thermostat de température T .
A/ Traitement classique
L’énergie mécanique d’un oscillateur de quantité de mouvement p et de position x est donnée
par

Christian Carimalo 79 TD de Thermodynamique - L3


p2 1
E(p, x) = + m ω 2 x2
2m 2

La probabilité qu’un oscillateur possède cette énergie à dp et dx près à la température T est


donnée par

E(p, x)
 
P (p, x)dpdx = A exp − dpdx
kT

où A est une constante et k la constante de Boltzmann.


1◦ ) Déterminer A.
2◦ ) Donner le nombre moyen dN (p, x) d’oscillateurs d’énergie E(p, x) à dp et dx près.
3◦ ) Déterminer l’énergie moyenne totale de l’ensemble des N oscillateurs. En déduire la
capacité calorifique Cv de cet ensemble.
4◦ ) Déterminer les valeurs moyennes de l’énergie cinétique et de l’énergie potentielle d’un
seul oscillateur. Conclusion ?

B/ Traitement quantique
En mécanique quantique, l’énergie d’un oscillateur est quantifiée : elle ne peut prendre que
1

les valeurs discrètes données par En = n + h̄ ω où n est un entier positif ou nul. La
2
probabilité Pn qu’un oscillateur possède cette énergie est maintenant

1 En
 
Pn = exp −
Z kT

1◦ ) Déterminer Z.
2◦ ) En déduire l’énergie interne U de l’ensemble des N oscillateurs puis Cv .
3◦ ) Quelles sont les limites de U et Cv lorsque kT  h̄ ω et lorsque kT  h̄ ω ?
4◦ ) Tracer qualitativement les variations en fonction de T de U et Cv .

III - Orientation de dipôles électriques


On considère un gaz formé de N molécules de chlorure d’hydrogène HCl contenu dans un
−→
volume V à la température T , placé dans un champ électrique constant E orienté suivant
l’axe Oz. Les molécules HCl possèdent un moment dipolaire électrique p = 0, 36 10−29 Cm.
Ces dipôles vont avoir tendance à s’orienter selon le champ électrique.
1◦ ) Rappeler l’expression de l’énergie d’interaction du dipôle électrique avec le champ électrique
extérieur.
2◦ ) On admet que l’orientation d’un dipôle est indépendante des autres. Déterminer la proba-
bilité pour le moment dipolaire d’une molécule pointe dans un angle solide dΩ = sin θdθdϕ,
−→
dans une direction faisant l’angle θ avec E .
−→
3◦ ) En déduire la polarisation P , moment dipolaire moyen par unité de volume, puis la
permittivité relative définie comme

Christian Carimalo 80 TD de Thermodynamique - L3


P
 
εr = lim 1+
E→0 ε0 E

Comment varie-t-elle avec la température ?


4◦ ) Déterminer l’énergie moyenne du gaz puis sa capacité calorifique.

IV - Paramagnétisme
A/ Traitement classique
−→
Placé dans un champ magnétique B , l’air se comporte comme une substance paramagné-
tique. Ceci vient du fait que la molécule d’oxygène possède un moment magnétique permanent
−→
m.
On considère N molécules d’oxygène dans un volume V à la température T . Le champ
−→
magnétique B est orienté suivant l’axe Oz. L’énergie d’interaction entre le dipôle magnétique
et le champ magnétique extérieur s’écrit Em = −mz B où mz est la composante du moment
magnétique suivant la direction du champ appliqué.
1◦ ) On admet que l’orientation d’un dipôle est indépendante des autres. Déterminer la proba-
bilité pour le moment dipolaire d’une molécule pointe dans un angle solide dΩ = sin θdθdϕ,
−→
dans une direction faisant l’angle θ avec B .
2◦ ) En déduire l’aimantation M , moment magnétique moyen par unité de volume, puis
M

la susceptibilité magnétique définie par χm = lim . Comment varie-t-elle avec la
B→0 B
température ?
3◦ ) Déterminer l’énergie moyenne du gaz due à l’interaction magnétique et la capacité calo-
rifique correspondante.

B/ Traitement quantique
On considère une assemblée de N atomes dans un volume V à la température T . Chaque
−→
atome possède un moment magnétique permanent m . On applique un champ magnétique
−→
constant B . On suppose ici que la projection du moment magnétique d’un atome sur
−→
la direction de B ne peut prendre que deux valeurs : +m, ce qui conduit à l’énergie
E1 = −mB, ou −m, conduisant à l’énergie E2 = +mB.
1◦ ) Pour un atome, quelles sont les probabilités P1 = P (−mB) et P2 = P (+mB) de se
trouver dans les états d’énergies E1 et E2 respectivement ?
2◦ ) Déterminer la valeur moyenne < m > du moment magnétique d’un atome.
3◦ ) En déduire l’expression de l’aimantation M .
4◦ ) Déterminer l’énergie moyenne U de l’ensemble des N atomes, associée à cette interaction
et la capacité calorifique correspondante Cv .
5◦ ) Que deviennent les expressions de M , U et Cv lorsque kT  mB et lorsque kT  mB ?

Christian Carimalo 81 TD de Thermodynamique - L3


V - Calcul du nombre de micro-états d’un gaz parfait
Une particule de masse m est contenue dans une boı̂te cubique de volume V = L3 (0 ≤
x ≤ L, 0 ≤ y ≤ L, 0 ≤ z ≤ L). Selon la mécanique quantique, l’énergie de la particule est
quantifiée et ne peut prendre que les valeurs données par

h2  2 2 2

Enx ,ny ,nz = n x + n y + n z
8mL2

h étant la constante de Planck et nx , ny , nz sont des entiers relatifs.


1◦ ) Déterminer l’ordre de grandeur du nombre d’états quantiques Ω(E) distincts dont
l’énergie est inférieure ou égale à E.
2◦ ) En déduire la forme du nombre Ω(U, V, N ) de micro-états accessibles à un ensemble de
N particules indépendantes d’énergie totale U contenues dans un volume V .

VI - Chaı̂ne polymérique linéaire


Soit une chaı̂ne polymérique linéaire comportant N chaı̂nons de longueur a. Un chaı̂non
particulier ne possède pas de centre d’inversion et l’on peut lui associer un vecteur pointant
soit vers la droite, soit vers la gauche. En l’absence de force extérieure, chaque chaı̂non a une
probabilité égale d’être droit ou gauche. On note L la longueur totale de la chaı̂ne et T la
température.
1◦ ) Calculer le nombre de configurations de la chaı̂ne conduisant à une longueur totale L.
2◦ ) Calculer l’entropie de la chaı̂ne lorsque a  L  N a (on remplacera la distribution
binomiale par sa limite gaussienne). Montrer que S(L) = S0 − AL2 et déterminer S0 et A.
3◦ ) Calculer la force F à appliquer pour que la longueur de la chaı̂ne soit égale à L.

VII - Gaz parfait : micro-états et entropie - 1


Le nombre de micro-états accessibles à N molécules d’un gaz parfait monoatomique d’énergie
interne U contenues dans un volume V peut se mettre sous la forme

Ω(U, V, N ) = C(N ) U 3N/2 V N

1◦ ) Déterminer l’entropie et la température du gaz à l’équilibre en fonction de U, V, N . On


notera k la constante de Boltzmann.
2◦ ) En imposant à l’entropie d’être extensive, déterminer l’équation vérifiée par C(N ) puis
montrer que C(N ) doit être de la forme C(N ) = eα N N −5N/2 , avec α réel.

On considère un récipient rigide de volume V séparé en deux compartiments (1) et (2) par
une paroi mobile et diatherme. Le récipient est isolé thermiquement. On introduit initialement
dans le compartiment (1) N1 molécules de gaz parfait à la température initiale T1i , et dans
le compartiment (2) N2 molécules de gaz parfait à la température T2i . On note U1i et U2i
les énergies internes initiales des gaz (1) et (2) respectivement et U l’énergie interne totale
de l’ensemble des deux gaz. Une fois l’équilibre atteint, la température est Tf et la pression
Pf dans chaque compartiment.
3◦ ) Déterminer Tf et Pf en fonction de V, U, N1 , N2 .

Christian Carimalo 82 TD de Thermodynamique - L3


4◦ ) Déterminer les volumes V1f , V2f et les énergies internes U1f et U2f des deux gaz à l’état
d’équilibre en fonction de V, U, N1 , N2 .
5◦ ) Partant de cet état d’équilibre, on enlève la paroi. Déterminer la variation consécutive
d’entropie ∆S du système isolé (récipient de volume V ) dans les deux cas suivants : les deux
gaz sont différents ; les deux gaz sont identiques. Conclure.

VIII - Gaz parfait : micro-états et entropie - 2


Le nombre de micro-états accessibles à N molécules d’un gaz parfait monoatomique d’énergie
interne U contenues dans un volume V est donné par
N  3N/2
V U

Ω(U, V, N ) = e5N/2
N v0 N u0

où v0 est un volume et u0 une énergie, petite devant kT , k étant la constante de Boltzmann.
On considère 2N molécules de gaz parfait à la température T0 , contenues dans un récipient de
volume 3 V comportant deux compartiments : l’un, (1), de volume V , contient N molécules,
l’autre, (2), de volume 2 V , contient aussi N molécules. La paroi séparant les deux compar-
timents est diatherme.
1◦ ) Calculer Ω1 (U, V, N ) et Ω2 (U, 2V, N ) et le nombre total Ω0 de micro-états accessibles
de l’ensemble.
2◦ ) On enlève la paroi. Quels sont alors les nombres moyens de molécules respectifs < N1 >
et < N2 > dans les compartiments (1) et (2) ?
< N1 >
3◦ ) On remet la paroi dans sa position initiale. Calculer Ω1 ( U, V, < N1 >) et
N
< N2 >
Ω2 ( U, 2V, < N2 >) ainsi que le nombre total ΩT de micro-états accessibles. Com-
N
parer ΩT à Ω0 . Quelle conclusion en tire-t-on sur l’irréversibilité du processus ? Application
numérique : calculer le rapport ΩT /Ω0 pour N = 1010 .
4◦ ) On s’intéresse toujours au même dispositif mais cette fois avec N1 molécules dans le
compartiment (1) et 2N − N1 molécules dans le compartiment (2). On pose a = V /v0 .
N1
a) Etudier la fonction entropie S1 = S1 ( U, V, N1 ) du gaz du compartiment (1) lorsque
N
celui-ci contient N1 molécules à la température T0 . Pour N1  a, tracer S1 en fonction de
N1 .
2N − N1
b) Même questions pour, successivement, l’entropie S2 = S2 ( U, 2V, 2N − N1 ) du
N
gaz du compartiment (2) et pour l’entropie totale ST = S1 + S2 . Revenir ensuite sur le
résultat du 3◦ ).
5◦ ) On pose x = N1 − < N1 >. Donner le développement de ST (x) au second ordre en x. En
déduire que ΩT (x), nombre total de micro-états accessibles pour x donné est une gaussienne
selon cette variable. En donner la variance.

IX - Entropie statistique d’un gaz réel


Le nombre de micro-états accessibles à N molécules d’un gaz réel d’énergie interne U occu-
pant un volume V est donné par

Christian Carimalo 83 TD de Thermodynamique - L3


N !3N/2
V − NB U + AN 2 /V

Ω(U, V, N ) = e5N/2
N v0 N u0

A, B, v0 , u0 étant des constantes.


1◦ ) Déterminer l’entropie de ce gaz en fonction de U, V et N . On notera k la constante de
Boltzmann.
2◦ ) En utilisant l’identité thermodynamique, déterminer la température T du gaz puis sa
pression P et son potentiel chimique µ.
3◦ ) Exprimer U et S en fonction de T, V, N .
4◦ ) Montrer qu’en fonction de T et du volume molaire v, l’énergie interne molaire u et
l’entropie molaire s s’expriment comme

a 3R 3R
u(T, v) = − + T + u00 , s(T, v) = R ln(v − b) + ln T + s0
v 2 2

où R est la constante des gaz parfaits. On définira les constantes a, b en fonction de A, B et
du nombre d’Avogadro NA , puis on donnera les expressions de u00 et s0 .

Christian Carimalo 84 TD de Thermodynamique - L3


Corrigé TD8

I - Loi barométrique

M gz
 
A/ 1◦ ) P (z) = n(z)kT = P0 exp − .
RT
∞ ∞ −1 ∞

RT d
Z Z  Z
2◦ ) e−αz dz

H= ; < z >= z n(z)dz n(z)dz =− ln = H.
Mg 0 0 dα 0 α=1/H
mg 4πr3
H = avec m = ρv, v = ; on trouve m = 4 10−15 kg, puis H = 10 µm : difficile
kT 3
matériellement d’observer la répartition des particules suivant z.
3◦ ) Expérience de Jean Perrin.
a) La densité apparente change et devient ρ0 = ρ − ρe (Archimède), d’où
H 0 = 6H = 60 µm.
b) Posons r = n(90µm)/n(0) = 17/100 ; on obtient
R m0 gz RT ln(1/r)
k= = , NA = = 6, 8 1023 .
NA T ln(1/r) m0 gz

dP m mP 1 dP
B/ 1◦ ) T (z) = T0 − az ; = −ρg avec ρ = NA = , d’où l’équation =
dz Vmol. kT (z) P dz
m az b mg
 
− . En l’intégrant, on trouve P (z) = P0 1 − , avec b = , puis n(z) =
k(T0 − az) T0 ka
P (z) P0 az b
 
= 1− .
kT (z) k(T0 − az) T0
2◦ ) a = 7 10−3 K/m ; z1 = 5474 m ; T1 = 251, 7 K = −21, 5 ◦ C.

xxxxxxxxxxxxxxxxxxxxxxxxxxxx

II - Oscillateur harmonique

ω
Z
A/ 1◦ ) A est fixé par la condition de normalisation P (p, x)dpdx = 1. On trouve A = .
2πkT
2◦ ) dN (p, x) = N P (p, x)dpdx.
1
Z Z
3◦ )
U = N u = N E P (p, x)dpdx. Posons β = . On a U = N A Ee−βE dpdx =
kT
∂ ∂ 1 Nω
Z
−N A e−βE dpdx = −N A = , soit U = N kT , puis Cv = N k.
∂β ∂β A 2πA
p2 mω 2 x2
Z Z
4◦ ) Ec = , Ep = , < Ec >= Ec P (p, x)dpdx, < Ep >= Ep P (p, x)dpdx ; on
2m 2
kT
trouve < Ec >=< Ep >= : il y a équipartition de l’énergie.
2

Christian Carimalo 85 TD de Thermodynamique - L3


∞ ∞
En √ X h̄ω
X    
n
B/ 1◦ )Condition de normalisation : Z = exp − = q q où q = exp − ,
n=0
kT n=0
kT

q 1
soit Z = = .
1−q h̄ω
2 sinh
kT

X ∂ h̄ω h̄ω
2◦ ) Comme Z = exp (−βEn ), on a U = −N ln Z = N coth ; puis
n=0
∂β 2 2kT
2
h̄ω 1

Cv = N k .
2kT h̄ω
2 sinh2
kT
3◦ ) Pour kT  h̄ ω, on obtient U ' N kT , Cv ' N k : on retrouve le cas classique ; pour
N h̄ω
kT  h̄ ω, ' et Cv → 0.
2
4◦ )

U Cv

xxxxxxxxxxxxxxxxxxxxxxxxxxxx

III - Orientation de dipôles électriques

−→ −→
1◦ ) Ep = − p · E = −pE cos θ.
1

Ep
 Z 
Ep
 1 Z
pE
2◦ ) dP = exp − dΩ, avec Z = exp − dΩ = 2π du exu où x = , soit
Z kT kT −1 kT
sinh x
Z = 4π .
x
N N N ∂ 1
3◦ ) Px = Py = 0, Pz = < p cos θ >= − < Ep >= ln Z où β = , soit
V VE V E ∂β kT
Np d sinh x 1
encore Pz = L(x) où L(x) = ln = coth x − est la fonction de Langevin.
V dx x x
x N p2 N
Lorsque E → 0, on a x  1 et L(x) ' . D’où r = 1 + . On note que =
3 3V 0 kT V
pression 1
, donc r − 1 ∝ 2 .
kT T
dU 1 dU d
4◦ ) U = N < Ep >= −V EPz = −N pEL(x) ; Cv = =− 2 = N kx2 L(x),
" # dT kT dβ dx
x2
soit Cv = N k 1 − . A noter ici que ce U et ce Cv sont relatifs à l’interaction entre
sinh2 x
les dipôles électriques et le champ extérieur. L’énergie interne complète du gaz doit tenir
compte de l’energie de translation, de l’énergie de rotation, voire de l’énergie de vibration
des molécules. Si l’on peut considérer le gaz comme parfait, le théorème d’équipartition de

Christian Carimalo 86 TD de Thermodynamique - L3


3N kT
l’énergie attribue l’énergie à la première et l’énergie N kT à la seconde.
2
xxxxxxxxxxxxxxxxxxxxxxxxxxxx

IV - Paramagnétisme
A/ Traitement classique
1

Em
 Z
Em
  1 Z
mB
1◦ ) dP = exp − dΩ, avec Z = exp − dΩ = 2π du exu où x = ,
Z kT kT −1 kT
sinh x
soit Z = 4π .
x
N N N ∂ 1
2◦ ) Mx = My = 0, Mz = < m cos θ >= − < Em >= ln Z où β = , soit
V VB V B ∂β kT
Nm d sinh x 1
encore Mz = L(x) où L(x) = ln = coth x − est la fonction de Langevin.
V dx x x
x N m2
Lorsque B → 0, on a x  1 et L(x) ' . D’où χm = 1 + . On note que
3 3V 0 kT
N pression 1
= , donc χm − 1 ∝ 2 .
V kT T
dU 1 dU d
3◦ ) U = N < Em >= −V BMz = −N mBL(x) ; Cv = =− 2 = N kx2 L(x),
" # dT kT dβ dx
x2
soit Cv = N k 1 − .
sinh2 x

B/ Traitement quantique
ex e−x
1◦ ) P 1 = , P2 = , avec Z = ex + e−x = 2 cosh x.
Z Z
m x Nm
2◦ ) et 3◦ ) < m >= e − e−x = m tanh x ; Mz =

tanh x.
Z V
N mB dU x2
4◦ ) U = −N mB tanh x, Cv = − = N k .
kT 2 dx cosh2 x
Nm
5◦ ) Pour kT  mB, soit x  1, on a Mz ' x, U ' −N mBx, Cv ' N kx2 . Ces
V
grandeurs tendent vers 0 : l’agitation thermique devient grande et l’on a un désalignement
des moments magnétiques.
Nm
Pour kT  mB, soit x  1, Mz ' , U ' −N mB, Cv ' 4N kx2 e−2x → 0 : pour de
V
très basses températures, on a un alignement complet des moments magnétiques, le degré
de liberté correspondant à l’interaction magnétique est alors “gelé”.

xxxxxxxxxxxxxxxxxxxxxxxxxxxx

V - Calcul du nombre de micro-états d’un gaz parfait


s
h2
1◦ ) Posons ` = , puis x = nx `, y = ny `, z = nz `, de sorte que Enx ,ny ,nz =
8mL2
2x + 2y + 2z . Dans un espace (x , y , z ), on peut représenter l’état correspondant aux trois
nombres nx , ny et nz , par un point de coordonnées `(nx + 1/2), `(ny + 1/2), `(nz + 1/2)
situé au centre d’un cube dont les côtés ont pour longueur `. On passe de cet état à un état

Christian Carimalo 87 TD de Thermodynamique - L3


voisin en faisant varier les nombres nx , ny et nz de ±1, et ce faisant, on passe dudit cube
à l’un de ses 6 cubes voisins. Chaque état (nx , ny , nz ) se voit ainsi attribuer un et un seul
cube de volume v = `3 . Comme un changement de signe des entiers nx , ny et nz conduit
à la même énergie Enx ,ny ,nz , à un état d’énergie donnée correspond en fait 8 cubes dans
l’espace (x , y , z ), définissant dans cet espace un certain domaine ∆ de volume v 0 = 8`3 .
Pour évaluer le nombre d’états correspondant chacun à une énergie donnée inférieure ou
égale à E il suffit de compter le nombre de domaines ∆ contenus dans la sphère définie
√ 4πR3
par 2x + 2y + 2z ≤ R2 , où R = E. Ce nombre est évidemment Ω(E) = , soit
3v 0
4πV 3
Ω(E) = 2
(2mE) 2 .
3h
3N
2◦ ) Ω(U, V, N ) ∝ V N U 2 (et tenir compte éventuellement de l’indiscernabilité des parti-
cules en divisant par N !).

xxxxxxxxxxxxxxxxxxxxxxxxxxxx

VI - Chaı̂ne polymérique linéaire

1◦ ) Soit n+ et n− les nombres de chaı̂nons pointant, respectivement, vers la droite et vers


la gauche. On a N = n+ + n− , L = a(n+ − n− ), et le nombre de configurations cherché est
N!
Ω = CnN+ =
n+ ! n− !
L N n
 
2◦ ) Posons n = n+ −n− = . On a n± = 1± . Supposant n  1 et n  N , on uti-
a 2 N √ 1
lise tout d’abord la formule de Stirling pour exprimer les factorielles : N ! ' 2π N N + 2 e−N ,
etc, d’où
1 1 N 1 N
  
ln Ω ' ln √ + (N + ) ln N − (1 + x) + ln + ln(1 + x)
2π 2 2 2 2

N 1 N
  
− (1 − x) + ln + ln(1 − x)
2 2 2
où l’on a posé x = n/N . Puis, tenant compte de ce que x  1, on effectue un développement
limité de ln Ω. On remarque que ln Ω est une fonction paire de x et le premier terme de son
développement est du second ordre en x. On peut procéder à ce développement en
 calculant
d N 1−x 1 1 1

les deux dérivées premières de ln Ω : ln Ω = ln + − = 0 pour
dx  2 1 + x 2 1 − x 1 + x
d2 N 1 1 1 1 1
 
x = 0 ; 2 ln Ω = − + + 2
+ = −N +1 pour x = 0.
dx 2 1−x 1+x 2 (1 − x) (1 + x)2
x2 2N +1
On obtient ainsi ln Ω ' ln Ω0 − N (N  1), avec Ω0 = √ . L’entropie de la chaı̂ne
2 2πN
k
est S = k ln Ω = S0 − AL2 où S0 = k ln Ω0 , A = .
2N a2
∂S LkT
 
3◦ ) F = −T = .
∂L U N a2

xxxxxxxxxxxxxxxxxxxxxxxxxxxx

Christian Carimalo 88 TD de Thermodynamique - L3


VII - Gaz parfait : micro-états et entropie - 1

3N k ∂S 1 3N k
 
1◦ )S = k ln Ω = k ln C(N ) + ln U + N k ln V ; = = , soit U =
2 ∂U V T 2U
3N kT
.
2
2◦ ) On doit avoir S(λU, λV, λN ) = λS(U, V, N ), quel que soit le réel (positif) λ, ou
5N λ
Ω(λU, λV, λN ) = Ωλ (U, V, N ), soit C(λN )λ 2 = C λ (N ). Comme cette relation doit être
5N
vraie pour tout λ > 0, on peut choisir λ = 1/N , ce qui conduit à C(N ) = C(1)N N − 2 , ou
encore, C(N ) = eα N N −5N/2 , en posant C(1) = eα où α est un réel. Ce résultat peut être
également déduit en raisonnant comme suit. L’entropie étant extensive doit pouvoir s’écrire
U V
sous la forme S(U, V, N ) = N Ψ( , ). Introduisant les variables intensives U/V et V /N ,
S h 5 1 i 3 N UN V
on a = ln N 2 C N (N ) + ln + ln . La forme préconisée est obtenue si et seule-
kN h 5 1 i 2 N N
ment si ln N 2 C N (N ) est égal à une constante réelle α, indépendante de N . On obtient
ainsi
  3N  N
αN U 2 V
Ω(U, V, N ) = e
N N

3N1 kT1i 3N2 kT2i 3k


3◦ ) A l’état initial, U1i = , U2i = , Ui = U1i + U2i = [N1 T1i + N2 T2i ] ;
2 2 2
3N1 kTf 3N2 kTf 3N kTf
à l’état final, U1f = , U2i = , Uf = . A partir du moment où
2 2 2
les deux gaz sont présents dans leurs compartiments respectifs, l’ensemble des deux gaz
effectue une transformation adiabatique au cours de laquelle il ne reçoit aucun travail. Par
N1 T1i + N2 T2i N1 kTf N2 kTf
conséquent, Uf = Ui et Tf = . Puis P1f = P2f = Pf = = =
N V1f V2f
k(N1 + N2 )Tf N kTf 2U 2U
= ; Tf = , Pf = .
V1f + V2f V 3N k 3V
N1 V N2 V N1 U N2 U
4◦ ) V1f = , V2f = , U1f = , U2f = .
N N N N
5◦ ) A l’état initial f de cette nouvelle transformation, le nombre de micro-états de l’ensemble
des deux gaz est, compte tenu des résultats de la question 4◦ ),

 5N  N
U V

2
Ωfens. = Ω1 (U1f , V1f , N1 )Ω2 (U2f , V2f , N2 ) = e α1 N1 α2 N2
e
N N

On admet que l’enlèvement de la paroi ne demande aucun travail. Par conséquent l’ensemble
“récipient” est un système isolé et son énergie interne au nouvel état final f 0 est la même
que celle de l’état initial f . Admettant de plus que les deux gaz parfaits s’ignorent lors de
leur mélange, on en déduit que la température à l’issue de l’opération est encore Tf . D’où
U1f 0 = U1f , U2f 0 = U2f . Dans l’état final f 0 , chacun des gaz occupe le volume total V .
• Si les gaz sont différents et s’ignorent, le nombre de micro-états de l’ensemble à l’état f 0
est
 5N  N1  N2
U V V

0 2
Ωfens. = Ω1 (U1f , V, N1 )Ω2 (U2f , V, N2 ) = e α1 N1 α2 N2
e
N N1 N2

Christian Carimalo 89 TD de Thermodynamique - L3


0
" #
Ωfens. N N
 
et l’on a ∆S = k ln f
= k N1 ln + N2 ln > 0, la positivité de cette variation
Ωens. N1 N2
rendant compte de l’irréversibilité de l’opération.
• Si les gaz sont identiques, la description de l’état final est toute autre. En effet, on a alors
un unique gaz parfait de N molécules occupant le volume V et de température Tf . Le nombre
  5N  N
0 U 2 V
de micro-états correspondant est Ωfens. = Ω(U, V, N ) = eαN = Ωfens. (α1 =
N N
α2 = α), d’où ∆S = 0 : les molécules étant indiscernables, le processus d’enlèvement de la
paroi doit alors être considéré comme réversible.

xxxxxxxxxxxxxxxxxxxxxxxxxxxx

VIII - Gaz parfait : micro-états et entropie - 2

1◦ ) Les gaz se trouvant dans l’un ou l’autre compartiment étant à la même température
3N kT0
T0 , leurs énergies internes respectives sont U1 = U2 = U = . Les nombres de
2
micro-états accessibles correspondant aux compartiments (1) et (2) sont respectivement
N 
V U 3N/2 5N/2
 
Ω1 (U, V, N ) = e , Ω2 (U, 2V, N ) = 2N Ω1 (U, V, N ), et le nombre
N v0 N u0
de micro-états accessibles pour l’ensemble est Ωens. = 2N Ω21 (U, V, N ).
2◦ ) A l’équilibre, la densité particulaire est uniforme :

< N1 > < N2 > < N1 > + < N2 > 2N


= = =
V 2V 3V 3V
2N 4N
d’où < N1 >= , < N2 >= .
3 3
2N
3◦ ) Les premier et second compartiments contiennent en moyenne, respectivement,
3
4N
molécules et molécules. La température restant la même, les énergies internes corres-
3
2U 4U
pondantes sont en moyenne U1 = et U2 = . On a
3 3
  2N   2N  N
< N1 > 3 3 V 3 U
Ω1 ( U, V, < N1 >) = e5N/3 ,
N 2 N v0 N u0
  4N   4N  2N
< N2 > 3 3 V 3 U
Ω2 ( U, V, < N2 >) = e10N/3 , et
N 2 N v0 N u0
 2N
3
ΩT = Ω1 (· · ·)Ω2 (· · ·) = Ω21 (U, V, N )
2
On obtient donc ΩT /Ω0 = (9/8)N > 1. On en tire comme conclusion que dans le processus
d’enlèvement puis de remise de la cloison de séparation, il y a eu croissance d’entropie :
9
∆S = N k ln . Ce résultat n’est pas étonnant car on est passé d’un état initial “ordonné”
8
avec N molécules dans chacun des compartiments de volumes différents à un état désordonné,
plus probable, correspondant à une uniformité de densité particulaire dans l’ensemble des deux
compartiments. Le résultat indique aussi que ce processus est irréversible. Pour N = 1010 ,
ΩT /Ω0 ' 109 .

Christian Carimalo 90 TD de Thermodynamique - L3


4◦ ) Rédaction laissée au lecteur.
5◦ ) L’entropie totale s’écrit S = S0 − kN1 ln N1 − kN2 ln N2 + kN2 ln 2. Posant N2 =
∂S
2N − N1 , S se ramène à une fonction de N1 dont les deux premières dérivées sont =
∂N1
2N − N1 ∂2S k k 2N
k ln −k ln 2, = − . La dérivée première est nulle pour N1 =
N1 ∂N12 2N − N1 N1 3
3
et pour cette valeur de N1 , la dérivée seconde vaut −k < 0, ce qui indique que cette
4N
valeur de N1 correspond au maximum de l’entropie. Développons S au second ordre " en x#:
3 2 3x2
S = S0 − k x . Avec cette expression approchée on trouve ΩT (x) = ΩT (0) exp − .
4N 4N
Il s’agit d’une gaussienne dont la variance est 2N/3.

xxxxxxxxxxxxxxxxxxxxxxxxxxxx

IX - Entropie statistique d’un gaz réel


Le nombre de micro-états accessibles à N molécules d’un gaz réel d’énergie interne U occu-
pant un volume V est donné par
N !3N/2
V − NB U + AN 2 /V

Ω(U, V, N ) = e5N/2
N v0 N u0

A, B, v0 , u0 étant des constantes.


V − NB 3N k U + AN 2 /V
1◦ ), 2◦ ), 3◦ ) S(U, V, N ) = k ln Ω(U, V, N ) = kN ln + ln +
N v0 2 N u0
5N k
.
2
∂S 1 3N k 1 3N kT N 2A
 
• == ou U = − ;
∂U
V,N T 2 U + AN 2 /V 2! V
∂S N kT 3N kT AN 2 1 N kT N 2A
 
•P =T = + − 2 = − ;
∂V U,N V − NB 2 V U + N 2 A/V V − NB V2
∂S 2N Ak V − N B 3kT 3kT kT V
 
• µ = −T = −kT − − kT ln − ln +
∂N U,V V N v0 2 2u0 V − NB
4◦ ) Rédaction laissée au lecteur.

xxxxxxxxxxxxxxxxxxxxxxxxxxxx

Christian Carimalo 91 TD de Thermodynamique - L3


TD9

- Théorie cinétique -

I - Vitesses moyennes
Dans un gaz homogène et isotrope à l’équilibre thermodynamique à la température T , le
−→
nombre dNv de particules du gaz dont la vitesse est comprise entre v de composantes
−→ −→
(vx , vy , vz ) et v + dv de composantes (vx + dvx , vy + dvy , vz + dvz ) est donné par la
distribution de Maxwell-Boltzmann :

m 2
 
dNv = A exp − (v + vy2 + vz2 ) dvx dvy dvz
2kT x

m étant la masse des particules et k la constante de Boltzmann.


1◦ ) Calculer la constante A sachant que le nombre total de particules est N .
2◦ ) Calculer la probabilité p(v)dv pour qu’une particule ait un vecteur vitesse dont le module
est compris entre v et v + dv. En déduire la vitesse (module) la plus probable vp .
3◦ ) Calculer la valeur moyenne vm du module de la vitesse.

4◦ ) Calculer la vitesse quadratique moyenne v ? = < v 2 >.
5◦ ) Calculer la valeur moyenne < |vx | > de la valeur absolue de la composante vx . Exprimer
le résultat en fonction de vm .
Z +∞
2 1
Formulaire I(n) = tn e−at dt avec a > 0 ; on donne I(n + 1) = n I(n − 1),
r 0 2a
1 π 1
I0 = , I1 = .
2 a 2a

II - Ordre de grandeur du libre parcours moyen


Pour évaluer la distance moyenne parcourue par une molécule d’un gaz entre deux collisions,
c’est-à-dire, son libre parcours moyen `, on suppose que toutes les molécules du gaz sont
des sphères dures de rayon a. On note n la densité moléculaire. On donne la constante de
Boltzmann k = 1, 38 10−23 J K−1 .
1◦ ) Quelle est la section efficace de collision σ d’une molécule ?
2◦ ) Déterminer ` en supposant que le volume “balayé” par une molécule entre deux chocs
est de l’ordre de grandeur du volume moyen qui lui est offert.
3◦ ) Evaluer la vitesse moyenne v d’une molécule : la vitesse quadratique moyenne suffit pour
obtenir un ordre de grandeur. En déduire la fréquence moyenne de collision f = 1/τ , où τ est
la durée moyenne séparant deux collisions, en admettant que les molécules ont une vitesse
relative peu différente de leur vitesse moyenne.

Christian Carimalo 92 TD de Thermodynamique - L3


4◦ ) Application numérique. Evaluer ` et f pour une molécule d’hydrogène (a = 1, 45 Å),
dans les conditions T = 273 K, P = 105 Pa. Que deviennent ces valeurs pour i) T = 273 K,
P = 10−5 Pa ; ii) T = 1000 K , P = 105 Pa ?

III - Microfuite
Un gaz de densité particulaire n est en équilibre thermique de température T dans une
enceinte percée d’un petit trou de surface s. On admet que le nombre de molécules qui
s’échappent en une seconde dans le vide extérieur par cette ouverture est suffisamment faible
pour que l’équilibre dans l’enceinte ne soit pas totalement perturbé.
1◦ ) Calculer le nombre de molécules qui sortent du trou pendant une durée dt en ayant une
norme de vitesse initiale comprise entre v et v + dv.
2◦ ) Soit φ le flux de molécules sortantes, c’est-à-dire, le nombre de molécules qui sortent de
<v>
l’enceinte par unité de temps. Montrer que φ = n s
4

3 ) Calculer l’énergie cinétique totale Ec transportée par unité de temps par les molécules
qui s’échappent. Comparer l’énergie cinétique moyenne c transportée par une molécule qui
s’échappe à l’énergie cinétique moyenne d’une molécule dans l’enceinte.
4◦ ) a) Trouver l’évolution de la pression P dans un réservoir de volume V = 0, 02 m3 conte-
nant initialement de l’air à la pression P1 = 2, 5 105 Pa, communiquant par une ouverture
de section s avec l’atmosphère extérieure de pression P0 = 105 Pa, pour une température
uniforme de 300 K.
b) Application numérique. Calculer s sachant que P − P0 = (P1 − P0 )/100 au bout d’une
durée d’une heure. L’air sera assimilé à de l’azote pur de masse molaire 28 g. On rappelle la
constante des gaz parfaits R = 8, 31 J K−1 mole−1 .

IV - Deux enceintes identiques et contenant le même gaz sont maintenues à des températures
différentes T1 et T2 . Elles communiquent par un très petit trou. On suppose le régime sta-
tionnaire, c’est-à-dire, que le débit total de molécules traversant l’ouverture est nul et que
l’équilibre du gaz dans chacune des enceintes n’est pas perturbé. Déterminer le rapport des
densités moléculaires n1 /n2 ainsi que celui des pressions P1 /P2 du gaz dans les deux en-
ceintes.

V - Radiomètre de Crookes
Cet appareil est constitué d’une ampoule de verre, à gaz très raréfié, contenant un moulinet
très mobile à ailettes de mica noirci sur une de leurs faces. Lorsque l’appareil est éclairé, le
moulinet se met à tourner. On se propose d’étudier les mécanismes mettant le moulinet en
mouvement.

A/ Pression de radiation
On sait qu’à une onde électromagnétique monochromatique de fréquence ν se propageant
à la vitesse c dans le vide est associé un photon d’énergie hν et de quantité de mouvement
hν/c parallèle à la direction de propagation de l’onde, h étant la constante de Planck.
1◦ ) L’éclairement J de l’onde est l’énergie lumineuse transportée par l’onde par unité de
surface et par unité de temps. L’exprimer en fonction du flux de photons incidents, c’est-à-

Christian Carimalo 93 TD de Thermodynamique - L3


dire, le nombre de photons par unité de surface et par unité de temps.
2◦ ) On suppose que les photons arrivent perpendiculairement à la surface s des ailettes du
radiomètre.
a) Calculer la pression de radiation exercée par les photons sur la surface noircie, supposée
être parfaitement absorbante.
b) Calculer la pression de radiation sur l’autre face des ailettes, supposée, elle, parafaitement
réfléchissante.
c) En déduire l’existence d’une force susceptible de mettre le moulinet en mouvement.
Déterminer le sens et l’intensité de la force de radiation. Application numérique : J = 10 W
m−2 , s = 3 cm2 .

B/ En réalité, on observe un mouvement inverse à celui qui serait dû à la seule pression de
radiation. Cet effet a pour cause le gaz résiduel dans l’ampoule scellée et l’échauffement de
la face noircie des ailettes.
1◦ ) Calculer le libre parcours moyen des molécules du gaz résiduel à l’aide du modèle des
sphères dures de rayon 1 Å, pour une pression P = 0, 1 atm et une température T = 300 K.
2◦ ) Ce libre parcours moyen étant grand devant les dimensions de la cellule, les molécules
qui heurtent les ailettes viennent des parois maintenues à la température T . On simplifie la
réalité en considérant un modèle de jet moléculaire monocinétique de même concentration ρ
que le gaz résiduel et perpendiculaire à la paroi. Déterminer la vitesse des molécules du jet
pour que la pression sur la face réfléchissante des ailettes soit celle du gaz réel.
3◦ ) La face noircie des ailettes absorbe l’énergie de la lumière incidente et s’échauffe. En
régime stationnaire, elle est à une température constante supérieure à T . L’énergie lumineuse
reçue est alors intégralement cédée aux molécules incidentes lors de leurs chocs avec la
surface.
a) Calculer le nombre de molécules heurtant les ailettes par unité de temps et par unité de
surface.
b) On suppose qu’après réflexion, les molécules ont une vitesse v 0 . Exprimer la variation
d’énergie cinétique de l’ensemble des molécules heurtant une surface δS pendant la durée δt
en fonction de ρ, v et v 0 .
c) Calculer v 0 en faisant un bilan énergétique.
4◦ ) Calculer la pression sur la face absorbante. En déduire le sens et l’intensité de la force
qui s’exerce sur le moulinet. Application numérique : T = 300 K, M = 40 g.

VI - Compression isentropique d’un gaz parfait


On se propose de retrouver à partir de la théorie cinétique l’évolution d’un gaz parfait lors
d’une compression isentropique.
Un gaz parfait monoatomique constitué de N atomes de masse m est enfermé dans un
récipient fermé par un piston de surface S (figure 14). Le piston se déplace suivant la verticale
descendante à la vitesse u constante. Le déplacement étant supposé quasi-statique, le gaz
reste à l’équilibre (homogène et isotrope) et sa pression P reste sensiblement égale à la
pression extérieure Pext , laquelle, bien sûr, ne reste pas constante au cours de la compression.

Christian Carimalo 94 TD de Thermodynamique - L3


L’ensemble est isolé thermiquement.

Pext

u
P = Pext

Figure 14 – Compression isentropique d’un gaz parfait

1◦ ) On note dNv le nombre d’atomes du gaz ayant pour composantes de vitesse (vx , vy , vz )
à (dvx , dvy , dvz ) près. Sans qu’il soit nécessaire d’expliciter la fonction de distribution des
vitesses, définir la vitesse quadratique moyenne v ? et la moyenne < vz >+ de vz sur l’intervalle
0 ≤ vz ≤ +∞.
1◦ ) Déterminer la quantité de mouvement transférée au piston mobile par un atome de vitesse
initiale de composantes vx , vy , vz lorsque celle-ci effectue un choc élatique avec le piston.
2◦ ) Déterminer le nombre d’atomes de vitesse initiale vx , vy , vz à dvx , dvy , dvz près qui
heurtent le piston mobile pendant la durée δt. On note V le volume occupé par le gaz. On
remarquera que seuls les atomes tels que vz ≥ u peuvent rencontrer le piston et l’on admettra
que u  v ? et u < vz >+ .
3◦ ) a) En déduire que pendant toute la compression, la pression du gaz est donnée par
2U
P = où U est l’énergie interne du gaz à l’équilibre.
3V
b) En utilisant la forme différentielle du premier principe, trouver alors la relation entre U et
V caractérisant la compression isentropique.
4◦ ) Définir la température cinétique du gaz à l’équilibre. Comment varie-t-elle au cours de
cette compression ?

VII - Agitation thermique d’un petit miroir


Un miroir M vertical est solidaire d’un fil de torsion d’axe vertical ∆. Le dispositif est placé
dans une enceinte contenant un gaz à la température T . Les chocs incessants des particules
du gaz sur le miroir provoquent de faibles oscillations de celui-ci autour de ∆, oscillations
dont on repère l’amplitude par l’angle de rotation θ du miroir autour de ∆, par rapport à sa
position de repos. On note C la constante de torsion du fil, J le moment d’inertie du miroir
par rapport à ∆. L’énergie totale du miroir en rotation a pour expression

1 1
E = J θ̇2 + Cθ2
2 2

1◦ ) Expliquer pourquoi la probabilité pour que le miroir fasse un angle compris entre θ et
θ + dθ tandis que sa vitesse angulaire est comprise entre θ̇ et θ̇ + dθ̇ est

Christian Carimalo 95 TD de Thermodynamique - L3


E

P (θ, θ̇) = A e kT dθ dθ̇

où k est la constante de Boltzmann. Déterminer la constante A.


2◦ ) En déduire la probabilité P (θ) dθ pour que l’angle de rotation se trouve compris entre θ
et θ + dθ.
3◦ ) Calculer les valeurs moyennes < θ >, < θ2 >, puis la variance σ 2 =< θ2 > − < θ >2 .
Montrer que σ 2 = kT /C.
4◦ ) Pour T = 300 K, C = 10−15 SI, on mesure σ 2 = 4, 12 10−6 rd2 . En déduire la valeur
de la constante de Boltzmann, dont on donnera trois chiffres significatifs.
Z +∞ r
−ax2 π
On rappelle que e dx = où a > 0.
−∞ a

Christian Carimalo 96 TD de Thermodynamique - L3


Corrigé TD9

I - Vitesses moyennes
Z
1◦ ) La constante A est déterminée par la condition N = dNv . On a A−1 N = K 3 où K =
Z +∞ 3
m π m
r 
h i 2
exp −Bvx2 dvx , avec B = , soit K = . On obtient donc A = N
−∞ 2kT B 2πkT
2◦ ) Pour obtenir la probabilité p(v)dv, il faut intégrer la probabilité dNv /N sur ltoutes les
valeurs possibles des deux angles définissant la direction du vecteur vitesse. Comme dNv /N
A h i
est à symétrie sphérique, on trouve p(v)d(v) = 4πv 2 exp −Bv 2 . La vitesse la plus pro-
N
1
bableest celle qui rend p(v) maximum : vp = √ .
B
s
Z ∞
8kT
3◦ ) v m = v p(v)dv = .
0 πm

r
3kT
4◦ ) v? = < v2 >= .
m
Z ∞  Z ∞ −1
5◦ ) < |vx | >= |vx | exp[−Bvx2 ] dvx exp[−Bvx2 ] dvx .
Z ∞ −∞ Z ∞ −∞ Z ∞
Or, |vx | exp[−Bvx2 ] dvx = 2 u exp[−Bu2 ] du = u exp[−Bu0 ] du0 = 1/B, et
Z ∞ −∞ r 0 0
π 1 vm
exp[−Bvx2 ] dvx = . D’où < |vx | >= √ = .
−∞ B πB 2
xxxxxxxxxxxxxxxxxxxxxxxxxxxx

II - Ordre de grandeur du libre parcours moyen


1◦ ) σ = π(2a)2 (sphère de protection).
1
2◦ ) `nσ = 1, soit ` = .

3◦ ) La vitesse quadratique moyenne est obtenue en appliquant le théorème d’équipartition
mv 2 √
r
3kT 3kT
de l’énergie : < >= , soit v ? = < v 2 > = . Pour une évaluation, on peut
2 2 m
`
identifier la moyenne du module de la vitesse, vm , à v ? . A partir de là, on trouve τ = ,
vm
puis f = 1/τ .
P kT T P
4◦ ) On a n = , d’où ` = , et ` ∝ , f ∝ √ . Pour la molécule d’hydrogène (M = 2
kT σP P T
g/mole), on trouve
• ` = 1, 4 10−7 m , v ? = 1, 84 km/s, f = 1, 3 1010 chocs/s, pour T = 273K, P = 105 Pa ;
• ` = 1, 4 km, f = 1, 3 choc/s, pour T = 273K, P = 10−5 Pa ;
• ` = 5, 2 10−7 m , v ? = 3, 5 km/s, f = 6, 7 109 chocs/s, pour T = 1000 K, P = 105 Pa.

xxxxxxxxxxxxxxxxxxxxxxxxxxxx

Christian Carimalo 97 TD de Thermodynamique - L3


III - Microfuite
−→ −→
1◦ ) Soit dNs ( v ) le nombre de molécules ayant un vecteur vitesse v pointant vers le trou
et qui, à la date t, sont contenues dans le volume ABCD (voir figure) dont les arêtes AD
et BC ont pour longueur vdt, volume dont l’extension est s cos θvdt. Si l’on ignore l’effet
des chocs des molécules entre elles, ce qui est le cas si le libre parcours moyen est très

grand devant s, ces molécules vont toutes passer par le trou pendant la durée dt. Or, le
nombre total de molécules contenues le volume ABCD est ns cos θvdt, où n est la densité
−→
moléculaire de l’enceinte, et la proportion de ces molécules ayant le vecteur vitesse v est

C
vdt v

B D

z θ z’
s
A

−→ −→ −→
dNs ( v ) = ns cos θvdt dP ( v ) où dP ( v ) est la probabilité de trouver une molécule ayant
−→
un vecteur vitesse v à dvx , dvy , dvz près, avec la condition vz > 0. L’équilibre de l’enceinte
−→
n’étant pas perturbé par la fuite, on admet que dP ( v ) est donné par la loi de Maxwell-
3
m m

−→ −→ 2
Boltzman : dP ( v ) = A exp[−Bv 2 ] d3 v , avec B = ,A= . Utilisant les
2kT 2πkT
coordonnées sphériques (v, θ, ϕ), le nombre de molécules sortantes dNs cherché est calculé
comme suit :
Z π Z ∞ Z ∞
2
3 2
dNs = nsdt(2π)A cos θ sin θdθ v exp[−Bv ] dv = nsdtπA v 3 exp[−Bv 2 ] dv
0 0 0
Z Z ∞
2 3 −→
et comme A−1 = exp[−Bv ] d v = 4π v 2 exp[−Bv 2 ] dv, on obtient finale-
espace entier r 0
<v> 8kT
ment 6 dNs = nsdt (< v >= ).
4 πm
dNs <v>
2◦ ) φ = = ns .
dt 4
mv 2 −→ nms
3◦ ) dEc = dNs ( v ) et, par unité de temps, Ec = < v 3 >. Or,
2 8

1 d2
Z ∞ Z ∞ Z ∞
1 2 −Bu 1
5
v exp[−Bv ] dv = 2
u e du = e−Bu du =
0 2 0 2 dB 2 0 B3
Z ∞ −1 3
B

2
tandis que v 2 exp[−Bv 2 ] dv = 4πA = 4π
0 π
6. A noter que la formule qui suit peut être obtenue, sans expliciter l’expression de < v >, en supposant
−→
simplement que la densité de probabilité dP ( v )/[dvx dvy dvz ] est à sym’etrie sphérique.

Christian Carimalo 98 TD de Thermodynamique - L3


3
4π m 2kT 2

d’où < v3 >= 3 et Ec = ns π . On en déduit l’énergie cinétique moyenne
(πB) 2 2 πm
transportée par une molécule qui s’échappe : c = Ec /φ = 2kT (énergie de translation d’une
3kT
molécule à l’intérieur de l’enceinte : ).
2
4◦ ) a) La variation totale, pendant la durée dt, du nombre N de molécules contenues
dans l’enceinte est la différence entre le nombre dNe de molécules, venant de l’atmosphère
extérieure, qui entrent dans l’enceinte pendant la durée dt, et le nombre de molécules dNs
N P
qui sortent de l’enceinte pendant cette même durée : dN = dNe − dNs . On a n = = ,
V kT
P0
et la densité moléculaire de l’atmosphère reste constante et égale à n0 = . D’où, notant
kT
que < v > est le même à l’intérieur où à l’extérieur de l’enceinte (même température),
dN dNe dNs s<v> s<v> V dP
= − = [n0 − n] = [ P0 − P ] = , soit
dt dt dt 4 4kT kT dt
dP P − P0 4V
=− , avec χ = , et par intégration :
dt χ s<v>
P1 − P0
t = χ ln ou P − P0 = (P1 − P0 ) e−t/χ
P − P0

t1 4V
b) χ = , < v >= 476 m/s ; d’où s = 2 ln 10 = 0, 2 mm2 .
2 ln 10 t1 < v >

xxxxxxxxxxxxxxxxxxxxxxxxxxxx

IV -

Prolongation de l’exercice précédent : n1 < v1 >= n2 < v2 >, soit, dans le modèle du gaz
P1 P2
parfait, et compte tenu de l’égalité des volumes, √ = √ .
T1 T2

xxxxxxxxxxxxxxxxxxxxxxxxxxxx

V - Radiomètre de Crookes

A/ 1◦ ) J = N hν où N est le nombre de photons par unité de surface et par unité de temps.
hν J 2J J
2◦ ) a) Pabs. = N = . b) Préfl. = . c) F/s = Préfl. − Pabs. = , F = 10−11
c c c c
N. Cette force a tendance à faire reculer la face réfléchissante et avancer la face absorbante
devant le flux de photons (voir figure).

kT
B/ 1◦ ) ` = = 33 cm.

kT < v2 >
2◦ ) P = 2mρv 2 = ρkT , soit v 2 = = (v = 177 m/s).
2m 6
3◦ ) a) Le nombre de molécules heurtant les ailettes par unité de temps et par unité de surface
est N 0 = ρv.
0
" #
mv 2 mv 2
b) δEc = N 0 δtδS − .
2 2

Christian Carimalo 99 TD de Thermodynamique - L3


face noircie face r’efl’echissante

photons

h 0
i
c) δEc /[δtδS] = J, soit 2J = mρv v 2 − v 2 , doù v 0 .
 0 2
v 4J
4◦ ) Pabs. = mρv(v 0 + v), Préfl. = 2mρv 2 = P ; de = 1+ , on tire v 0 /v = 1, 8
v Pv
Pabs. 1 v0
 
(v 0= 319 m/s), et = 1+ = 1, 4. Dans ce mécanisme, la pression s’exerçant sur
Préfl. 2 v
une face absorbante (0, 14 Pa) est plus grande que celle s’exerçant sur une face réfléchissante.
Il en résulte une force F 0 = s(Psf abs. − P ) = 1, 2 10−5 N, de loin plus importante que celle
calculée précédemment et qui tend à faire tourner le moulinet dans le sens inverse de celui
indiqué dans la figure ci-dessus.

xxxxxxxxxxxxxxxxxxxxxxxxxxxx

VI - Compression isentropique d’un gaz parfait


dNv −→
1◦ ) dPv = est la probabilité de trouver un atome avec la vitesse v , à dvx , dvy , dvz
N Z
près. La vitesse quadratique moyenne v ? est définie par v ?2 = v 2 dPv , l’intégration triple
Z
étant effectué sur tout l’espace des vitesses (−∞ < vx < +∞, etc) ; < vz >+ = vz dPv .
vz ≥0

1◦ ) Soit R le référentiel lié au laboratoire, supposé galiléen, et R0 le référentiel lié au piston,


−→
lui aussi galiléen puisque u est constant. Si v est la vitesse d’un atome dans R, sa vitesse
−→ −→ −→ −→
dans R0 est v 0 = v −u ez où ez est le vecteur unitaire de la verticale descendante. Dans
R0 , le choc élastique d’un atome sur le piston provoque un changement de sa quantité de
−→ −→
mouvement égal à ∆ p0 = −2mvz0 ez , le choc ne se produisant, bien entendu, que si vz0 < 0
(l’atome doit s’y diriger dans une direction ascendante), soit vz < u ; R et R0 étant galiléens,
cette variation est aussi celle mesurée dans R. On note que dans R la vitesse de l’atome
−→ −→ −→
après le choc est vf = v −2(vz − u) ez .

2◦ ) On détermine aisément ce nombre δNv0 en se plaçant dans R0 : dNv0 = nS|vz0 |δt dPv ,
N
avec n = .
V
3◦ ) a) Dans R, la variation d’énergie cinétique d’un atome ayant effectué un choc avec le
mh 2 2
i
piston est ∆Ec = v − viz = 2mu(u − vz ). L’énergie interne du gaz parfait est la
2 fz
somme des énergies cinétiques de ses constituants, et sa variation pendant la durée δt est
Z Z
δU = ∆Ec δNv0 = 2m n u S δt (u − vz )2 dPv
vz <u vz <u

Christian Carimalo 100 TD de Thermodynamique - L3


−→
Comme on sait, le transfert de quantité de mouvement +2mvz0 ez vers le piston s’interprète
en terme de pression. Cette pression est donnée par
Z Z
P = 2m|vz0 | nS|vz0 | dPv = 2m n (u − vz )2 dPv
vz <u vz <u

Rapprochant cette formule de celle de δU , et notant que la variation du volume du gaz


pendant δt est −Suδt (le volume diminue dans la compression), on trouve la relation
δU = −P δV , qui est tout à fait naturelle, s’agissant d’une transformation adiabatique. D’un
autre côté, on suppose que u  v ? et u < vz >+ , ce qui fait de cette transformation une
transformation quasi-statique (et réversible).Z En faisant dans la formule précédente, l’approxi-
mation (u − vz )2 ' vz2 , il vient P ' 2m n vz2 dPv . Admettant alors que la distribution
vz <0 Z Z
des vitesses possède la symétrie sphérique, on peut écrire 2 vz2 dPv = vz2 dPv +
vz <0 vz <0
1 2 m 2
Z
vz2 dPv =< vz2 >= < v 2 >, d’où, à la limite u → 0, P = N < v >. Or, à
vz >0 3 3V 2
m 2
l’équilibre, on a U = N < v > et par conséquent, lors de cette transformation adiaba-
2
2U
tique réversible (donc isentropique), on peut écrire P = où U est l’énergie interne du
3V
gaz à l’équilibre.
2 δV δU 2 δV
b) De la relation δU = −P δV = − U on tire + = 0, dont l’intégration donne
2
3 V U 3 V
U V 3 = constante1 .
4◦ ) Le théorème d’équipartition de l’énergie cinétique donne pour ce gaz parfait monoato-
3RT
mique U = , relation qui peut être considérée comme une définition de la température
2
thermodynamique T . Lors de la transformation isentropique, T et V sont donc liés par
2
T V 3 = constante2 . Dans cette compression, T augmente, ce qui signifie que l’agitation
thermique au sein du gaz augmente.

xxxxxxxxxxxxxxxxxxxxxxxxxxxx

VII - Agitation thermique d’un petit miroir



JC
1◦ ) Rédaction laissée au lecteur ; A = .
2πkT
" #
Cθ2
r
C
2◦ ) P (θ) dθ = exp − dθ.
2πkT 2kT
Z +∞
d C 1 kT

−aθ2
3◦ ) < θ >= 0, < θ2 >= − ln e , soit < θ2 >=
dθ où a = = .
da −∞ 2kT 2a C
Cθ2 kT
On retrouve ici le théorème d’équipartition de l’énergie : < >= . La variance est
2 2
kT
σ 2 ≡< θ2 >= .
C
Cσ 2 4, 12 −23
4◦ ) k = = 10 SI = 1, 37 10−23 SI.
T 3
xxxxxxxxxxxxxxxxxxxxxxxxxxxx

Christian Carimalo 101 TD de Thermodynamique - L3


TD10

- Mélanges -

I - Mélange de gaz parfaits


A/ On considère un mélange de q gaz n’ayant pas d’interaction chimique entre eux. L’état du
mélange est défini par la température T , la pression P et les nombres de moles ni , i = 1, ..., q
des divers constituants du mélange. On note G l’enthalpie libre du mélange.
1◦ ) Montrer que G s’exprime uniquement en fonction des ni et des potentiels chimiques µi
des gaz dans le mélange.
q
X
2◦ ) On note xi = ni /n la concentration du constituant i dans le mélange, où n = ni ,
i=1
nombre total de moles de gaz du mélange.
a) Les variables xi sont-elles intensives ou extensives ?
b) En utilisant les notions de variables intensives ou extensives, montrer que le potentiel
chimique µi n’est fonction que de T, P et des concentrations xi . De combien de variables
intensives indépendantes µi est-il fonction ?
3◦ ) Etablir les relations de Gibbs-Duhem sous leur forme explicite. Préciser leur nombre.
B/ Les constituants du mélange sont des gaz parfaits.
1◦ ) Rappeler sans démonstration l’expression du potentiel chimique µi en fonction de l’en-
thalpie libre molaire gi du constituant i à l’état pur, à la température T et à la pression P ,
et de xi . De quelles variables dépend gi ?
2◦ ) Montrer que les relations de Gibbs-Duhem sont soit automatiquement satisfaites, soit
permettent de calculer le volume V et l’entropie S du mélange.
3◦ ) On note respectivement vi et si le volume molaire et l’entropie molaire du gaz i pur à
la température T et à la pression P .
a) Expliciter V en fonction de n, vi et xi . Comparer le résultat obtenu à l’expression du
volume V0 occupé par les gaz avant mélange, chacun d’entre eux étant alors dans un état
défini par T et P .
b) Expliciter S en fonction de n, si et xi . Comparer S à l’expression de l’entropie S0 des q
gaz avant mélange. Commenter.

II - Formation de brouillard
A/ On considère l’air atmosphérique comme un mélange d’air sec et de vapeur d’eau. Son
mv
degré d’humidité est défini par le rapport Π = de la masse de vapeur d’eau et de la
ma
masse d’air sec qui y sont mélangés. L’air sec ainsi que la vapeur d’eau, saturée ou non,

Christian Carimalo 102 TD de Thermodynamique - L3


seront considérés comme des gaz parfaits de masses molaires Ma et Mv respectivement et
leur mélange comme un mélange idéal de gaz parfaits à la pression P0 .
On tiendra compte du fait qu’en toute circonstance la masse de vapeur d’eau reste toujours
très inférieure à celle de l’air sec, et l’on effectuera les approximations qui en découlent
naturellement : Π  1.
1◦ ) a) On note respectivement Pv et Pa les pressions partielles de la vapeur d’eau et de l’air
sec. Rappeler leurs définitions.
b) Etablir la relation existant entre P0 , Pa et Pv .
P v Mv
c) Montrer que Π = .
P0 M a
2◦ ) Deux masses d’air humide caractérisées par leurs températures T1 et T2 , leurs masses
d’air sec ma1 et ma2 , leurs degrés d’humidité Π1 et Π2 , leurs pressions égales à P0 , sont
mélangées adiabatiquement sous la pression P0 sans qu’il y ait condensation de la va-
peur d’eau. La chaleur massique à pression constante de l’air humide Cp est supposée
indépendante de la température et du degré d’humidité (Π  1). Quels sont, en fonction de
T1 , T2 , ma1 , ma2 , Π1 , Π2 :
a) le degré d’humidité final du mélange ;
b) la variation d’enthalpie du système des deux masses d’air (enthalpie de mélange) ;
c) la température finale du mélange ?

B/ Formation de brouillard
3◦ ) On considère maintenant une masse d’air à la température Celsius t ◦ C contenant de la
vapeur d’eau saturante. Dans le tableau ci-dessous, on donne les valeurs, exprimées en mm
de mercure, de la pression de vapeur saturante Ps de la vapeur d’eau pour diverses valeurs
de t.

t ◦ C 0 5 10 15 20 25 30

Ps mm Hg 4,6 6,6 9,2 12,8 17,4 23,8 31,8

a) Tracer sur papier millimétré le graphe du degré d’humidité saturante Πs de l’air en fonction
de t en utilisant le résultat de la question 1◦ ) c), sachant que P0 = 760 mm Hg, Ma = 29 g,
Mv = 18 g.
b) Ce graphe sépare le plan t, Π en deux régions. Dire à quelle phase vapeur ou liquide de
l’eau correspond chacune des deux régions.
4◦ ) On considère maintenant un système Σ isolé thermiquement et constitué initialement
de deux masses d’air saturées d’humidité, de même masse ma d’air sec et de températures
respectives T1 et T2 . Leurs degrés d’humidité respectifs sont Π1 (T1 ) et Π2 (T2 ). Ces deux
masses d’air se mélangent sous la pression P0 . On constate alors une condensation (ou
liquéfaction) d’une partie de la vapeur d’eau sous forme de gouttelettes d’eau liquide en
suspension dans l’atmosphère, c’est-à-dire, la formation d’un brouillard.

Christian Carimalo 103 TD de Thermodynamique - L3


L’état final du système Σ est donc un équilibre à la température finale T entre, d’une part, un
air humide de masse d’air sec égale à 2ma et contenant de la vapeur d’eau saturante avec un
degré d’humidité Πs (T ), et, d’autre part, des gouttelettes d’eau liquide en suspension dans
l’atmosphère. Le but de cette question est de déterminer dans le diagramme de la question
3◦ ) a) les coordonnées du point représentatif de l’air humide à l’état final.
On note L la chaleur latente massique de vaporisation de l’eau, que l’on supposera constante,
et m la masse de vapeur d’eau qui s’est condensée lors du mélange adiabatique des deux
masses d’air humide.
a) Exprimer m en fonction de ma , Πs (T1 ), Πs (T2 ) et Πs (T ).
b) Quelle quantité de chaleur Q est mise en jeu lors de la condensation ?
c) Exprimer la variation d’enthalpie de l’air humide en fonction de ma , Cp , T, T1 et T2 .
d) déduire alors l’expression de la variation d’enthalpie ∆H du système Σ. Ecrire l’équation
permettant d’obtenir la température finale T de Σ.
e) Montrer alors que l’on a

Πs (T1 ) + Πs (T2 ) Cp T1 + T2
 
Πs (T ) = + −T
2 L 2

f) Résoudre graphiquement l’équation précédente, c’est-à-dire, donner la valeur numérique


en degré Celsius de la température d’équilibre air humide-liquide.
On donne : T1 = 10◦ C, T2 = 25◦ C, L = 575 cal g−1 , Cp = 0, 25 cal K−1 g−1 .

III - Phases comportant plusieurs constituants


Un système fermé est constitué de deux phases (a) et (b) en équilibre, à la température T ,
sous la pression P . Chaque phase contient deux constituants (1) et (2) qui sont chimiquement
stables. On note respectivement na1 , na2 , µa1 , µa2 les nombres de moles et potentiels chimiques
de (1) et (2) dans la phase (a), nb1 , nb2 , µb1 , µb2 les nombres de moles et potentiels chimiques
des deux constituants dans la phase (b).
1◦ ) Montrer que µa1 = µb1 , µa2 = µb2 .
2◦ ) On considère une solution d’eau salée en contact avec du sel solide et de la vapeur d’eau.
Soit x la fraction molaire de l’eau dans la solution.
a) De quelles variables dépendent les potentiels chimiques impliqués ?
b) Ecrire les équations sur les potentiels chimiques qui expriment l’équilibre du système.
!
∂µ`1
c) Rappeler la relation implicite de Gibbs-Duhem. Etablir la relation entre x, et
∂x T
!
∂µ`2
.
∂x T

3◦ ) Déduire deux équations reliant dT, dP, dx. Ces équations font intervenir les entropies et
volumes molaires des constituants dans le mélange s`1 , sv1 , s`2 , ss2 , v ` , v1v , v2` , v2s , ainsi que les
dérivées par rapport à x des potentiels chimiques de l’eau et du sel dans la solution.

Christian Carimalo 104 TD de Thermodynamique - L3


4◦ ) En utilisant les résultats 2◦ ) c), trouver la relation différentielle qui existe entre les
variations dT et dP du système. Seuls interviennent les entropies et volumes molaires des
constituants dans le mélange et x.

IV - Coefficients d’activité
On considère une solution non idéale de deux liquides A et B à la température T et à la
pression P , n’interagissant pas chimiquement. On note na et nb le nombre de moles de cha-
cun des liquides, xa et xb leurs concentrations molaires, µa et µb les potentiels chimiques
correspondants. A l’état pur, leurs enthalpies libres molaires sont ga (T, P ) et gb (T, P ) res-
pectivement. On introduit les coefficients d’activité γa et γb de chacun des liquides dans la
solution, définis par :

µa = ga (T, P ) + RT ln (γa xa ) , µb = gb (T, P ) + RT ln (γb xb ) (1)

1◦ ) Quelle est la variance du mélange ?


2◦ ) Montrer qu’à température et pression constantes on a

∂µa ∂µb
   
xa + xb =0
∂xa P,T ∂xa P,T

3◦ ) A l’aide des relations (1), montrer alors que

∂ ln γa ∂ ln γb
   
xa = xb (2)
∂xb P,T ∂xa P,T

4◦ ) On admet que l’enthalpie libre de la solution est de la forme

G = na ga (T, P ) + nb gb (T, P ) + RT (na ln xa + nb ln xb ) + A (na + nb ) x2a xb

a) Calculer µa et µb .
b) Expliciter γa et γb en fonction de A, xa , xb , R et T .
c) Vérifier que la relation (2) est bien satisfaite.

Christian Carimalo 105 TD de Thermodynamique - L3


Corrigé TD10

I - Mélange de gaz parfaits


X
A/ 1◦ ) L’identité thermodynamique dU = −P dV + T dS + µi dni définit les potentiels
i
chimiques µi . L’enthalpie libre G est obtenue à partir deX
U par la transformation de Legendre
G = U + P V − T S, et l’on a dG = V dP − SdT + µi dni , ce qui permet de redéfinir
i
∂G
 
les potentiels chimiques comme µi = ; G, ainsi que les ni sont des grandeurs
∂ni T,P
extensives, tandis que T , P et les µi sont des grandeurs intensives. Dans le changement
infinitésimal de taille du système : ni → n01 = ni (1 + dλ), G est changé en G0 =XG(1 + dλ),
tandis que T , P et les µi sont inchangés. Or, G0 − G = dG = V dP − SdT + µi dni ≡
X i
µi dni puisque dT = 0, dP = 0, et comme dG = G dλ, dni = ni dλ, il vient, après
i X
simplifications par dλ : G = µ i ni .
i

2◦ ) a) Intensives ! !
b) µi étant intensif ne dépend que des variables intensives T , P , et de combinaisons intensives
des ni . On peut donc dire que µi n’est fonction que de T, P et des concentrations xi .
q
X
Ces dernières ne sont pas indépendantes puisque xi = 1. Il y a donc seulement q − 1
i=1
concentrations indépendantes et au total, chaque µi dépend de q + 1 variables (en rajoutant
T et P ).
" #
X X X X
3◦ ) On a dG = V dP − SdT + µi dni = d µi n i = µi dni + ni dµi . Ecrivant
i i i i
q−1 !
∂µi ∂µi ∂µi
    X
ensuite dµi = dT + dP + dxj , puis identifiant les
∂T P,x ∂P T,x j=1
∂xj T,P,xk6=j
coefficients des différentielles, on trouve les q + 1 relations

q q !
∂µi ∂µi ∂µi
X     X
V = ni , S=− , 0= xi (j = 1, · · · , q − 1)
i=1
∂P T,x ∂T P,x i=1
∂xj T,P,xk6=j

RT
B/ 1◦ ) µi (T, P, [x]) = gi (T, P ) + RT ln xi , dgi (T, P ) = dP − si dT .
P
2◦ ) Choisissons d’exprimer xq comme fonction des autres concentrations,
! soit xq = 1 −
∂µi RT
x1 − · · · − xq−1 . On a alors, pour i, j = 1, · · · , q − 1 : = δij , et
∂xj T,P,x xi
! k6=j

∂µq RT
=− . On a ainsi (on suppose xi 6= 0)
∂xj T,P,xk6=j
xq

Christian Carimalo 106 TD de Thermodynamique - L3


q ! q−1
X ∂µi X RT RT
xi = xi δij − xq = 0
i=1
∂xj T,P,xk6=j i=1
xi xq

∂µi ∂gi RT ∂µi ∂gi


       
Puis, avec = = , = + R ln xi = −si (T, P ) +
∂P T,x ∂P T P ∂T P,x ∂T P
R ln xi , on obtient
q q
∂µi RT nRT
X   X
ni = ni = =V
i=1
∂P T,x i=1
P P

q q
∂µi
X   X
ni = ni [−si (T, P ) + R ln xi ] = −S(T, P, [ni ])
i=1
∂T P,x i=1

q
ni RT X
3◦ ) a) vi = ,V = vi = V0 ;
P i=1
q
X q
X
b) S(T, P, [ni ]) = ni si (T, P ) − ni R ln xi = S0 (T, P, [ni ]) + Smélange où Smélange =
i=1 i=1
q
X
− ni R ln xi > 0. L’entropie de mélage est positive, ce qui est naturel puisqu’on passe
i=1
d’une configuration ordonnée où les gaz sont séparés à une configuration désordonnée où les
gaz sont mélangés.

xxxxxxxxxxxxxxxxxxxxxxxxxxxx

II - Formation de brouillard

RT RT
A/ 1◦ ) a) Pv = nv , Pa = na .
V V
RT Pv nv Pa na
b) P0 = Pv + Pa = (nv + na ) , = , = .
V P0 na + nv P0 na + nv
mv ma Pv mv mv Ma
b) Ecrivant nv = , na = , on a = ' .
Mv Ma P0 mv + ma Mv /Ma ma Mv
mv P v Mv
D’où Π = = .
ma P0 Ma
mvf ma1 Π1 + ma2 Π2
2◦ ) a) Mvf = mv1 + mv2 , maf = ma1 + ma2 , Πf = = .
maf ma1 + ma2
b) La pression extérieure reste constante, tandis que la transformation est adiabatique. La
variation d’enthalpie du système des deux masses d’air (enthalpie de mélange) est donc nulle.
c) On peut négliger l’enthalpie de la vapeur devant celle de l’air sec. L’équation ∆Ha = 0
ma1 T1 + ma2 T2
conduit alors à Cp [(ma1 + ma2 )Tf − mai T1 − ma2 T2 ] = 0, soit Tf = .
ma1 + ma2

18 Ps
B/ 3◦ ) a) b) Avec Π = , on trouve les valeurs numériques listées dans le tableau
29 P0
ci-dessous

Christian Carimalo 107 TD de Thermodynamique - L3


t ◦ C 0 5 10 15 20 25 30

Π × 103 3,75 5,4 7,5 10,4 14,2 19,4 26

et la courbe (C) de la figure (15)

Figure 15 – Courbe Π(t)

4◦ ) a) m = ma [Πs (T1 ) + Πs (T2 ) − 2Πs (T )].

b) Q = −mL.

c) ∆Hair humide = ma Cp [2T − T1 − T2 ].

d) ∆Htot = 0 = −mL + ma Cp [2T − T1 − T2 ].


m Cp
e) = [2T − T1 − T2 ] = Π1 + Π2 − 2Πs , d’où
ma L
Πs (T1 ) + Πs (T2 ) Cp T1 + T2
 
Πs (T ) = + −T
2 L 2

f) Dans le plan (Π, t), la relation du e) donne la droite (D), dont l’intersection avec la courbe
(C) donne la température d’équilibre air humide-liquide.

Christian Carimalo 108 TD de Thermodynamique - L3


17, 5 − t
Equation de (D) : 103 Π = 13, 5 + . On trouve graphiquement la température
2, 3
d’équilibre t ' 18, 5 ◦ C.

xxxxxxxxxxxxxxxxxxxxxxxxxxxx

III - Phases comportant plusieurs constituants

1◦ ) L’équilibre du système à température et pression données


X se définit par un minimum de
la fonction d’état enthalpie libre G = U − T S + P V = nϕ ϕ
c µc . Or, à T et P fixés, la
c,ϕ
X
différentielle de G est dG = µϕ ϕ
c dnc et cette différentielle doit donc être nulle au point
c,ϕ
d’équilibre. Pour le système considéré ici, n1 = na1 + nb1 et n2 = na2 + nb2 sont donnés et
on peut considérer que, par exemple, n a a
h i 1 et n2h sont lesiseules variables indépendantes. On
écrira ainsi dG comme dG = µa1 − µb1 dna1 + µa2 − µb2 dna2 = 0, équation qui conduit aux
relations cherchées : µa1 = µb1 , µa2 = µb2 .
2◦ ) a) Il y a ici 2 constituants, l’eau et le sel, et 3 phases : la solution, la vapeur et le solide.
La variance est donc v = c+2−ϕ = 2+2−3 = 1 : le système est univariant. On peut choisir
n`
l’unique variable comme étant la fraction molaire x de l’eau dans la solution, x = ` eau ` .
neau + nsel
Les potentiels chimiques dépendent donc a priori des trois variables T , P et x. Cependant,
le sel étant l’unique constituant de la phase solide et l’eau l’unique constituant de la phase
vapeur, les potentiels chimiques µssel et µveau ne sont en fait fonctions que de T et P .
b) Considérons un X X de l’équilibre ([T, P, x] → [T + dT, PX
déplacement infinitésimal + dP, x +
dx]). Comme G = nϕ µ
c c
ϕ
, on a dG = [ µ ϕ
c dn ϕ
c + n ϕ
c dµ ϕ
c ] = V dP −SdT + µϕ ϕ
c dnc ,
c,ϕ c,ϕ c,ϕ
X
et par conséquent, nϕ ϕ
c dµc = V dP − SdT .
c,ϕ

c) Le membre
 ϕ  de droite de la relation du b) ne fait pas intervenir dx. On en déduit que
X
ϕ ∂µc
nc = 0. D’où, en tenant compte du fait que µssel et µveau ne dépendent pas
c,ϕ ∂x T,P
! !
∂µ`eau ∂µ`sel
de x : n`eau + n`sel = 0, soit encore,
∂x T,P
∂x T,P
! !
∂µ`eau ∂µ`sel
x + (1 − x) =0
∂x T,P
∂x T,P

3◦ ) Dans ce déplacement d’équilibre, on doit avoir dµveau = dµ`eau et dµssel = dµ`sel . En


détaillant, on trouve
!
∂µ`eau
dµveau = v
veau dP − sveau dT = dµ`eau = `
veau − s`eau + dx
∂x T,P
!
∂µ`sel
dµssel = vsel
s
dP − sssel dT = dµ`sel = vsel
`
− s`sel + dx
∂x T,P

1−x
4◦ ) Posons r = . En utilisant le résultat du 2◦ ) c), on déduit aisément la relation :
x

Christian Carimalo 109 TD de Thermodynamique - L3


h i
dP s`eau − sveau + r s`sel − sssel
= ` v + r v` − vs
 
dT veau − veau sel sel
On retrouve la formule de Clapeyron valable pour un corps pur dans les deux cas r = 0
(x = 1) et r = ∞ (x = 0).

xxxxxxxxxxxxxxxxxxxxxxxxxxxx

IV - Coefficients d’activité

1◦ ) c = 2, ϕ = 1, donc v = 3 (trois variables indépendantes, par exemple, T , P et xa ).


∂µϕ ∂µa ∂µb
     
c
X X
2◦ ) nϕ ϕ
c dµc = V dP −SdT , donc nϕ
c = 0 = na +nb ,
c,ϕ c,ϕ ∂xa T,P ∂xa T,P ∂xa T,P
soit
∂µa ∂µb
   
xa + xb =0
∂xa T,P ∂xa T,P

∂ ∂
3◦ ) On utilise le fait que ≡− . Le calcul ne pose aucune difficulté :
∂xa ∂xb
∂ ln γa ∂ ln γb
   
xa = xb
∂xb T,P ∂xa T,P

na nb nb n2a
 
4◦ ) G = na ga (T, P ) + nb gb (T, P ) + RT na ln + nb ln +A
na + nb na + nb (na + nb )2
a) µa = ga + RT ln xa + 2Axa x2b , µb = gb + RT ln xb + Ax2a (xa − xb ) ;
A A 2
b) ln γa = 2 xa x2b , ln γb = x (xa − xb ) ;
RT RT a
∂ ∂ 2
c) xa [2xa x2b ] = 2xa [x (1 − xb )] = 2xa xb (2 − 3xb )
∂xb ∂xb b
∂ ∂
xb [x2a (xa − xb )] = xb [x2 (2xa − 1)] = 2xa xb (3xa − 1) = 2xa xb (2 − 3xb )
∂xa ∂xa a
La relation du 3◦ ) est bien satisfaite.

xxxxxxxxxxxxxxxxxxxxxxxxxxxx

Christian Carimalo 110 TD de Thermodynamique - L3


TD11

- Equilibres de phases, propriétés colligatives -

I - Mélange liquide-vapeur, loi de Raoult


Un corps pur A liquide est en équilibre avec sa vapeur à la température T et sous la pression
P . Soient gv et g` les enthalpies libres molaires respectives des phases vapeur et liquide de
A.
1◦ ) Rappeler la condition d’équilibre des deux phases liquide et vapeur de A.

On introduit dans la phase liquide un corps B non volatil tel que la solution obtenue soit
idéale et non diluée. On note xA et xB les titres molaires respectifs de A et B dans cette
solution.
2◦ ) Ecrire la condition d’équilibre de la solution avec la phase vapeur.
3◦ ) A température constante T , la pression d’équilibre de la solution avec la phase vapeur A
subit une petite variation dP lorsque le titre de B varie de dxB . Exprimer la différence vv − v`
des volumes molaires de la vapeur A et du liquide A en fonction de la dérivée par rapport à
P d’une fonction de xB , T et R, constante des gaz parfaits.
4◦ ) En déduire que l’introduction du corps B dans la solution (son titre passant de 0 à xB )
entraı̂ne une variation ∆P de la pression d’équilibre, telle que

∆P v`
 
ln 1 + = ln(1 − xB ) + ∆P
P RT

On supposera que la vapeur est un gaz parfait et que v` reste constant. Que devient ∆P/P
lorsque xB  1 : la solution est alors diluée (loi de Raoult) ? On justifiera les approximations
faites et on vérifiera que ∆P/P est bien petit devant 1.
5◦ ) A pression constante P, la température d’équilibre de la solution avec la phase vapeur
A subit une petite variation dT lorsque le titre de B varie de dxB . Exprimer la différence
h` − hv des enthalpies molaires du liquide A et de la vapeur A en fonction de la dérivée par
rapport à T d’une fonction de xB , T et R.
6◦ ) En déduire que l’introduction du corps B dans la solution entraı̂ne une variation ∆T de
la température d’équilibre, telle que

∆T R
 =− ln(1 − xB )
∆T

`v
T2 1+
T

où `v est la chaleur latente molaire de vaporisation de A, que l’on suppose constante. Quelle
est la variation de temérature ∆T lorsque la solution est diluée ?

Christian Carimalo 111 TD de Thermodynamique - L3


∆P
7◦ ) Application numérique. Calculer et ∆T dans le cas où la solution est obtenue en
P
introduisant 1/10 mole de soluté dans un litre d’eau. On donne : la chaleur latente massique
de l’eau 540 cal g−1 ; la température d’ébullition de l’eau 373 K.

II - Cryoscopie

Un liquide contenant en solution des corps A et B est en équilibre à la température T sous


la pression atmosphérique P0 avec un solide constitué du corps A pur. La solution liquide
est idéale et diluée. On note TA et `A la température de fusion et la chaleur latente molaire
de fusion de A pur à la pression P0 respectivement, xA et xB les fractions molaires dans la
solution.

1◦ ) Ecrire la condition d’équilibre à la pression P0 entre le liquide A pur et le solide A pur.

2◦ ) Faire de même pour l’équilibre entre la solution liquide A+B et le solide A pur à P0 .

3◦ ) En utilisant le fait que la solution est diluée, et que par conséquent la température
d’équilibre T est voisine de TA , établir une expression approchée de ∆T = T − TA en
fonction de TA , `A et xB .

4◦ ) Application

a) Sachant que le radiateur d’une automobile contient 12 ` d’eau, de combien abaissera-t-on


le point de congélation en additionnant 5 kg de prestone (glycol C2 H4 (OH)2 ) ?

b) Combien de kg de zérone (alcool méthylique CH3 OH) devrait-on utiliser pour parvenir
au même résultat ? On suppose que les produits sont purs à 100%.

La chaleur latente de fusion de la glace est de 79,6 cal g−1 .

III - Solution diluée en équilibre avec sa vapeur, loi de Henry

Une solution idéale diluée de deux liquides est en équilibre à la température T et sous la
pression P avec sa phase vapeur considérée comme un mélange idéal de gaz parfaits. Les
constituants 1 et 2 sont respectivement le solvant et le soluté. On note P10 la pression
de vapeur saturante du solvant pur à la température T ; x`1 et x`2 sont les fractions molaires
respectives du solvant et du soluté dans la solution, µ`1 et µ`2 les potentiels chimiques respectifs
du solvant et du soluté dans la solution. Les mêmes grandeurs dans la phase vapeur sont
notées xv1 , xv2 , µv1 , µv2 respectivement.

1◦ ) a) Rappeler la condition d’équilibre du solvant en solution avec sa phase vapeur, et


l’exprimer en fonction des enthalpies libres molaires du solvant pur en phase liquide et en
phase vapeur et des fractions molaires correspondantes.

b) En considérant que le volume molaire v1` du solvant en phase liquide est petit devant son
volume molaire v1v en phase vapeur, montrer que la loi de Raoult donnant à température
fixée la pression partielle de vapeur P1v du solvant en fonction de x`1 s’écrit

P1v = P10 x`1

2◦ ) A partir de la relation différentielle de Gibbs-Duhem pour cette solution, montrer que


x`1 dµ`1 + x`2 dµ`2 = 0.

Christian Carimalo 112 TD de Thermodynamique - L3


3◦ ) a) Récrire le potentiel chimique du solvant dans la phase vapeur en fonction de T, P et
P1v puis en fonction de T, P et x`1 .
b) Récrire le potentiel chimique du soluté dans la phase vapeur en fonction de T, P et P2v ,
pression partielle du soluté.
c) Différentier à T et P fixés les relations d’équilibre du solvant et du soluté.
d) En utilisant alors la loi de Raoult et la relation établie au 2◦ ), montrer que P2v est donnée
par la loi de Henry :

P2v = K2 x`2

où K2 est une constante appelée constante de Henry.


4◦ ) Application. A la profondeur de 100 m dans les océans la mesure de la concentration
de O2 est d’environ 0, 25 10−3 mol/litre. On veut comparer cette valeur avec celle prédite
par la loi de Henry, en supposant l’équilibre entre oxygène atmosphérique et oxygène dissous,
considéré comme soluté. Sachant que la pression partielle d’oxygène dans l’atmosphère est
d’environ 0,2 atm. et que K2 ' 4 104 atm., calculer la concentration d’oxygène dissous
à partir de la loi de Henry. Comparer ce résultat avec la mesure de cette concentration.
Conclusion.

IV - Ebullioscopie
On note u` (T, P ), h` (T, P ), s` (T, P ) respectivement, l’énergie interne, l’enthalpie et l’entro-
pie d’une mole d’eau liquide à la température T et sous la pression P . Dans les mêmes
conditions, on note uv (T, P ), hv (T, P ), sv (T, P ), respectivement, les mêmes fonctions pour
la vapeur d’eau.
1◦ ) Définir les potentiels thermodynamiques f , énergie libre molaire, et g, enthalpie libre
molaire, et donner leurs expressions pour l’eau liquide et la vapeur d’eau en fonction des
seules grandeurs introduites plus haut.
2◦ ) a) Montrer que si la pression est fixée, l’eau liquide ne peut être en équilibre avec sa
vapeur qu’à une température donnée.
b) En la justifiant soigneusement, écrire l’équation traduisant l’équilibre de l’eau liquide avec
sa vapeur.
3◦ ) On dissout nA moles d’un corps pur A non volatil dans N moles d’eau liquide, avec la
na
fraction molaire xA '  1. En admettant que la solution obtenue soit idéale, donner
N
l’expression du potentiel chimique de l’eau dans la solution.
4◦ ) Soit T0 la température d’ébullition de l’eau pure sous la pression atmosphérique normale
P0 .
a) Calculer la température d’ébullition Te de la solution idéale définie en 3◦ ), sous la pression
P0 . Justifier soigneusement le calcul et les hypothèse faites.
b) Exprimer ∆T = Te − T0 en fonction i) des seules grandeurs introduites jusqu’ici dans le
problème ii) de la chaleur latente molaire d’ébullition ` de l’eau pure. Quel est le signe de
∆T ?

Christian Carimalo 113 TD de Thermodynamique - L3


5◦ ) Montrer que la mesure de ∆T correspondant à une masse m du corps A dissout dans
une masse M d’eau permet de déterminer la masse molaire MA de A.
6◦ ) Application numérique. A la température T0 sous la pression P0 , on a ` = 40550 SI,
∆T = 0, 02 K. On donne m = 0, 01 kg, M = 1 kg. Calculer MA .

V - Osmose
A/ On considère une enceinte à paroi indéformable formée de deux compartiments (1) et
(2) séparés par une membrane M perméable uniquement à l’eau, doublée d’une cloison C
strictement imperméable à tout liquide.
On introduit dans le premier compartiment une solution aqueuse comportant na moles d’un
soluté A dissout dans n1 moles d’eau et, dans le second compartiment une autre solution
aqueuse comportant nb moles d’un soluté B dissout dans n2 moles d’eau.
Les pressions dans les deux compartiments seront supposées maintenues identiques et égales
constamment à P0 . L’ensemble est à température constante T0 .
Les solutions sont supposées idéales. Les concentrations molaires respectives xa et xb des
solutés A et B sont faibles devant les concentrations de l’eau, x1 et x2 .
1◦ ) On retire la cloison C. Décrire le phénomène qui s’ensuit, selon les valeurs des concen-
trations initiales de A et B.
2◦ ) Calculer, à l’équilibre, les nouvelles concentrations molaires n01 et n02 de l’eau dans chacun
des compartiments, en fonction de ne = n1 + n2 , na et nb .

B/ Dans cette partie, les pressions dans les deux compartiments sont supposées différentes
et fixées à P1 dans le premier compartiment et à P2 dans le second. On impose P1 > P2 .
La température est la même dans les deux compartiments et égale à T0 . On admettra que
dans le domaine des pressions considéré, le volume molaire de l’eau liquide est constant et
égal à v.
Les deux solutions aqueuses précédentes étant présentes dans leurs compartiments respectifs,
on enlève la cloison C.
1◦ ) Ecrire en termes de potentiels chimiques la relation que devraient vérifier les concentra-
tions de l’eau pour que le système reste en équilibre.
2◦ ) Mettre cette relation sous la forme

Π(x1 , x2 , T0 , v) = P1 − P2

où Π est la pression osmotique lorsque la cloison C est retirée. Expliciter Πm en fonction
des données.
3◦ ) Si la relation précédente n’est pas satisfaite, montrer que lorsqu’on enlève la cloison C,
il y a transfert d’eau pure d’un compartiment vers l’autre. Préciser, selon les valeurs de x1 et
x2 le sens de ce transfert.

Christian Carimalo 114 TD de Thermodynamique - L3


C/Osmose inverse Dans le cas où x2 > x1 et où ce transfert s’effectue vers le second com-
partiment, le phénomène observé est appelé osmose inverse. On peut le mettre à profit pour
obtenir de l’eau douce à partir de l’eau de mer.
Le compartiment (1) de l’enceinte contient de l’eau de mer considérée comme une solution
idéale, tandis que le second compartiment (2) contient de l’eau pure. L’ensemble est à la
température T0 et le volume molaire de l’eau est toujours supposé invariable.
1◦ ) Quelle différence de pression minimale ∆P doit exister entre les deux compartiments
pour que l’osmose inverse soit possible ?
2◦ ) Calculer numériquement ∆P sachant qu’à 25◦ C l’eau de mer contient 1 mole de sel par
litre.
3◦ ) Quel est le travail minimum nécessaire pour obtenir 1 litre d’eau pure ?

VI - Applications de l’osmose
A/ Quelle est la molarité (nombre de moles par litre) d’un soluté pour lequel la pression
osmotique est égale à 10 cm d’eau à 25◦ C ?
B/ La pression osmotique du sang est de 7,65 atm. à 37◦ C. Quelle quantité de glucose
(C6 H12 O6 ) doit-on utiliser par litre pour une injection intraveineuse qui doit avoir la même
pression osmotique que le sang ?
C/ Une solution d’hémocyanine de crabe, protéine pigmentée du crabe, est préparée par
dissolution de 0,75 g dans 12 m` d’eau. A 4◦ C, on observe une ascension de 2,6 mm de
la solution dans le tube prolongateur. La solution a une masse spécifique de 1 g cm−3 .
Déterminer la masse molaire de la protéine.

VII - Solution idéale


On considère un mélange d’alcool méthylique (CH3 OH, constituant 1), et d’alcool éthylique
(C2 H5 OH, constituant 2). Ces deux constituants sont totalement miscibles dans la phase
liquide qui sera considérée comme une solution idéale. On admettra aussi que la phase vapeur
du mélange se comporte comme un mélange idéal de gaz parfaits.
La phase liquide et la phase vapeur sont en équilibre à la température T0 . On note respecti-
vement x1 et y1 les fractions molaires du constituant 1 dans la phase liquide et dans la phase
vapeur, P1 et P2 les pressions partielles des constituants 1 et 2 dans la phase vapeur, P la
pression de vapeur à la température T0 .
1◦ ) Rappeler sans démonstration les relations exprimant la condition d’équilibre des deux
phases du mélange.
2◦ ) a) Rappeler la définition d’un mélange idéal de gaz parfaits.
b) Etablir la relation entre P et P1 .
3◦ ) Rappeler la définition d’une solution idéale.
4◦ ) Etablir les lois de Raoult reliant P1 et x1 d’une part, P2 et x1 d’autre part, en désignant par
P10 (T0 ) et P20 (T0 ) les pressions de vapeur saturante des constituants 1 et 2 à la température
T0 .
5◦ ) A la température T0 , la pression de vapeur P (x1 ) exprimée en millimètres de mercure

Christian Carimalo 115 TD de Thermodynamique - L3


est représentée par l’équation

P (x1 ) = 119 x1 + 135

Quelles sont les pressions de vapeur de chacun des constituants 1 et 2 à l’état pur ?
6◦ ) Exprimer x1 en fonction de y1 , P10 (T0 ), P20 (T0 ). En déduire la relation P (y1 ).
7◦ ) Sur une même figure, tracer les courbes représentant P (x1 ) (liquidus) et P (y1 ) (courbe
de rosée).
8◦ ) Le mélange contient globalement 45 g de CH3 OH, 97 g de C2 H5 OH et se trouve à
70% en phase vapeur. Déterminer
a) les compositions de la phase liquide et de la phase vapeur ;
b) la pression de vapeur P ;
c) la position du point représentatif du mélange dans le diagramme de la question 7◦ ).

VIII - Equilibre liquide-solide d’un mélange binaire - point d’eutexie


Une solution liquide supposée idéale contient deux constituants A et B qui ne sont pas
miscibles à l’état solide. On veut étudier les équilibres éventuels de cette solution, sous
pression atmosphérique constante P0 , avec une phase solide constituée soit du corps A pur,
soit du corps B pur, soit des corps A et B ensemble.
On adoptera les notations suivantes :
• TA , TB , LA , LB sont, respectivement, les températures de fusion et les les chaleurs latentes
molaires de fusion des corps A et B purs, respectivement, à la pression P0 . On suppose que
TA > TB ;
• xAA et xBA sont les fractions molaires respectives de A et B dans la solution en équilibre
avec le solide A ;
• xAB et xBB sont les fractions molaires respectives de A et B dans la solution en équilibre
avec le solide B ;
• gA` et g s sont les enthalpies libres molaires de A pour les phases liquide et solide, respec-
A
tivement. Les autres fonctions thermodynamiques seront notées de façon similaire.
1◦ ) Question préliminaire
A partir des définitions des fonctions thermodynamiques enthalpie H et enthalpie libre G,
démontrer que

∂ G
  
2
H = −T
∂T T P,n

2◦ ) Déterminer les variances des équilibres “solution - solide A (ou B)” et “solution - (solide
A + solide B)”.
3◦ ) Rappeler l’expression du potentiel chimique d’un constituant dans une solution idéale,
en précisant de quelles variables il dépend.

Christian Carimalo 116 TD de Thermodynamique - L3


4◦ ) Rappeler, en le justifiant, la relation entre la chaleur latente molaire de fusion LA et les
enthalpies molaires. De combien de variables dépend LA ? On supposera dans la suite que
LA et LB sont constantes.
5◦ ) Exprimer la condition d’équilibre de la solution binaire avec le solide A seul. En déduire
une expression générale de ln xAA en fonction de T et P0 .
∂ LA
 
6◦ ) Démontrer que [ln xAA ] = où R est la constante des gaz parfaits. En
∂T P0 RT 2
déduire xAA (T ), P0 étant fixé. Déterminer de la même manière xBB (T ). En déduire xAB (T ).
7◦ ) Montrer qualitativement que les courbes représentant T (xAA ) et T (xAB ) se coupent en
un point E de coordonnées xE et TE , appelé point d’eutexie. A quel équilibre correspond ce
point ? En rappeler la variance.
8◦ ) Pour simplifier, on admet que la courbe représentant T (xAB ) entre les points (0, TB )
et (xE , TE ) ainsi que la courbe T (xAA ) entre les points (xE , TE ) et (1, TA ) peuvent être
assimilée à des segments de droites. On donne TA = 1400 K, TB = 600 K, TE = 400 K,
xE = 0, 75.
On considère alors un mélange liquide binaire A+B à la température Ti = 600 K sous la
pression P0 , pour lequel xiA = 0, 3.
a) Placer le point représentatif de ce mélange sur un graphique simplifié.
b) On refroidit progressivement le mélange, à pression constante P0 . Déterminer numérique-
ment la température à laquelle la phase solide B apparaı̂t.
c) La température finale est Tf = 440 K. Montrer qu’on observe alors un état d’équilibre
entre une solution liquide A+B et le solide B. Déterminer numériquement la fraction molaire
xBB de B.

IX -
A/ On considère un mélange liquide binaire à la température T constituant une solution
idéale de n`1 moles d’un composé A et de n`2 moles d’un composé B, On note x`1 et x`2 les
titres molaires correspondants. On pose n = n`1 + n`2 .
1◦ ) Rappeler la définition d’une solution idéale. En déduire les expressions de l’enthalpie libre
de mélange ∆Gid et de l’entropie de mélange ∆Sid .
2◦ ) Cette solution idéale est en équilibre avec sa phase vapeur, celle-ci constituant un mélange
idéal de gaz parfaits. On note xv1 et xv2 les titres molaires correspondants.
Etablir les lois de Raoult (lois de la tonométrie) donnant l’expression des pressions partielles
P1v et P2v de chacun des composés dans la phase vapeur en fonction de x`2 . On notera P10 et
P20 les pressions de vapeur des composés purs à la température T .

B/ On considère maintenant une solution binaire non idéale, à la température T , dont l’en-
tropie de mélange est donnée par
h i rn`2
∆S = −nR x`2 ln α + x`1 ln(1 − α) , avec α =
n`1 + rn`2

où r est une constante supérieure à 1 ; R est la constante des gaz parfaits et n = n`1 + n`2 .

Christian Carimalo 117 TD de Thermodynamique - L3


1◦ ) Montrer que cette entropie de mélange peut se mettre sous la forme

∆S = ∆Sid + ∆Sexc

où ∆Sid est l’entropie de mélange d’une solution idéale et ∆Sexc est l’entropie en excès pour
la solution réelle. Montrer que
h n oi
∆Sexc = −nR x`2 ln r − ln 1 + (r − 1)x`2

2◦ ) Calculer l’enthalpie libre d’excès ∆Gexc sachant qu’il n’y a pas d’enthalpie d’excès.
3◦ ) Montrer que le coefficient d’activité γ1 (x`2 ) est donné par
" #
1 x`2 (r − 1)
γ1 (x`2 ) = exp
1 + (r − 1)x`2 1 + (r − 1)x`2

On rappelle que ce coefficient est tel que µ1 (T, P, {xi }) = g1 (T, P ) + RT ln(γ1 x1 ).
3◦ ) Cette solution non idéale est en équilibre à la température T avec sa phase vapeur
supposée être un mélange idéal de gaz parfaits.
a) Récrire les lois de Raoult donnant la pression partielle de chacun des composés en fonction
du titre molaire correspondant dans la solution et du coefficient d’activité.
b) Déduire de la question précédente l’expression du rapport P1v /P10 en fonction de x`2 et
γ1 (x`2 ).
c) Calculer le rapport P1v /P10 pour les valeurs suivantes de x`2 :

x`2 0,05 0.1 0,2 0,3 0,4 0,5 0,6 0,7 0,8 0,9

)
Pour r = 10, tracer la courbe P1v /P1 en fonction de x`1 . La comparer à la courbe obtenue
pour la solution idéale.

Christian Carimalo 118 TD de Thermodynamique - L3


Corrigé TD11

I - Mélange liquide-vapeur, loi de Raoult

1◦ ) gv (T, P ) = g` (T, P ) (v = 1).


2◦ ) gv (T, P ) = µ`A (T, P, xA ) = g` (T, P ) + RT ln xA (solution idéale ; v = 2).
∂ 1
 
3◦ ) RT ln xA = gv (T, P ) − g` (T, P ), d’où [ln xA ] = [vv − v` ]
∂P T RT
RT ∂
 
4◦ ) On admet que v` est pratiquement constant et que vv ' , donc [ln xA ] '
P ∂P T
1 v`
− . Intégrons cette relation (à T fixé) de xA = 1 (xB = 0) à xA = 1 − xB :
P RT
P + ∆P v` ∆P
ln(1 − xB ) = ln −
P RT
∆P v`
 
D’où la relation cherchée : ln 1 + = ln(1 − xB ) + ∆P .
P RT
∆P v`
 
Pour xB  1, on a aussi ∆P/P  1, et la relation ci-dessus devient 1− ' −xB ,
P vv
soit encore en admettant que v`  vv ,

∆P
' −xB
P
L’ajout de sel dans l’eau fait baisser la pression de vapeur.
∂ 1 ∂ gv − g` ∂ g g s h
      
5◦ ) [ln xA ]= [ ] . Or, = − 2 − = − 2,
∂T P R ∂T T  P ∂T T P T T T
∂ hv − h` `v
h étant l’enthalpie. Ainsi, [ln xA ] =− =− .
∂T P RT 2 RT 2
R
6◦ ) Par intégration (à P fixé) tout en supposant que `v est constant : ln(1 − xB ) =
`v
1 1 ∆T ∆T R
− =− , soit  =− ln(1 − xB ).
∆T

T + ∆T T T (T + ∆T ) `v
T2 1+
T
RT 2
Lorsque la solution est diluée (xB  1), on a ∆T ' xB : loi de Raoult pour l’ébullios-
`v
copie. Après introduction d’un soluté, la température d’ébullition est plus élevée.
1000 nsol. 0, 1 ∆P
7◦ ) neau = ; xB = = ' 1, 8 10−3 . D’où = −1, 8 10−3 ;
18 neau + nsol. 0, 1 + 1000/18 P
8, 32 × (373)2
∆T = 1, 8 10−3 = 5 10−2 K.
540 × 4, 18 × 18
xxxxxxxxxxxxxxxxxxxxxxxxxxxx

Christian Carimalo 119 TD de Thermodynamique - L3


II - Cryoscopie
1◦ ) gA
s (T , P ) = g ` (T , P ).
A 0 A A 0

2◦ ) g s (T, P0 ) = gA
` (T, P ) + RT ln x .
0 A

3◦ ) RT ln xA = g s (T, P0 )−gA
` (T, P ). Si la solution est diluée, x  1 et | ln x | ' x  1.
0 B A B
On fait alors les approximations suivantes : g s (T, P0 )−gA
` (T, P ) ' g s (T , P )−g ` (T , P )+
0 A 0 A A 0
h
` s
i RTA2
(T − TA ) sA (TA , P0 ) − sA (TA , P0 ) ≡ (∆T )`A (TA )/TA . D’où ∆T ' − xB .
`A
12 3 5 3 nprest. RTA2
4◦ ) a) neau = 10 , nprest. = 10 , d’où xB = = 0, 108 ; K = =
18 62 neau + nprest. `A
8, 32 × (273, 16)2
= 103, 65 K/mole. On trouve ∆T ' −11◦ C.
79, 6 × 4, 18 × 18
malc. /32 mprest.. /62
b) On obtient le même résultat si x0B = xB , soit = ,
malc. /32 + 12/18 mprest. /62 + 12/18
32
soit malc. = mprest. ' mprest. /2 (plus économique).
62
xxxxxxxxxxxxxxxxxxxxxxxxxxxx

III - Solution diluée en équilibre avec sa vapeur, loi de Henry

1◦ ) a) µv1 = g1v (T, P ) + RT ln xv1 = µ`1 = g1` (T, P ) + RT ln x`1 (solution idéale, mélange idéal
Pv
de gaz parfaits) ; xv1 = 1 .
P
b) En considérant la vapeur comme un gaz parfait, on a la relation g1v (T, P ) = g1v (T, P 0 ) +
P P
RT ln 0 , et notamment, g1v (T, P ) = g1v (T, P10 ) + RT ln 0 . En considérant le volume
P P1
molaire du liquide comme quasiment constant, on a g1` (T, P ) ' g ` (T, P10 ) + v1` (P − P10 ).
P xv Pv
Comme g1v (T, P10 ) = g1` (T, P10 ), on déduit de a) : RT ln ` 10 = RT ln ` 1 0 ' v1` (P − P10 ),
x1 P1 x1 P1
et, en considérant que v1 /(RT /P )  1, on a approximativement P1 = P10 x`1 .
` v
!
X X X
2◦ ) dG = V dP − SdT + µϕ ϕ
c dnc = d µϕ ϕ
c nc , d’où nϕ ϕ
c dµc = V dP − SdT .
c,ϕ c,ϕ c,ϕ
Les potentiels chimiques d’une phase donnée ne dépendent que de T , P et des fractions
molaires des constituants dans cette phase. Le second membre de la relation de Gibbs-Duhem
ci-dessus !ne contenant pas ! les différentielles de ces fractions molaires, on a par exemple
∂µ1` ∂µ2`
x`1 `
+ x`2 = 0 (x`1 + x`2 = 1), équation manifestement satisfaite avec
∂x1 T,P ∂x`1 T,P
∂ ∂
l’hypothèse de la solution idéale car x`1 ` [ln x`1 ] + x`2 ` [ln x`2 ] = x`1 /x`1 + (−1)x`2 /x`2 = 0.
∂x1 ∂x1
P10 Pv
3◦ ) a) b) µv1 = g1v (T, P ) + RT ln x`1 + RT ln ; µv2 = g2v (T, P ) + RT ln 2 .
P P
P v
c) d) En plus rapide : µv2 = µ`2 donne g2v (T, P ) + RT ln 2 = g2` + RT ln x`2 , d’où
P
" #
Pv g v (T, P ) − g2` (T, P )
RT ln ` 2 = g2` (T, P ) − g2v (T, P ), soit P2v = x`2 P exp − 2 = K2 x`2
x2 P RT

Christian Carimalo 120 TD de Thermodynamique - L3


où K2 est une fonction de T et P .
103
4◦ ) 1 ` d’eau contient moles d’eau. Avec les données, la loi de Henry donne pour la
18
0, 2
fraction molaire de O2 : x`O2 = = 5 10−6 , ce qui donne, par litre d’eau : x2 =
4 104
103
5 10−6 × = 2, 77 10−4 , valeur théorique proche de la valeur mesurée.
18
xxxxxxxxxxxxxxxxxxxxxxxxxxxx

IV - Ebullioscopie
On note u` (T, P ), h` (T, P ), s` (T, P ) respectivement, l’énergie interne, l’enthalpie et l’entro-
pie d’une mole d’eau liquide à la température T et sous la pression P . Dans les mêmes
conditions, on note uv (T, P ), hv (T, P ), sv (T, P ), respectivement, les mêmes fonctions pour
la vapeur d’eau.
1◦ ) f = u − T s, g = f + P v ; fv = uv − T sv , gv = uv − T sv + P vv ; f` = u` − T s` ,
g` = u` − T s` + P v` .
2◦ ) a) La variance de ce système est v = c + 2 − ϕ = 1 + 2 − 2 = 1 : l’équilibre est défini
par une seule variable intensive. Si la pression est donnée, la température est fixée.
b) G = nv gv (T, P ) + n` g` (T, P ), enthalpie libre du système (eau vapeur)-(eau -liquide) est
minimum à l’équilibre entre les deux phases, à T et P fixés, ce qui se traduit par dG =
dnv [gv (T, P ) − g` (T, P )] = 0 (nv + n` = 1). La condition d’équilibre à T et P donnés est
donc l’égalité gv (T, P ) = g` (T, P ) (qui montre qu’à P donné, l’équilibre n’est réalisé qu’à la
température fixée par cette égalité).
3◦ ) µ` = g` (T, P )+RT ln xeau , avec xeau = 1−xA et xA  1. Donc µ` ' g` (T, P )−RT xA .
4◦ ) a) b) La condition d’équilibre entre la vapeur d’eau et l’eau liquide est maintenant
µ` = gv , d’où l’égalité gv (T, P0 ) − g` (T, P0 ) ' −RT xA ' −RT0 xA . Mais

`(T0 )
gv (T, P0 ) − g` (T, P0 ) ' −∆T [sv (T0 , P0 ) − s` (T0 , P0 )] = −∆T
T0

RT02 xA
On obtient ainsi : ∆T ' > 0.
`(T0 )
nA nA m Meau
5◦ ) x A = ' = . La mesure de ∆T permet d’obtenir xA , puis MA .
N + nA M MA M
RT02 m 8, 32 × (373)2
6 ◦ ) MA = Meau = × 0, 01 × 18 = 257 g.
`∆T M 40550 × 0, 02

xxxxxxxxxxxxxxxxxxxxxxxxxxxx

V - Osmose
Remarque générale : on ignore ici les effets éventuels de la pesanteur, ce qui peut être justifié
si la dimension verticale des compartiments est suffisamment faible.
A/ 1◦ ) 2◦ ) Les solutions étant supposées idéales, les potentiels chimiques de l’eau dans
le compartiment (1) et dans le compartiment (2) sont respectivement µ1 = g` (T0 , P0 ) +
RT0 ln x1 et µ2 = g` (T0 , P0 ) + RT0 ln x2 , g` étant l’enthalpie libre molaire de l’eau liquide,

Christian Carimalo 121 TD de Thermodynamique - L3


n1 n2
et x1 = = 1 − xa , x2 = = 1 − xb . Avant que la cloison C soit enlevée, on
n1 + na n2 + nb
x1
a donc µ1 − µ2 = RT0 ln , et µ1 < µ2 si x1 < x2 , µ1 > µ2 si x1 > x2 .
x2
Lorsqu’on enlève la cloison, l’eau du compartiment (1) est mise en contact avec l’eau du
compartiment (2) et ces quantités d’eau ne peuvent être à ce moment en équilibre à cause
de la différence de leurs potentiels chimiques. Comme les transferts de matière s’effectuent
dans le sens décroissant des potentiels chimiques, si µ1 < µ2 de l’eau pure va s’écouler du
compartiment (2) vers le compartiment (1) à travers la menbrane M , de telle sorte à réduire
puis finalement éliminer ce déséquilibre, dû à la différence des concentrations x1 et x2 . Cet
écoulement s’arrêtera lorsque l’égalité des concentrations sera réalisé. On aura alors x0a = x0b ,
n0 n0 n0 + n02 ne ne na ne nb
ou 1 = 2 = 1 = , soit n01 = , n0 = . Bien entendu,
na nb na + nb na + nb na + nb 2 na + nb
l’écoulement s’effectue dans l’autre sens si au départ µ2 < µ1 .

B/ 1◦ ) 2◦ ) µ1 = g` (T0 , P1 )+RT0 ln x1 = µ2 = g` (T0 , P2 )+RT0 ln x2 . Considérant que l’eau


liquide est quasiment incompressible, on peut écrire g` (T0 , P1 ) − g` (T0 , P2 ) = v(P1 − P2 ),
RT0 x2
et l’égalité des potentiels chimiques se traduit par P1 − P2 = ln , ou, puisque xa =
v x1
RT0
1 − x1  1, xb = 1 − x2  1 : P1 − P2 = Π, avec Π ' (xa − xb ).
v
x2 v(P1 − P2 )
 
3◦ ) Supposons µ1 < µ2 au départ, soit > exp > 1 ; dans ce cas, de
x1 RT0
l’eau pure s’écoulera du compartiment (2) vers le compartiment (1), jusqu’à égalité des
concentrations. L’écoulement se fera dans l’autre sens si au départ µ1 > µ2 .
C/ 1◦ ) Dans les conditions de l’osmose inverse, on doit avoir µ1 > µ2 , soit
x2 v(P1 − P2 ) RT0 x2
 
1< < exp , ou ∆P = P1 − P2 > ln
x1 RT0 v x1
La différence de pression minimale ∆P à imposer entre les deux compartiments pour que
RT0 x2
l’osmose inverse soit possible est donc ∆m P = ln . Ici, x2 = 1, 1 − x1 = xsel  1,
v x1
RT0
d’où ∆m P = xsel .
v
RT0 nsel nsel 1
2◦ ) ∆ m P = = RT0 = 8, 32 × 298 × −3 ' 25 atm.
v ne Veau 10
3◦ ) Wm = ∆m P δV = 25 105 × 10−3 = 2500 J.

xxxxxxxxxxxxxxxxxxxxxxxxxxxx

VI - Applications de l’osmose

nsoluté nsoluté
A/ La pression osmotique est donnée par Π = RT = ρeau g∆h. Donc =
V V
ρeau g∆h 1 × 10 × 0, 1
= = 4 10−4 mole/litre.
RT 2500
nsoluté msoluté
B/ Π = RT = RT , donc
V Msoluté V
msoluté ΠMsoluté 7, 65 × 1, 013 × 105 × 180 × 10−3
= = = 54, 1 g/`
V RT 8, 32 × 310

Christian Carimalo 122 TD de Thermodynamique - L3


nsoluté msoluté msoluté RT
C/ Π = RT = ρeau g∆h = RT , d’où Msoluté = , soit
V Msoluté V V ρeau g∆h
0, 75 8, 32 × 277
Msoluté = = 542 kg
0, 125 10 × 9, 81 × 2, 6 10−3
3

xxxxxxxxxxxxxxxxxxxxxxxxxxxx

VII - Solution idéale

1◦ ) Voir le cours. 2◦ ) a) Voir le cours.


2◦ ) b) P1 = y1 P (de même, P2 = y2 P , avec y2 = 1 − y1 ).
3◦ ) Voir le cours.
P1
4◦ ) µ`1 = g1` (T0 , P ) + RT ln x1 = µv1 = g1v (T0 , P1 ) = g1v (T0 , P10 (T0 )) + RT0 ln 0 . Si
P1 (T0 )
l’on admet que g1` (T0 , P ) varie peu avec P , on peut écrire g1` (T0 , P ) ' g1` (T0 , P10 (T0 )). Il
vient alors P1 ' x1 P10 . Avec des approximations similaires, on trouve aussi P2 ' x2 P20 . On
obtient ainsi P1 + P2 = P = x1 P10 + x2 P20 .
5◦ ) P10 = P (x1 = 1) = 254, P20 = P (x1 = 0) = 135 (en mm Hg).
y1 P20
6◦ ) P1 = y1 P = x1 P10 = y1 x1 P )1 + (1 − x1 )P20 , d’où x1 =
 
, et
P10 + y1 (P20 − P10 )
x1 0 P10 P20
P = P1 , soit P = 0 .
y1 P1 − y1 (P10 − P20 )
P10 − P20
On note que y1 − x1 = y1 (1 − y1 ) 0 ≥ 0.
P1 − y1 (P10 − P20 )
7◦ )
P
254

P(x1)

P(y1)
L
200
L+V

C D
170,4
E
V

135 0,44 1
0,3 0,4 0,5 x1 y1 X1

Figure 16 – Courbes liquidus et courbe de rosée

8◦ ) Remarques. Notons n1 le nombre de mole total du constituant (1), n2 celui du constituant


(2). Le nombre de mole total du mélange est n = n1 + n2 , la fraction molaire du constituant
n1 n2
(1) est X1 = et celle du constituant (2) est X2 = . Soient nv1 , n`1 les nombre de
n n

Christian Carimalo 123 TD de Thermodynamique - L3


mole du constituant (1) en phase vapeur et en phase liquide respectivement, nv2 et n`2 les
grandeurs similaires pour le constituant (2). Les nombres de mole total en phase vapeur et
en phase liquide sont respectivement nv = nv1 + nv2 et n` = n`1 + n`2 , et les fractions molaires
nv n` nv1,2
respectives en phase vapeur et en phase liquide sont xv = , x` = . On a y1,2 = ,
n n nv
n`1,2
x1,2 = . On note que la fraction molaire totale du constituant (1) s’écrit
n`
n1 nv nv n` n`
X1 = = 1 + 1 = y1 x v + x 1 x `
n nv n n` n

On note que X1 − x1 = xv (y1 − x1 ), y1 − X1 = x` (y1 − x1 ) et par suite, x1 ≤ X1 ≤ y1 . Le


X1 − x1 xv CD
rapport = s’interprète géométriquement comme dans la figure (16).
y1 − X1 x` DE
La donnée de X1 fournit une autre relation entre y1 et x1 , laquelle, combinée avec la relation
entre ces dernières variables qui a éé établie au 6◦ ), conduit à une équation du second degré
permettant d’obtenir y1 , puis x1 .
45 97 1, 4
On a MCH3 OH = 32 g, MC2 H5 OH = 46 g, n1 = = 1, 4, n2 = = 2, 1, X1 = =
32 46 3, 5
135y1
0, 4, xv = 0, 7, x` = 0, 3, d’où la relation 3x1 + 7y1 = 4. Comme x1 = , on en
254 − 119y1
déduit l’équation du second degré : 833y12 − 2659y1 + 1016 = 0, ayant pour seule solution
acceptable y1 = 0, 44. Il vient ensuite x1 = 0, 3, puis P ' 170, 4 mm Hg. Dans la figure
(16), D est le point représentatif du mélange.

xxxxxxxxxxxxxxxxxxxxxxxxxxxx

VIII - Equilibre liquide-solide d’un mélange binaire - point d’eutexie

∂G ∂G/T
X    
1◦ ) G = H − T S, dG = V dP − SdT + µk dnk , d’où = −S, et =
k
∂T P,n ∂T P,n
G S H ∂ G
 
− 2
− = − 2 , soit H = −T 2 .
T T T ∂T T P,n

2◦ ) v = 2 pour l’équilibre “solution - solide A (ou B)” ; v = 1 pour l’équilibre “solution -


(solide A + solide B)”.
3◦ ) µi (T, P, xi ) = µ0i (T, P ) + RT ln xi , où, souvent, µ0i est pris comme l’enthalpie libre
molaire du constituant (i) pur, dans la phase considérée.
h i
4◦ ) Voir le cours : LA = LA (T ) = T s`A (T, P (T )) − ssA (T, P (T )) .

g s (T, P0 ) − g ` (T, P0 )
5◦ ) µ`A = µsA , soit g ` (T, P0 ) + RT ln xAA = g s (T, P0 ), soit ln xAA = .
RT
∂ ∆h
 
6◦ ) D’après le 1◦ ), [ln xAA ] = 2
où ∆h = h`A (T, P0 ) − hsA (T, P0 ). Si l’on
∂T P0 RT
admet que ∆h est quasiment constant vis-à-vis de T , on peut l’identifier à sa valeur pour la
température de
 fusion à la pression P0 (TA ), soit ∆h = LA = constante. On  obtient ainsi
∂ LA LA 1 1

[ln xAA ] = 2
. Intégrant cette relation, on obtient ln xAA = − , où
∂T P0 RT R TA T
l’on a tenu compte du fait que T = TA correspond au cas où le constituant A est à l’état

Christian Carimalo 124 TD de Thermodynamique - L3


pur sous les deux phases
 liquide et solide. En procédant de même pour le constituant B, on
LB 1 1
trouve ln xBB = − . On a xAB = 1 − xBB .
R TB T
d LA dT (xAA ) ∂ LB
 
7◦ ) [ln xAA ] = 2
> 0, donc > 0, tandis que [ln xBB ] = >0
dT RT dxAA ∂T P0 RT 2
dT (xAB )
conduit à < 0. Représentons les deux fonctions T (xAA ) et T (xAB ) dans un même
dxAB
diagramme (x, T ). La courbe représentative de T (xAB ) commence au point (0, TB ) et va en
décroissant à mesure que xAB augmente. La courbe représentative de T (xAA ) va croissant
jusqu’au point terminal (1, TA ). Il est donc évident que ces deux courbes se coupent en un
point E(xE , TE ). Ce point correspond à la fois à un équilibre entre la solution A+B et le
solide A, et à l’équilibre entre la solution A+B et le solide B : c’est l’équilibre, monovariant,
entre la solution A+B et les deux solides A et B.
800
8◦ ) T (xAB ) = 600 − xAB , T (xAA ) = −2600 + 4000 xAA .
3

1500

TA

liquide (A+B)
1000

liq +
TB solide A
M
liq. + solide B
N E
TE

solides A et B

1
0,3
0,5 0,6 xE xAA xAB

Figure 17 – Diagramme de phases simplifié d’un mélange

a) Point M de la figure (17).


b) T = 520 K.
c) Une fois le processus de solidification de B engagé, on suit la courbe T (xAB ). Pour
3
T = 440 K, on se trouve au point N de cette courbe. Donc xAB = (600 − 440) = 0, 6,
800
et xBB = 1 − xAB = 0, 4. Si l’on continue, au point E on voit apparaı̂tre le solide A.
Note : au point E, on a xAA = xAB = xE , xAB = xBA = 1 − xE .

xxxxxxxxxxxxxxxxxxxxxxxxxxxx

Christian Carimalo 125 TD de Thermodynamique - L3


IX -
" #
x`1 x` rx`
B/ 1◦ ) ∆Sexc = ∆S − ∆Sid = nR x`1 ln + x`2 ln 2 , avec α = ` 2 ` =
1−α α x1 + rx2
rx`2 x`1 n o
`
, 1−α = `
, d’où ∆Sexc = nR (x`1 + x`2 ) ln[1 + (r − 1)x`2 ] − x`2 ln r ,
1 + (r − 1)x2 1 + (r − 1)x2
soit
h n oi
∆Sexc = −nR x`2 ln r − ln 1 + (r − 1)x`2
h n oi
2◦ ) ∆Gexc = −T ∆Sexc = nRT x`2 ln r − ln 1 + (r − 1)x`2 .

3◦ ) On 0 0 ` ` ` `
h a G = G + i∆Gid + ∆Gexc , avec G = n1 g1 (T, P ) + n2 g2 (T, P ), ∆Gid =
nRT x`1 ln x`1 + x`2 ln x`2 . Le potentiel chimique µ`1 est défini par
!
∂G
= µ`1 = g1` (T, P ) + RT ln x`1 γ1 .
∂n`1 `
!T,P,n2 !
∂G0 ∂∆Gid
Or, = g1` (T, P ), = RT ln x`1 . On peut donc obtenir le coeffi-
∂n`1 T,P,n`2
∂n`1 T,P,n`2
!
∂∆Gexc
cient d’activité γ1 au moyen de la relation RT ln γ1 = . On a
∂n`1 T,P,n`2
1 n` + rn`2
∆Gexc = n`2 ln r − (n`1 + n`2 ) ln 1` , d’où
RT n1 + n`2
h i (r − 1)x`2
ln γ1 = − ln 1 + (r − 1)x`2 + , soit
1 + (r − 1)x`2
" #
1 x`2 (r − 1)
γ1 (x`2 ) = exp
1 + (r − 1)x`2 1 + (r − 1)x`2

3◦ ) a) b) De la relation d’équilibre µ`1 = g1` (T, P ) + RT ln[x`1 γ1 ] = g1v (T, !


P1v ) = g v (T, P10 ) +
P v P v ` `
g (T, P ) − g (T, P1 ) 0 P v1 ` P 0
RT ln 10 , on tire ln ` 1 0 = 1 ' 1 − 1 , et comme v1` 
P1 [x1 γ1 P1 ] RT RT P
RT P1v
, on a approximativement R = 0 ' x`1 γ1
P P1
u
exp
Remarque On a γ1 = 1 + u avec u = (r − 1)x` . Posons 1 − y = u , y = 1 . Il
2
1+u 1+u 1+u
vient alors ln γ1 = 1 − y + ln y, qui est une quantité toujours négative. Donc γ1 < 1 et la
Pv
courbe représentant R(x`2 ) reste toujours en dessous de la droite 10 = 1 − x`2 correspondant
P1
au cas de la solution idéale.

x`2 0,05 0.1 0,2 0,3 0,4 0,5 0,6 0,7 0,8 0,9

1 − x`2 0,95 0.9 0,8 0,7 0,6 0,5 0,4 0,3 0,2 0,1

R 0,89 0.76 0,54 0,39 0,29 0,21 0,15 0,1 0,06 0,03

Christian Carimalo 126 TD de Thermodynamique - L3


R
1

0,5

x2l
0,5 1

Figure 18 – Tracé qualitatif de R(x`2 ) pour r = 10

xxxxxxxxxxxxxxxxxxxxxxxxxxxx

Christian Carimalo 127 TD de Thermodynamique - L3


TD12

- Potentiels chimiques généralisés -

I - Suspension colloı̈dale
Un fluide homogène est contenu dans un récipient et maintenu à la température constante
T0 . On désigne par Oz l’axe vertical ascendant dont l’origine se trouve à la base du récipient.
Concernant la pression, on ignorera l’effet de la pesanteur et l’on supposera donc la pression
uniforme et égale à la pression atmosphérique Pa . La poussée d’Archimède sera également
ignorée.
Dans le récipient se trouve une solution liquide très diluée d’une protéine Na A. On suppose la
protéine entièrement dissociée en ions Na+ et en ions A− , chaque espèce d’ions constituant
alors un soluté très dilué, soluté (1) pour Na+ , soluté (2) pour A− . La solution sera supposée
idéale.
Expérimentalement, on observe une différence de potentiel électrique U entre la base du
récipient et la surface libre de la solution. On veut déterminer la différence de potentiel
électrique entre la base du récipient et un point de cote z dans la solution.
1◦ ) En s’inspirant en partie du problème de l’atmosphère isotherme du TD7, justifier le fait
que le potentiel généralisé de l’une ou l’autre espèce d’ions dans la solution a pour expression

µ?i (T0 , P, z) = gi (T0 , P ) + RT0 ln xi (z) + NA (mi gz + qi V (z)) i = 1, 2

où mi , qi , xi (z) sont respectivement la masse, la charge et la fraction molaire à la cote z de


l’ion considéré ; NA est le nombre d’Avogadro, R la constante des gaz parfaits.
2◦ ) Que peut-on dire de µ?i ? Justifier la réponse.
3◦ ) a) Sachant que U est de l’ordre du millivolt, comparer q1 U et m1 gh pour Na+ . On
donne : q1 = 1, 6 10−19 C, m1 = 3, 82 10−26 kg, h = 10 cm.
b) Compte tenu de ce résultat, déduire une expression simplifiée de µ?1
c) Trouver alors l’expression de x1 (z) en fonction de V (z), x1 (0), et des autres données.
4◦ ) Trouver l’expression de x2 (z) en fonction de V (z), x2 (0), m2 , T0 .
5◦ ) a) Localement, la solution est électriquement neutre. Quelle équation permet d’exprimer
ce fait ?
b) En déduire l’expression de V (z) en fonction de z, m2 , g.
c) Application. On trouve U = 5 10−4 V. En déduire une valeur numérique de la masse
molaire de la protéine.

Christian Carimalo 128 TD de Thermodynamique - L3


II - Etude d’un diélectrique
A/ On considère un diélectrique homogène, linéaire et isotrope. L’échantillon, constitué de N
noles, de volume V , soumis à la pression P et à la température T , est placé dans un champ
−→ −→
électrique uniforme E . On note v le volume molaire et P la polarisation par mole. On
rappelle que le travail élémentaire des forces électriques appliquées au seul matériau est, par
mole, d̄ We = E dP. On admettra que la polarisation molaire P est liée au champ appliqué
E par la relation

P a
= ε − ε0 = (1)
vE vT

ε étant la permittivité du diélectrique, ε0 ' 9 10−12 SI celle du vide, a une constante.


1◦ ) Ecrire la fonction enthalpie libre généralisée G? et montrer que G? = N µ où µ est le
potentiel chimique. En déduire la relation de Gibbs-Duhem généralisée.
2◦ ) Exprimer le potentiel chimique dans le cas où le diélectrique est un liquide incompressible.

B/ Deux cuves (1) et (2) contenant le même diélectrique liquide, supposé incompressible,
sont en communication comme indiqué à la figure (19).

(1) (2)

A1 A2

Figure 19 – Diélectrique liquide, avant application du champ électrique

−→
La cuve (1) peut-être placée dans le champ électrique E , la température étant fixée à
T . Comparer les deux potentiels chimiques aux deux points A1 et A2 situés à la même
profondeur, quand on vient juste d’établir le champ électrique. On peut prévoir l’apparition
d’une dénivellation entre les surfaces libres des liquides dans les deux cuves, correspondant à
une différence de pression ∆P = PA1 − PA2 .
1◦ ) Lorsque l’équilibre est établi, calculer cette dénivellation h. On notera M1 la masse
molaire du liquide diélectrique et g l’accélération de la pesanteur.
2◦ ) A quoi fait penser ∆P ?
3◦ ) Application numérique. Le liquide diélectrique est de l’eau à 300 K. Sa permittivité vaut
alors ε = 83 ε0 . Calculer h pour E = 105 V/m.

Christian Carimalo 129 TD de Thermodynamique - L3


Corrigé TD12

I - Suspension colloı̈dale

1◦ ) Rédaction laissée au lecteur.


2◦ ) Suivant le raisonnement utilisé au TD7, la solution étant à l’équilibre, µ?i doit être
indépendant de z.
m1 gh 3, 82 10−26 × 10 × 0, 1
3◦ ) a) = ' 2, 4 10−4  1.
q1 U 1, 6 10−19 × 10−3
b) Compte tenu de ce résultat, on peut, concernant µ?1 , ignorer l’effet de pesanteur et prendre
pour cette grandeur l’expression simplifiée µ?1 ' g1 (T0 , Pa ) + RT0 ln x1 (z) + NA q1 V (z))
c) Exprimant que µ?1 est indépendant de z, on trouve

NA q1 V (z)
 
x1 (z) = x1 (0) exp −
RT0

avec la convention V (0) = 0.


4◦ ) En exprimant l’indépendance de µ?2 vis-à-vis de z, on trouve aussi
NA (m2 gz + q2 V (z)
 
x2 (z) = x2 (0) exp −
RT0
5◦ ) a) x1 (z) = x2 (z).
m2 gz
b) q1 V (z) = m2 gz + q2 V (z), et comme q2 = −q1 , on trouve V (z) = .
2q1
m2 gh 2NA q1 U
c) U = , d’où la masse molaire de la protéine : M2 = ' 100 kg.
2q1 gh
xxxxxxxxxxxxxxxxxxxxxxxxxxxx

II - Etude d’un diélectrique

A/ 1◦ ) G? = U − T S + P V − N EP, et dG? = −SdT + V dP − N PdE + µdN . Dans


un changement de taille infinitésimal N → N (1 + dλ), T → T , P → P , E → E, G? →
G? (1 + dλ). En exploitant l’expression générale de dG? , on trouve aisément G? = µN . En
différentiant cette dernière expression de G? , on trouve la relation de Gibbs-Duhem généralisée
N dµ = −SdT + V dP − N PdE, ou dµ = −sdT + vdP − PdE
en introduisant l’entropie molaire s et le volume molaire v.
aE
2◦ ) Si v peut être considéré comme constant, on a d [µ − P v] = −sdT − dE, ou
" # " # T
aE 2 aE 2
d µ − Pv + = dT −s − . Le membre de gauche de cette relation étant une
2T 2T 2
aE 2
d.t.e, le facteur de dT du membre de droite est nécessairement une fonction de T : s+ 2 =
2T
−ψ(T ). Notant ϕ(T ) une primitive de ψ(T ), on trouve, par intégration :

Christian Carimalo 130 TD de Thermodynamique - L3


aE 2
µ = Pv − + ϕ(T )
2T

aE 2
B/ 1◦ ) A l’équilibre, µA1 = − + vPA1 + ϕ(T ) = µA2 = vPA2 + ϕ(T ), d’où l’on tire
2T
aE 2 E2
∆P = PA1 −PA2 = = (ε−ε0 ) (à noter qu’on retrouve ici l’expression d’une pression
2vT 2
M1 aE 2
électrostatique). D’un autre côté, ∆P = ρgh, où ρ = . On en déduit h = .
v 2gM1 T
Remarque. Il eut été plus correct de considérer dès le départ un potentiel chimique généralisé
incluant l’effet de la pesanteur : µ?? = µ? + M1 gz. Plus précisément, pour la première cuve :
aE 2
µ??
1 = P1 v− +ϕ(T )+M1 gz, et pour la seconde cuve : µ?? 2 = P2 v+ϕ(T )+M1 gz, z étant
2T
la cote du point envisagé par rapport au fond de la cuve. Chacun des liquides des deux cuves
étant pour sa part en équilibre propre, ces potentiels chimiques doivent être indépendants de
z. Ceci amène à écrire, pour la cuve (1) : vP1 (z) + M1 gz = vP0 + M1 gH1 , H1 étant la cote
de la surface libre du liquide (1) par rapport au fond de la cuve, laquelle se trouve à la pression
atmosphérique P0 ; de même, pour la cuve (2) : vP2 (z)+M1 gz = vP0 +M1 gH2 . On en déduit,
M1 g M1 g
pour la cuve (1) : P1 (z) = P0 + (H1 −z), et pour la cuve (2) : P2 (z) = P0 + (H2 −z).
v v
On retrouve simplement par ce biais la loi fondamentale de l’hydrostatique. Pour les deux
M1 g
point A1 et A2 se trouvant à la même cote z, on a donc PA1 − PA2 = (H1 − H2 ).
v
Puis, exprimant que les deux liquides sont en équilibre mutuel, par exemple pour une cote z
aE 2
donnée, vPA1 − + ϕ(T ) + M1 gz = vPA2 + ϕ(T ) + M1 gz, ce qui conduit aux équations
2T
écrites plus haut.
2◦ ) ∆P est l’équivalent de la pression osmotique.
a 410 E 2 M1
3◦ ) De ε − ε0 = = 82ε0 on tire a/T = 82ε0 v et h = avec ρ = . On trouve
vT ρg v
h ' 0, 4 mm.

xxxxxxxxxxxxxxxxxxxxxxxxxxxx

Christian Carimalo 131 TD de Thermodynamique - L3


TD13

- Réactions chimiques -

I - Réactions chimiques entre gaz parfaits


A température T et pression P constantes, ν1 moles d’un gaz A1 réagissent avec ν2 moles
d’un gaz A2 pour donner deux gaz A3 et A4 suivant la réaction d’équilibre

ν1 A1 + ν2 A2 ←→ ν3 A3 + ν4 A4

Quand il s’est formé N3 moles de A3 , on dit que le degré d’avancement de la réaction est
ε = N3 /ν3 .
1◦ ) Evaluer les nombres de moles N1 , N2 , N4 des gaz A1 , A2 , A4 en fonction des nombres
νk (k allant de 1 à 4) et de ε.
2◦ ) On admet que les quatre gaz sont parfaits et que leur mélange est idéal. On note N
le nombre total de moles du système. Rappeler l’expression du potentiel chimique µk de
l’un quelconque des gaz Ak en fonction de son enthalpie libre molaire gk lorsqu’il est seul,
Nk , N, T et R, constante des gaz parfaits.
3◦ ) On envisage une variation infinitésimale dε de ε. Déterminer la variation consécutive
dGT,P de l’enthalpie libre du mélange en fonction des µk , νk et dε.
4◦ ) Soit ∆G la variation d’enthalpie libre du système au cours de la réaction. Pour la définir,
on suppose que avant et après réaction, les gaz sont séparés.
∂G
 
Déterminer en fonction de ∆G, des Nk , νk , de N, T et R. En donner la valeur
∂ε T,P
pour : ε = 0, ε = 1 et ε = εe , valeur de ε à l’équilibre de la réaction.
5◦ ) Tracer qualitativement l’évolution G(ε) au cours de cette réaction à T et P constants.
6◦ ) En utilisant 3◦ ) et 4◦ ),
a) déterminer à l’équilibre (ε = εe ) la relation entre les µk et νk et le rapport

xν33 xν44
Kc =
xν11 xν22

où xk = Nk /N est la concentration du gaz Ak ;


∂Kc ∂Kc
   
b) Calculer et . Commenter les résultats.
∂T P ∂P T

Christian Carimalo 132 TD de Thermodynamique - L3


II - Synthèse de l’ammoniac
On étudie la réaction de synthèse de l’ammoniac en phase gazeuse à température T et
pression P constantes :

N2 + 3 H2 ←→ 2 N H3

On notera respectivement par les indices 1, 2, 3 les quantités se rapportant à l’azote, l’hy-
drogène et l’ammoniac. On admettra que les trois gaz sont parfaits et que leur mélange est
idéal. Quand il s’est formé n3 moles d’ammoniac, on définit le degré d’avancement λ de la
réaction par λ = n3 /2.
1◦ ) Rappeler l’expression du potentiel chimique µi de l’un quelconque des gaz en fonc-
tion de son enthalpie libre molaire gi (T, P0 ), P0 étant la pression de référence (pression
atmosphérique) et de Pi , pression partielle du gaz dans le mélange.
2◦ ) Pour une variation infinitésimale dλ de λ, déterminer la variation dGT,P de l’enthalpie
libre du mélange en fonction des µi et de dλ.
3◦ ) On définit la variation d’enthalpie libre de référence de la réaction ∆G0 (T ) comme la
différence, à la pression de référence P0 , entre les enthalpies libres des produits de la réaction,
et celles des réactifs, en supposant que avant et après la réaction les gaz sont séparés :

∆G0 (T ) = 2 g3 (T, P0 ) − g1 (T, P0 ) − 3 g2 (T, P0 )

P32 P02
Déterminer, à l’équilibre, la relation entre ∆G0 (T ) et la constante d’équilibre Kp = .
P1 P23
4◦ ) On part d’une mole d’azote et de trois moles d’hydrogène et on opère sous une pression
totale constante et égale à 1 atm. et à température constante T0 = 723 K.
a) Calculer Kp sachant que ∆G0 (T0 ) = 60, 6 kJ.
b) Quel est le nombre de moles d’ammoniac obtenu ?
Quel est le rendement de la réaction de synthèse dans ces conditions ?
5◦ ) Sous quelle pression totale faudrait-il opérer pour obtenir une mole d’ammoniac à
l’équilibre, les autres conditions restant les mêmes ?

III - Dissociation de N2 O4
On considère la réaction de dissociation du tétraoxyde d’azote N2 O4 , nommé corps (a), en
dioxyde d’azote N O2 , nommé corps (b), en phase gazeuse :

N2 O4 ←→ 2 N O2

Le mélange des deux gaz sera supposé constituer un mélange idéal de gaz parfaits.
Initialement, on introduit n0 noles de gaz (a) pur dans un récipient vide de volume V placé
dans un thermostat à la température T . Lorsque le mélange comporte na moles de gaz (a)
et nb moles de gaz (b), le taux de dissociation de N2 O4 est par définition
n0 − na
α=
n0

Christian Carimalo 133 TD de Thermodynamique - L3


" #
∆G0
On pose K = exp − avec ∆G0 = 2 gb (T, P0 ) − ga (T, P0 ), où ga et gb sont les
RT
enthalpies libres molaires respectives de (a) et (b) pour la température T et la pression de
référence P0 = 105 Pa = 1 atm. On notera P la pression totale du mélange.
1◦ ) Dire pourquoi l’énergie libre F du mélange des deux gaz est le potentiel thermodynamique
approprié pour cette réaction.
∂F
 
2◦ ) Exprimer en fonction des potentiels chimiques des deux gaz dans le mélange.
∂α T,V

3◦ ) En déduire la relation liant α, P, P0 et K lorsque l’équilibre de la réaction est réalisé.


4◦ ) Application. 18,4 g de N2 O4 sont placés dans un récipient vide de 6 ` à 27◦ C. La pression
d’équilibre est de 1 atm. On prendra R = 25/3 SI.
Calculer le taux de dissociation de N2 O4 à cette température ainsi que la valeur correspon-
dante de K.

Christian Carimalo 134 TD de Thermodynamique - L3


Corrigé TD13

I - Réactions chimiques entre gaz parfaits

1◦ ) N1 = ν1 (1 − ε), N2 = ν2 (1 − ε), N4 = εν4 .


Nk
2◦ ) µk = gk (T, P ) + RT ln xk , avec xk = .
N
3◦ ) dGT,P = dε [ν3 µ3 + ν4 µ4 − ν1 µ1 − ν2 µ2 ].
4◦ ) ∆G = ν3 g3 (T, P ) + ν4 g4 (T, P ) − ν1 g1 (T, P ) − ν2 g2 (T, P ).
∂G xν33 xν44 ∂G
   
= ∆G + RT ln ν1 ν2 . Pour ε → 0, x3 , x4 → 0 et → −∞ ; pour
∂εT,P x x ∂ε T,P
1 2
∂G ∂G
  
ε → 1, x1 , x2 → 0 et → +∞ ; pour ε = εe , = 0.
∂ε T,P ∂ε T,P
5◦ )

G( ε )

ε
0 εe 1

Figure 20 – Tracé (très) qualitatif de G(ε)


νk
A noter 7 que pour ε = 1/2, on a xk = où Σ = ν1 + ν2 + ν3 + ν4 : les constituants sont
Σ
dans la réaction en proportion de leurs coefficients stœchiométriques respectifs. La pente de
la courbe de G en ce point donne une indication sur la position de l’état du mélange vis-à-vis
du point d’équilibre. En effet, si la pente est positive, le point d’équilibre se trouve vers la
gauche, ce qui signifie que lorsque les constituants sont mélangés dans lesdites proportions,
la réaction aura tendance à s’effectuer vers la gauche, avec augmentation des proportions de
A1 et A2 . Si la pente est négative, le point d’équilibre se trouve vers la droite, et la réaction
s’effectuera plutôt vers la droite, avec augmentation des constituants A3 et A4 .
x ν3 x ν4
!
∂2G d
Calculons la dérivée seconde = RT ln K où K = 3ν1 4ν2 . Il vient
∂ε2 T,P
dε x1 x2

7. Voir M. W. Zemansky, “Heat and Thermodynamics”, 4ème ed., McGraw-Hill, 1957, p442.

Christian Carimalo 135 TD de Thermodynamique - L3


" #
d 1 ν12 ν2 ν2 ν2
[ln K] = + 2 + 3 + 4 − (∆ν)2 , où ∆ν = ν4 + ν3 − ν1 − ν2
dε n x1 x2 x3 x4

d (ν4 + ν3 )(ν1 + ν2 )
En fonction de ε, cette dérivée s’exprime comme ln K = > 0. On en
dε nε(1 − ε)
déduit que la fonction K(ε) (avec 0 < ε < 1) est strictement croissante, et par suite :
∂G
 
• que l’équation = 0 a une seule solution ε = εe ;
∂ε T,P
!
∂2G
• que > 0, et que cette solution correspond bien à un minimum de l’enthalpie
∂ε2 T,P
libre G(ε).
∆G
 
6◦ ) a) K = Kc = exp − .
RT
∂ ln Kc 1 ∂ ∆H
   
b) i) =− [∆G/T ] = où ∆H = ν3 h3 (T, P ) + ν4 h4 (T, P ) −
∂T P R ∂T P RT 2
ν1 h1 (T, P ) − ν2 h2 (T, P ) est la variation
 d’enthalpie,
 qui est aussi la chaleur de réaction
∂ ln Kc ∂εe d

(équation de van’t Hoff). Or, = [ln Kc ] et, comme il a été montré
∂T P ∂T P dεe
d ∂εe

plus haut, [ln Kc ] > 0. Cela signifie que et ∆H sont de même signe. Par
dεe ∂T P
conséquent, une élévation de température à pression constante provoque un déplacement
de l’équilibre dans le sens où la transformation est endothermique (dans le sens de gauche à
droite si ∆H > 0, dans l’autre sens si ∆H < 0) : c’est la loi de van’t Hoff (loi de modération).
∂ ln Kc ∆V RT
 
ii) = − où ∆V = ν4 v4 + ν3 v3 − ν1 v1 − ν2 v2 = ∆ν . On en tire
∂P T RT P
∂εe d ∆ν ∂εe
  
[ln Kc ] = − : et ∆V sont de signes opposés. Par suite, une aug-
∂P T dεe P ∂P T
mentation de pression à température constante provoque un déplacement de l’équilibre dans
le sens d’une diminution de volume : c’est la loi de Le Châtelier (qui est aussi une loi de
modération).

xxxxxxxxxxxxxxxxxxxxxxxxxxxx

II - Synthèse de l’ammoniac

Pi
1◦ ) µi = gi (T, Pi ) = gi (T, P0 ) + RT ln .
P0
2◦ ) dGT,P = dλ [2µ3 − µ1 − 3µ2 ]
∆G0
3◦ ) dGT,P = dλ ∆G0 (T ) + RT ln Kp ; A l’équilibre, dGT,P = 0, soit ln Kp = −
 
RT
4◦ ) On part d’une mole d’azote et de trois moles d’hydrogène et on opère sous une pression
totale constante et égale à 1 atm. et à température constante T0 = 723 K.
60, 6 × 1000
 
a) Kp = exp − = 4, 2 10−5 .
8, 32 × 723

Christian Carimalo 136 TD de Thermodynamique - L3


ni
b) n3 = 2λ, n1 = 1 − λ, n2 = 3(1 − λ), ntot = n = 4 − 2λ ; Pi = P0 (car Ptot = P0 ),
n
P2 P2 n2 n2 4λ2 (4 − 2λ)2 16 (1 − x2 )2
donc Kp = 3 03 = 3 3 = . Posant x = 1 − λ, il vient K p = ,
P1 P2 n1 n2 27(1 − λ)4 27 x4
1 27Kp √ −3 √
soit x = q √ avec a = 16 . Comme a ' 8 10  1, on obtient x ' 1 − a/2,
1+ a

soit λ ' a/2 ' 4 10−3 . On obtient donc environ 2λ ' 8 10−3 mole(s) de N H3 .
Le rendement r de la réaction est le rapport (nombre de mole d’ammoniac obtenu)/(nombre
de moles d’ammoniac pour une réaction complète). On a ici nN2 + nH2 = 4(1 − λ) et la
réaction serait complète pour λ = 1, auquel cas nN H3 = 2. Le rendement vaut donc ici
0, 4 % seulement.
n23 n2 P02 16 λ2 (2 − λ)2 P02
5◦ ) On veut ici un rendement de 1/2 = 50%. On a Kp = = =
s n1 n32 P 2 27 (1 − λ)4 P 2
16 P02 16
pour λ = 1/2. On obtient donc P = P0 = 356 atm.
3 P2 3Kp

xxxxxxxxxxxxxxxxxxxxxxxxxxxx

III - Dissociation de N2 O4

1◦ ) La réaction est effectuée à température et volume constants : dans cette situation,


l’énergie libre F du mélange des deux gaz est le potentiel thermodynamique approprié.
X
2◦ ) dFT,V = µi dni . Avec na = n0 (1 − α) et nb = 2n0 α, il vient
i
∂F
 
= n0 [2µb − µa ]
∂α T,V

Pi ni ∂F
 
3◦ ) µi = gi (T, P0 )+RT ln , avec Pi = P , n = na +nb = n0 (1+α). D’où =
" #
P 0 n ∂α T,V

0 Pb2 K Pb2 P n2b P 4α2


n0 ∆G + RT ln = n0 RT ln , en posant K = = = .A
Pa P0 K Pa P0 P0 na n P0 1 − α 2
∂F P 4α2
 
l’équilibre, = 0, ce qui conduit à la relation = K.
∂α T,V P0 1 − α 2
18, 4 PV
4◦ ) MN2 O4 = 92 g, donc n0 = = 0, 2. On a n = n0 (1 + α) = , d’où 1 + α =
92 RT
5
10 × 6 10 −3 6 1 1
= , et α = . On en déduit K = .
0, 2 × (25/3) × 300 5 5 6
xxxxxxxxxxxxxxxxxxxxxxxxxxxx

Christian Carimalo 137 TD de Thermodynamique - L3


TD14

- Transferts de chaleur, loi de Fourier, convection -

I - Conduction de la chaleur, production d’entropie


On considère un corps solide indéformable, homogène, de conductibilité thermique λ supposée
indépendante de la température, de volume V limité par une surface Σ. Ce corps est le siège
de phénomènes thermiques irréversibles. On admet qu’à chaque instant t et en chaque point
(x, y, z) du corps on peut défimir une température locale T (x, y, z, t). On note u(x, y, z, t)
et s(x, y, z, t) respectivement l’énergie interne et l’entropie par unité de volume au voisinage
−→
dudit point. On note U l’énergie interne du corps, JQ le vecteur densité de courant de chaleur.
1◦ ) Etablir l’équation intégrale de conservation de l’énergie pour le corps solide. En déduire
une expression locale de cette conservation (équation (1)).
2◦ ) En utilisant cette équation (1), montrer que l’équation locale pour l’entropie s’écrit
−→
∂s JQ 1 −→ −→
+ div = − 2 JQ · grad T = θ (2)
∂t T T

On identifiera le terme correspondant à la création d’entropie par unité de temps et par unité
de volume.
3◦ ) Rappeler l’hypothèse de Fourier sur la conduction de la chaleur et récrire les équations
(1) et (2) dans ce cadre et en régime permanent.

II - Une source de chaleur disposée dans un mur délivre uniformément une puissance ther-
mique de 1 kW m−3 . A une date donnée t, la température en un point d’abscisse x du mur est
T (x, t) = 900 − 300 x − 50 x2 , en ◦ C. Le mur a une épaisseur de 1m et une surface de 10 m2 ,
sa masse volumique est de 1600 kg m−3 , sa conductivité thermique est de 40 W m−1 K−1
et sa chaleur massique 4000 J kg−1 K−1 .
1◦ ) Quels sont les flux thermiques Φ0 et Φ1 entrant dans le mur x = 0m et sortant du mur
x = 1m, respectivement ?
2◦ ) Quelle est la puissance thermique reçue par le mur ?
∂T
3◦ ) Quelle est la vitesse de variation (x, t) ?
∂t

III - Double vitrage en régime permanent


On veut comparer l’isolation thermique obtenue par un simple vitrage d’épaisseur e, confi-
guration A, à celle, configuration B, résultant d’un double vitrage constitué de deux vitres
d’épaisseur e/2 séparés par une couche d’air d’épaisseur e/2 (voir figure 21). Dans les deux
cas, on se placera en régime permanent, les faces extrêmes étant maintenues aux températures
T1 et T2 < T1 respectivement. Le verre et l’air ont des condictivités thermiques λ1 et λ2 < λ1 ,
respectivement. On admettra que le vecteur densité de courant de chaleur est, dans tous les

Christian Carimalo 138 TD de Thermodynamique - L3


cas, parallèle à l’axe Ox lui-même perpendiculaire au vitrage. Les surfaces perpendiculaires
à Ox considérées seront choisies d’aire unité.

T1 T2 T1 Ti Tf T2

0 e 0 e/2 e 3e/2 x

−A− −B−

Figure 21 – Isolation thermique : -A- par simple vitrage ; -B- par double vitrage

A/ Configuration A : simple vitrage


−→
1◦ ) Déterminer T (x) à l’intérieur de la vitre. En déduire le vecteur densité de courant JQ
et le flux de chaleur ΦA traversant une section droite unité perpendiculaire à Ox. Définir la
résistance thermique de la vitre simple.
2◦ ) Déterminer, en fonction de λ1 , e, T1 et T2 , l’entropie créée par unité de temps dans le
volume de la vitre de section droite unité.

B/Configuration B : double vitrage


1◦ ) Déterminer la résistance thermique RT du double vitrage pour une section droite unité.
2◦ ) En déduire le flux de chaleur ΦB traversant une section droite unité du double vitrage,
uniquement en fonction de λ1 , λ2 , e, T1 et T2 . Le comparer à ΦA . Quelle est la configuration
la plus isolante ?

IV - Production d’entropie par transfert de chaleur


Soient deux corps solides indéformables A1 et A2 de même capacités calorifiques C et dont
leur très grande conductibilté thermique assure que leurs températures restent toujours uni-
formes. Ils sont reliés par une barre cylindrique indéformable, homogène, de résistance ther-
mique RT et de capacité calorifique négligeable. A l’instant initial (t = 0), les corps A1
et A2 , tout d’abord isolés l’un de l’autre et portés respectivement aux températures T01 et
T02 < T01 , sont reliés par la barre. L’ensemble comprenant des deux corps et la barre est
isolé du monde extérieur.
1◦ ) Quelle est la température d’équilibre des deux corps ?
2◦ ) En admettant que les résultats concernant les transferts de chaleur en régime permanent
sont valables en régime lentement variable, montrer que la loi d’évolution de la température
T1 (t) du corps A1 est

Christian Carimalo 139 TD de Thermodynamique - L3


T01 + T02 T01 − T02 2t
 
T1 (t) = + exp −
2 2 RT C

3◦ ) Calculer de même T2 (t), température de A2 .


4◦ ) Calculer la variation d’entropie du système des deux corps et de la barre entre l’état
initial à t = 0 et l’état d’équilibre final. Commenter son signe.
5◦ ) Calculer à chaque instant la vitesse de production de l’entropie en fonction de RT , T1 (t)
et T2 (t).

V - Association de deux barreaux cylindriques

T1 λ1 Tc λ2 T2

0 L 2L

Figure 22 – Association de deux barreaux cylindriques

On associe deux barreaux cylindriques de même longueur L et de même section droite s


constitués de deux matériaux différents de conductibilités thermiques λ1 et λ2 respectivement
(figure 22). Les températures T1 et T2 < T1 des faces qui ne sont pas en contact sont
maintenues constantes. La température de la surface de contact des deux barreaux est notée
Tc . Les surfaces latérales des barreaux sont isolées du monde extérieur.
1◦ ) Exprimer les résistances thermiques respectives R1 et R2 des deux barreaux et la résistance
thermique totale RT de cette association.
2◦ ) Montrer qu’en régime permanent la température Tc peut s’exprimer sous la forme Tc =
T1 + K(T2 − T1 ) et exprimer K en fonction de λ1 et λ2 .
3◦ ) Application. Lorsqu’on met sa main en contact avec une plaque de bois ou d’acier, on
ressent une impression immédiate de froid dans un cas et de chaud dans l’autre. Interpréter
à l’aide de la question 2◦ ) et des données suivantes : T1 = 310 K, T2 = 290 K ; matériau 1
(corps humain) λ1 = 0, 5 SI ; matériau 2 : λ2 = 0, 25 SI (bois) , λ2 = 16 SI (acier).

VI - Association de trois barreaux cylindriques


On associe trois barreaux cylindriques de même longueur L et de même section droite s,
constitués de trois matériaux différents de conductivités thermiques respectives λ1 , λ2 , λ3
(figure 23). La surface latérale des barreaux est isolée du monde extérieur. Les surfaces
extrêmes de l’ensemble sont aux températures constantes T1 et T2 < T1 respectivement. On
étudie le régime permanent du transfert de chaleur. Les températures des surfaces de contact
sont alors T10 et T20 (voir figure 23).

Christian Carimalo 140 TD de Thermodynamique - L3


1◦ ) Exprimer T10 et T20 en fonction λ1 , λ2 , λ3 , T1 et T2 .

T1 λ1 T’1 λ2 T’2 λ3 T2

x
0 L 2L 3L

Figure 23 – Association de trois barreaux cylindriques

2◦ ) On donne : λ1 = 208 W m−1 K−1 (aluminium), λ3 = 380 W m−1 K−1 (cuivre). Le


second barreau est en acier. On trouve T10 − T20 = 74, 9 K. Calculer λ2 . On donne T1 = 373
K, T2 = 273 K.

3◦ ) On donne L = 5 cm. Calculer le flux thermique par unité de section droite traversant
l’ensemble.

4◦ ) Représenter T (x).

VII - Isolation thermique d’un garage

On veut étudier l’influence de l’isolation thermique sur la puissance nécessaire au chauffage


d’un garage. Ce dernier est assimilé à un parallélépipède rectangle de longueur L, de largeur
` et de hauteur H. La porte d’entrée occupe la totalité d’une des petites faces. Sur l’une des
grandes faces verticales se trouve une fenêtre de surface S. On donne L = 6 m, ` = 3 m,
H = 2, 2 m, S = 1 m2 .

A/Isolation des murs et du plafond

Les murs et le plafond sont faits de briques pleines d’épaisseur e = 15 cm et de conductivité


thermique λ1 = 1, 16 W m−1 K−1 . Un chauffage maintient, en régime permanent, une
température Ti = 292 K à l’intérieur du garage quand la température extérieure est Te = 277
K.

1◦ ) a) Calculer la valeur de la résistance thermique de 1 m2 de mur de brique.

b) En déduire la puissance thermique (ou flux thermique) traversant 1 m2 de mur.

2◦ ) On veut réduire cette perte thermique d’un facteur 10 en isolant l’intérieur du garage
au moyen d’une couche de polymère expansé d’épaisseur e0 et de conductivité thermique
λ2 = 4, 06 10−2 W m−1 K−1 que l’on fixe sur la totalité des murs et du plafond.

Calculer la valeur de l’épaisseur e0 à donner à cet isolant.

B/ Isolation de la fenêtre

La vitre de la fenêtre, de surface S, est constituée d’un verre d’épaisseur d = 2 mm et de


conductivité thermique λ3 = 1, 16 W m−1 K−1 .

Christian Carimalo 141 TD de Thermodynamique - L3


3◦ ) Calculer la puissance thermique Pv perdue par conduction à travers la vitre. Que pensez-
vous de l’ordre de grandeur du résultat ?
4◦ ) En réalité, dans l’échange de chaleur entre l’air et le verre interviennent les phénomènes
de convection et de rayonnement.
On rappelle que si p est la puissance thermique échangée par convection et rayonnement à
travers une surface Σ séparant deux régions présentant un écart de température ∆T , on a

p = h ∆T Σ

(loi de Newton) où h est le coefficient d’échange thermique global. On prendra ici h = 9, 29
W m2 K−1 .
a) Calculer la puissance thermique Pv0 perdue à travers la vitre en tenant compte de ces
phénomènes.
b) Calculer la température T1 sur la face interne de la vitre et la température T2 sur sa face
externe.
5◦ ) On effectue un double vitrage de la fenêtre en rajoutant sur son côté intérieur une
seconde vitre de même surface que la première et d’épaisseur double. Entre les deux vitres
est emprisonnée une couche d’air sec d’épaisseur D = 35 mm et de conductivité thermique
λ4 = 2, 44 10−2 W m−1 K−1 et à l’intérieur de laquelle les phénomènes de convection et de
rayonnement sont négligeables.
Calculer la nouvelle puissance Pv00 perdue à travers ce double vitrage ainsi que les nouvelles
températures T10 et T20 des faces interne et externe.

C/ Evaluation de la puissance de chauffage


On donne ci-dessous les résistances thermiques des éléments du garage après isolation :
• murs et plafond (hors porte et fenêtre) : 1,3 K W−1 pour 1 m2 ;
• fenêtre avec son encadrement (surface de 1 m2 ) : 1,6 K W−1 ;
• porte avec son encadrement : 8 10−2 K W−1 (pour le total de 6,6 m2 ) ;
• les pertes thermiques par le sol sont négligeables grâce à l’existence d’un vide sanitaire.
6◦ ) Justifier que toutes les résistances thermiques sont disposées en parallèle. Calculer la va-
leur de la résistance thermique totale RT et en déduire la puissance thermique Pch nécessaire
au chauffage du garage dans les conditions de température indiquées plus haut.
7◦ ) La chaleur massique de l’air est c = 1 J g−1 K−1 et sa masse volumique est ρ = 1, 3
g `−1 . A la date t = 0, la température intérieure du garage est T0 = Te . On branche alors
un radiateur électrique délivrant la puissance Pr = 1 kW. La température extérieure reste
constamment égale à Te .
a) Démontrer que la température intérieure T du garage varie en fonction du temps t selon
la loi

t
  
T (t) = Te + RT Pr 1 − exp −
ρ ` L H c RT

Christian Carimalo 142 TD de Thermodynamique - L3


b) Au bout combien de temps la température intérieure du garage atteint-elle la valeur
Ti = 292 K ?

VIII - Régulation thermique d’un mammifère


On modélise très schématiquement un mammifère par un corps solide indéformable, ho-
mogène, en forme de boule de centre O et de rayon a. Son métabolisme est assimilé à une
production de chaleur en quantité qp par unité de volume et par unité de temps, constante
dans la boule de rayon a, nulle à l’extérieur de celle-ci. Cette énergie est transmise partielle-
ment au milieu extérieur de conductivité thermique λ. On cherche à déterminer, en régime
permanent, la température de surface du mammifère.
−→
On note JQ le vecteur densité de courant de chaleur. Comme le probème est à symétrie
−→ −→
sphérique, ce vecteur est radial. On a : JQ = J(r) er où r est la distance vis-à-vis du centre
−→
O de la boule, er le vecteur unitaire radial. La température locale T (r) ne dépend que de r
−→ dT −→
et grad T = (r) er .
dr
1◦ ) En faisant un bilan énergétique pour une boule de centre O de rayon r > a, montrer que

a3
J(r) = qp
3r2

2◦ ) Rappeler l’hypothèse de Fourier valable pour le milieu extérieur. En déduire l’équation


différentielle vérifiée par la température T (r) pour r > a.
3◦ ) Déterminer alors T (r) pour r > a, sachant que pour r → ∞, la température tend vers
une constante T0 .
4◦ ) En déduire la température Tc pour r = a, c’est-à-dire la température cutanée du mam-
mifère.
5◦ ) Représenter Tc en fonction de λ, a étant constant, puis en fonction de a, λ restant
constant, toutes les autres grandeurs restant égales par ailleurs. Commenter les courbes
obtenues.
6◦ ) a) Déterminer quelle doit être la valeur de qp pour assurer une température de surface
constante égale à 30◦ C, selon que le mammifère se trouve dans l’eau (λeau = 500 W m−1
K−1 ) ou dans l’air (λair = 5 W m−1 K−1 ). On donne T0 = 293 K, a = 25 cm.
b) Conclure quant au fait qu’on ne rencontre pas de petit mammifère marin dans la nature.

IX - Isolation d’une canalisation


Pour déterminer le flux thermique à travers une paroi, il faut tenir compte à la fois du
phénomène de conduction thermique dans le matériau dont elle est constituée et des phénomè-
nes de transport de chaleur par convection dans les fluides en mouvement. On se propose
dans ce problème d’étudier les échanges thermiques avec l’air extérieur d’un fluide circulant
dans une canalisation cylindrique.
A/ Un fluide circule dans une canalisation d’axe z 0 z, de rayon intérieur a1 et de rayon
extérieur a2 . La longueur L de la canalisation est supposée très grande devant a2 , ce qui
permet d’ignorer tout effet de bord.

Christian Carimalo 143 TD de Thermodynamique - L3


En régime indépendant du temps, la température du fluide sur la surface interne de la canali-
sation est constante et égale à T1 , tandis que la température de l’air au contact de la surface
extérieure est constante et égale à T2 < T1 . On note λ la conductivité thermique, constante,
du matériau de la canalisation. On étudie dans cette partie le transfert de chaleur à travers
la paroi.
1◦ ) a) Etablir l’équation locale de conservation de l’énergie dans la paroi. En précisant les
hypothèses, déduire l’équation locale de diffusion de la chaleur.
b) Un point M est repéré par ses coordonnées cylindriques (r, θ, z). On note T la température
locale en M . En utilisant des arguments de symétrie, montrer que ladite équation s’écrit

1 ∂ ∂T 1 ∂2T ∂2T 1 d dT
   
∆T = r + 2 2
+ 2
= r =0
r ∂r ∂r r ∂θ ∂z r dr dr

2◦ ) Résoudre cette équation et en déduire T (r), compte tenu des conditions aux limites.
3◦ ) On note Rc la résistance thermique de la paroi.
a) Rappeler son expression en fonction du flux de chaleur à travers la paroi et de la différence
de température T1 − T2 .
1 a2
b) Montrer que Rc = ln
2πλL a1

B/ On constate qu’en régime permanent, la température T0 du fluide au voisinage de l’axe


z 0 z est différente de T1 . Entre le cœur du fluide et la surface interne de la canalisation existe
un flux de chaleur φ de nature convective dont l’expression est donnée par

φ = hf S1 (T0 − T1 )

où la constante positive hf est le coefficient de convection du fluide, S1 l’aire de la surface


interne de la canalisation. De même, à l’extérieur de la canalisation, la température T00 de
l’air ambiant est différente de T2 , température de la surface extérieure de la canalisation, et
l’on écrit le flux de chaleur convectif correspondant sous la forme

φ = ha S2 (T2 − T00 )

où la constante ha est le coefficient de convection de l’air, S2 l’aire de la surface extérieure


de la canalisation.
1◦ ) Que peut-on dire du flux de chaleur φ s’écoulant depuis le cœur du fluide à la température
T0 vers l’air ambiant à la température T00 ?
2◦ ) En tenant compte à la fois de la convection dans les fluides et de la conduction de la
chaleur dans la paroi de la canalisation, déterminer la résistance thermique effective de la
T0 − T00
canalisation, définie par R = .
φ

C/ On veut isoler thermiquement la canalisation à l’aide d’un matériau de conductivité


thermique λ0 que l’on applique sur une épaisseur x. La surface extérieure de la canalisation
ainsi calorifugée a maintenant pour rayon a3 = a2 + x.

Christian Carimalo 144 TD de Thermodynamique - L3


1◦ ) Donner la nouvelle expression R0 de la résistance thermique effective en fonction de
R, λ0 , ha , a2 , L et x.
2◦ ) a) A quelle condition a-t-on R0 > R ?
b) On suppose x  a2 . Comment choisir le matériau isolant pour que la condition précédente
soit réalisée ?

D/ Applications numériques
1◦ ) Calculer Rc et R dans le cas où :
λ = 0, 9 W m−1 K−1 ; λ0 = 0, 03 W m−1 K−1 ; T0 = 310 K ; T1 = 304 K ; T2 = 290 K ;
T00 = 275 K ; a1 = 2 cm ; a2 = 2, 4 cm ; L = 2 m.
2◦ ) Le matériau isolant de conductivité thermique λ0 satisfait-il la condition de la question
C/ 2◦ ) b) ?

X - Bouclier thermique d’un vaisseau spatial


Lors de la rentrée d’un véhicule spatial dans l’atmosphère terrestre, l’air atmosphérique de-
vient un plasma très chaud (P) lorsqu’il rentre en contact avec la paroi du véhicule. Cette
paroi sert de bouclier thermique (B) au véhicule en absorbant le flux d’énergie.
On se propose d’étudier le comportement thermique de ce bouclier au cours de cette phase
de rentrée du véhicule dans l’atmosphère. On suppose que le transfert de chaleur entre (P) et
−→
(B) est unidimensionnel, selon un axe Ox. Notant ex le vecteur unitaire de cet axe, le vecteur
−→ −→
densité de courant de chaleur dans (B) sera donc pris sous la forme JQ = J(x, t) ex , et l’on
posera J0 (t) = J(0, t). On note respectivement ρ, C, λ et D, la masse volumique, la capacité
calorifique massique, la conductivité thermique, et le coefficient de diffusion thermique du
bouclier (B), ces grandeurs étant supposées constantes.
La date t = 0 correspond au moment où le flux thermique commence à frapper la paroi, la
température de (B) étant alors uniforme et égale à T0 .
On suppose que l’épaisseur de (B) est suffisante pour qu’on puisse la considérer comme
infinie. Soit L(t) l’épaisseur du matériau de (B) au-delà de laquelle on peut considérer que
J(x, t) ' 0.
A/
1◦ ) Etablir l’équation de diffusion de la chaleur dans (B) à partir de l’équation locale de
conservation de l’énergie et de l’hypothèse de Fourier. Quelle relation existe-t-il entre D, λ, ρ
et C ?
Z L(t) 
∂T J0 (t)

2◦ ) Montrer que dx =
0 ∂t x ρC
3◦ ) Démontrer qu’une distribution de température de la forme

J0 (t)
T (x, t) = T0 + (L(t) − x)2
2λL(t)

vérifie les conditions aux limites spatiales.


4◦ ) On considère la fonction

Christian Carimalo 145 TD de Thermodynamique - L3


Z L(t)
E(t) = ρ C [T (x, t) − T0 ] dx
0
Z t
a) Sachant que T (x, 0) = T0 , montrer que E(t) = J0 (t) dt. Que représente cette fonction ?
0

b) Montrer alors que


1/2 Z t 1/2
6D

L(t) = J0 (t) dt
J0 (t) 0

c) En déduire l’expression de la température T0 (t) à l’interface plasma-bouclier en fonction


de T0 , J0 (t), D, ρ et λ.
5◦ ) Les conditions du vol du véhicule spatial permettent de prévoir que J0 (t) augmente à
partir de t = 0, passe par un maximum puis décroı̂t. Démontrer que T0 (t) passe aussi par un
maximum correspondant à la phase décroissante de J0 (t).
6◦ ) Application numérique
On donne J0 (t) = −0, 5 t2 + 1000 t en W m−2 , λ = 25 W m−1 K−1 , ρ = 3170 kg m−3 ,
C = 1270 J kg−1 K−1 , T0 = 300 K.
La température T0 (t) passe par un maximum pour t = tM = 1368 s. Calculer T0 (tM ) et
L(tM ).

XI - Regel
Une barre rigide indéformable, de masse négligeable, en forme de parrallélépidède rectangle
de section droite ab, est placé sur un cube de glace d’arête c, à la température T0 = 273 K
et à la pression atmosphérique P0 = 1 atm. Deux masses M sont accrochées aux extrêmités
de la barre de part et d’autre du bloc de glace (figure 24).

b a

M M

Figure 24 – Expérience de regel

On constate que la glace fond en dessous de la barre et que la barre s’y enfonce. L’eau de
fusion passe au-dessus de la barre et se solidifie à nouveau au-dessus de celle-ci en libérant

Christian Carimalo 146 TD de Thermodynamique - L3


une quantité de chaleur. La chaleur ainsi libérée est transportée par conduction, suivant
la verticale descendante, au travers de la barre d’épaisseur a et une mince couche d’eau
d’épaisseur constante e située sous la barre.

glace
o

a barre

e couche d’eau

glace z

Figure 25 – Mécanisme de regel

Cette quantité de chaleur est ensuite absorbée par la glace sous cette couche (figure 25).
La barre traverse ainsi lentement le bloc de glace, la glace se reformant après son passage
(regel).
1◦ ) Soient P1 = P0 + ∆P et T1 = T0 + ∆T la pression et la température d’équilibre de la
glace sous la barre. On note L la chaleur massique de fusion de la glace, v` et vs les volumes
massiques de l’eau liquide et de la glace, g l’accélération de la pesanteur. On supposera
que les variations de température et de pression d’équilibre de part et d’autre de la barre
sont suffisamment faibles pour que L, v` , vs puissent être considérés comme constants, leurs
valeurs étant celles de l’état (T0 , P0 ).
a) Expliciter ∆P en fonction de M, b, c et g.
b) En utilisant la relation de Clapeyron, exprimer ∆T en fonction de L, v` , vs , T0 , M, b, c
et g.
2◦ ) Chaque déplacement infinitésimal dz de la barre s’accompagne de la fusion d’une quantité
infinitésimale dm de glace sous la barre. Quelle est la quantité de chaleur d¯Q nécessaire à
cette fusion ?
3◦ ) Cette quantité de chaleur résulte de la solidification de l’eau au-dessus de la barre. On
admet que la chaleur est intégralement transmise vers la glace au-dessous de la barre et que
le flux de chaleur correspondant (quantité de chaleur transmise par unité de temps) est donné
d̄ Q
par = γ ∆T où γ est la conductance thermique de l’ensemble “barre + couche d’eau”,
dt
que l’on supposera constante. On choisira les conventions de signe adéquat pour utiliser cette
expression du flux de chaleur.
dm
a) Exprimer en fonction de ∆T, L et γ.
dt
b) En déduire la vitesse de déplacement v de la barre.
4◦ ) Application numérique. On donne M = 10 kg, g = 10 ms−2 , vs = 1, 09 cm3 g−1 ,
a = b = 1 mm, c = 10 cm, L = 333 J g−1 , γ = 3 W K−1 .
Calculer numériquement en cm/h la vitesse de déplacement v de la barre.

Christian Carimalo 147 TD de Thermodynamique - L3


Corrigé TD14

I - Conduction de la chaleur, production d’entropie

dU dQ dQ −→ −→
Z Z
1◦ ) = , avec U = u(x, y, z, t)dv (dv = dxdydz), = − JQ · dS. D’où
dt dtZ −→ −→ V dt S
∂u −→
Z Z
dv = − JQ · dS= − div JQ dv, soit, localement,
V ∂t S V
∂u −→
+ div JQ = 0
∂t
∂u ∂s ∂s 1 −→ ∂s
2◦ ) Avec = T , l’équation précédente se récrit comme + div JQ = +
−→
∂t ∂t −→
∂t T ∂t
JQ −→ −→ 1 ∂s JQ 1 −→ −→
div − JQ · grad = + div + 2 JQ · grad T , soit
T T ∂t T T
−→
∂s JQ 1 −→ −→
+ div = − 2 JQ · grad T = θ
∂t T T

Le terme θ correspond à la création d’entropie par unité de temps et par unité de volume.
−→ −→
3◦ ) Hypothèse de Fourier : JQ = −λ grad T . En régime permanent (stationnaire) : ∆T =
−→
∂2T ∂2T ∂2T (grad T )2
+ + = 0 et θ = λ > 0.
∂x2 ∂y 2 ∂z 2 T2
xxxxxxxxxxxxxxxxxxxxxxxxxxxx

II -

∂T ∂T
 
1◦ ) J = −λ , Φ = JΣ ; d’où Φ0 = −λΣ = 40 × 10 × 300 = 120 kW,
∂x ∂x x=0
∂T
 
Φ1 = −λΣ = 160 kW.
∂x x=1m
2◦ ) Volume du mur : V = 10 m3 . Puissance thermique reçue par le mur = 1×10+120−160 =
−30 kW.
" #
∂u −→ ∂T ∂2T ∂T 1 ∂2T
3◦ ) + div JQ = P = ρc − λ 2 , d’où = P +λ 2
∂t ∂t ∂x ∂t ρc ∂x
1
= [1000 − 100 × 40] = −4, 7 10−4 K/s.
1600 × 4000
xxxxxxxxxxxxxxxxxxxxxxxxxxxx

III - Double vitrage en régime permanent

d2 T
A/ 1◦ ) De = 0 on tire T (x) = ax + b. Compte tenu des conditions aux limites, on
dx2
T1 − T2 dT λ1
trouve T (x) = T1 − x et J = −λ1 = (T1 − T2 ) ; le flux de chaleur traversant
e dx e
une section droite unité perpendiculaire à Ox est ΦA = J et le flux traversant une section

Christian Carimalo 148 TD de Thermodynamique - L3


λ1 Σ
droite de surface Σ est φA = JS = (T1 − T2 ). La résistance thermique correspondante
e
T1 − T2 e
est définie par = . La résistance thermique de la vitre simple de section droite
φA λ1 Σ
e
unité est donc RA = (en unité convenable).
λ1
−→
◦ (grad T )2
2 ) L’entropie créée par unité de temps et par unité de volume est θ = λ1 =
2 T2
λ1 dT dT d 1
  
= −λ1 . L’entropie créée par unité de temps dans le volume de la vitre
T 2 dx dx dx T Z x=e
dT d 1 λ1 1 1
   
de section droite unité est donc ∆S = −λ1 dx = (T1 −T2 ) − =
x=0 dx dx T e T2 T1
λ1 (T1 − T2 )2 (T1 − T2 )2
= .
e T1 T2 RA T1 T2

e e
B/ 1◦ ) T1 − Ti = R1 ΦB , Ti − Tf = R2 ΦB , Tf − T2 = R3 ΦB , avec R1 = , R2 = ,
2λ1 2λ2
R3 = R1 . D’où T1 − T2 = RT ΦB , avec RT = R1 + R2 + R3 = RA + R2 .
T1 − T2 ΦB RA 1
2◦ ) Φ B = ; = = < 1. La configuration (B) est bien sûr la plus
RT ΦA RT λ1
1+
2λ2
isolante.

xxxxxxxxxxxxxxxxxxxxxxxxxxxx

IV - Production d’entropie par transfert de chaleur

1◦ ) ∆Utot = ∆U1 + ∆U2 + ∆Ubarre = 0 (système isolé). La capacité calorifique de la barre


étant négligeable : ∆Ubarre ' 0. Comme ∆U1 = C(Tf − T01 ), ∆U2 = C(Tf − T02 ), on en
T01 + T02
déduit Tf = .
2
∂U1 ∂T1 T1 − T2 ∂U2 ∂T2
2◦ ) = C = −Φ = − , = C = Φ. On a donc T1 + T2 =
∂t ∂t RT ∂t ∂t
∂T1 T1 − T2 T1 − Tf
constante = 2Tf , et =− = −2 . Cette équation différentielle a pour
∂t  RT C RT C
2t
solution T1 −Tf = K exp − , la constante K devant être ajustée à la condition initiale
RT C
T01 − T02
T1 (0) = T01 . On trouve K = T01 − Tf = , d’où
2
T01 + T02 T01 − T02 2t
 
T1 (t) = + exp −
2 2 RT C

T01 + T02 T01 − T02 2t


 
3◦ ) T2 = 2Tf − T1 = − exp − .
2 2 RT C
4◦ ) et 5◦ ) La variation d’entropie de l’ensemble, ∆Stot = ∆S1 + ∆S2 + ∆Sbarre , peut tout
d’abord être calculée de façon classique comme suit. D’une part, la capacité calorifique de la
barre étant négligeable, ∆Sbarre peut être ignoré. D’autre part, faisant usage de la formule
dT Tf
dS = C , valable pour chacun des deux corps, on trouve ∆Sk = C ln , et finalement
T T0k
Tf2
∆Stot = C ln
T01 T02

Christian Carimalo 149 TD de Thermodynamique - L3


La transformation (adiabatique pour l’ensemble) est irréversible et l’on a bien ∆Stot > 0,
1
puisque Tf2 = (T01 + T02 )2 > T01 T02 .
4
D’un autre côté, c’est le transfert de chaleur au travers de la barre qui provoque cette variation
−→
(grad T )2
d’entropie, avec, au sein de cette barre, un taux de création d’entropie θ = λ , par
T2
unité de temps et par unité de volume, λ étant la conductivité thermique de la barre. On doit
donc pouvoir obtenir tout aussi bien la variation totale d’entropie de l’ensemble en intégrant
ce taux sur tout le volume de la barre et sur l’intervalle de temps [0, +∞[. En effectuant un
calcul similaire à celui de la question 2◦ ) de l’exercice III, on obtient

(T1 − T2 )2 (T01 − T02 )2


Z ∞ Z ∞
exp[−t/τ ]
∆Stot = dt = dt
0 RT T1 T2 RT 0 (T01 + T02 )2 (T01 − T02 )2
− exp[−t/τ ]
4 4
RT C
où τ = . Avec le changement de variable u = exp[−t/τ ], cette expression prend la
4
forme

(T01 − T02 )2
Z 1
du
∆Stot = C
4 0 (T01 + T02 )2 (T01 − T02 )2
− u
4 4
# 1
Tf2
"
(T01 + T02 )2 (T01 − T02 )2
= −C ln − u = C ln

4 4
0
T01 T02

Comme il se doit, on trouve le même résultat !

xxxxxxxxxxxxxxxxxxxxxxxxxxxx

V - Association de deux barreaux cylindriques

1 L L 1 1
 
1◦ ) Rk = , k = 1, 2 ; RT = R1 + R2 = + .
λk s s λ1 λ2
T1 − Tc
2◦ ) On procède comme pour les diviseurs de tension en Electrocinétique : =
T1 − T2
R1 R1 λ2
. D’où Tc = T1 + K(T2 − T1 ), avec K = = .
R1 + R2 R1 + R2 λ1 + λ2
1
3◦ ) Kbois = ; Kacier = 0, 97. La température intermédiaire Tc (celle du contact) est plus
3
proche de T1 dans le cas du bois et plus proche de T2 dans le cas de l’acier, d’où les ressentis
de chaud dans le premier cas et de froid dans le second.

xxxxxxxxxxxxxxxxxxxxxxxxxxxx

VI - Association de trois barreaux cylindriques

1 L R1 R3
1◦ ) Rk = , RT = R1 + R2 + R3 , T1 − T10 = (T1 − T2 ) , T20 − T2 = (T1 − T2 ) ,
λk s RT RT
λ2 λ3 λ1 λ2
d’où T10 = T1 − (T1 − T2 ) , T20 = T2 + (T1 − T2 ) .
λ1 λ2 + λ2 λ3 + λ3 λ1 λ1 λ2 + λ2 λ3 + λ3 λ1

Christian Carimalo 150 TD de Thermodynamique - L3


T1 − T2 RT 1 1 λ1 λ3 T1 − T2
   
2◦ ) 0 0 = = 1+λ2 + , d’où λ2 = − 1 . Numériquement,
T1 − T2 R2 λ1 λ2 λ1 + λ3 T10 − T20
on trouve λ2 = 45 W m−1 K−1 .
Φ λ2 0
3◦ ) = (T − T20 ) = 6, 75 104 W/m2 .
s L 1

4◦ ) Avec T1 − T10 = R1 Φ = , on trouve T10 = 356, 8 K (83, 8 ◦ C), puis T20 = 281, 9 K
λ1 s
(8, 9 ◦ C). La courbe représentant T (x) est une succession de segments de droites.

T(x)

T1
T’
1

T’
2 x
T2
L 2L 3L

Figure 26 – Représentation de T (x)

xxxxxxxxxxxxxxxxxxxxxxxxxxxx

VII - Isolation thermique d’un garage

e
A/ 1◦ ) a) Rmur = ' 0, 13 K/W.
λ
Ti − Te
b) P = Φ = = 116 W/m2 .
R
2◦ ) Φ0 = Φ/10, d’où la nouvelle résistance thermique nécessaire : R0 = 10Rmur = Rmur +
e0 λ0
Risolant , soit Risolant = 9Rmur = 0 . On en déduit e0 = 9e ' 4, 7 cm.
λ λ
Ti − Te d
B/ 3◦ ) Pv = avec Rv = . Numériquement, Pv = 8700 W. L’énormité de cette
Rv λ3 S
valeur jette le doute sur cette évaluation des pertes thermiques.

r Rv r

Ti T1 T2 Te

Figure 27 – Série de résistances thermiques à prendre en compte


pour l’évaluation des pertes à travers la fenêtre

4◦ ) a) La prise en compte des phénomènes de convection et de rayonnement au moyen de


la loi de Newton revient à considérer que l’échange thermique s’effectue à travers la série de

Christian Carimalo 151 TD de Thermodynamique - L3


S
résistances thermiques représentée à la figure (27), où r = . La résistance thermique de
h
Ti − Te λ3

0
l’ensemble est R = Rv + 2r et l’on a Pv = 0 = Pv / 1 + 2 . Numériquement,
R0 hd
0
Pv = Pv /126 = 69 W, valeur plus raisonnable.
r 2r
b) On a Ti − T1 = T2 − Te = (Ti − Te ) , avec = 125. On trouve Ti − T1 =
Rv + 2r Rv
T2 − Te = 7, 5 K. D’où T1 ' 284, 6 K, T2 ' 284, 4 K.
5◦ )

r 2Rv Ra Rv r

Ti T’1 T’2 Te

Figure 28 – Résistances thermiques intervenant dans l’isolation de la fenêtre

La résistance thermique totale à prendre en compte après isolation de la fenêtre est R00 =
D
3Rv + 2r + Ra où Ra = est la résistance thermique de la couche d’air entre les deux
λ4 S
vitres. On trouve maintenant R00 = 1, 65 K/W, et Pv00 = 9, 1 W, T10 ' 16 ◦ C, T10 ' 5 ◦ C.

C/ 6◦ ) Le lecteur se convaincra aisément que toutes les résistances thermiques à considérer


sont disposées en parallèle. La surface totale des murs et plafond, hors porte et fenêtre, est
1 50 1 100
de 50 m2 . On en déduit la conductance totale : = + + = 51, 6 W/K, soit
RT 1, 3 1, 6 8
Ti − Te
RT ' 2 10−2 K/W, puis Pch = = 774 W.
RT
7◦ ) a) La puissance délivrée par le radiateur sert à chauffer l’air à l’intérieur du garage tout en
compensant les pertes thermiques. Soit T la température de l’air du garage à la date t > 0.
Entre les dates t et t + dt, cette température évolue de T à T + dT . Pendant ce laps de
temps, l’énergie fournie par le radiateur est Pr dt, l’énergie emmagasinée par l’air du garage
est cρV dT , où V = `LH est le volume intérieur du garage, et les pertes thermiques sont
T − Te
Pf dt = dt . On exprime la conservation de l’énergie en écrivant Pr dt = cρV dT +Pf dt,
RT
d’où l’équation différentielle

T − Te dT y dy
Pr = + `LHρc , ou + =0
RT dt τ dt
en posant y = T − Te − Pr RT et τ = RT cρ`LH. Intégrant cette équation
 en  tenant compte
t
de la condition initiale y(0) = −RT Pr , on obtient y(t) = −RT Pr exp − , et
τ
t
  
T (t) = Te + RT Pr 1 − exp −
τ

Pc
 
b) t = −τ ln 1 − . Avec τ = 997, 7 s, on trouve t = 1484 s ' 25 mn.
Pr
xxxxxxxxxxxxxxxxxxxxxxxxxxxx

Christian Carimalo 152 TD de Thermodynamique - L3


VIII - Régulation thermique d’un mammifère

−→
1◦ ) On admet que le flux de JQ sortant à travers la sphère de centre O et de rayon r est
égal à la puissance thermique totale développée par le corps, ce qui se traduit par la relation
4πa3 a3
4πr2 J(r) = qp . D’où J(r) = qp 2 .
3 3r
dT dT 1 a3
2◦ ) et 3◦ ) J(r) = −λ , d’où = − qp 2 , équation dont l’intégration conduit à
dr dr λ 3r
a3
T (r) = T0 + qp , compte tenu de la condition T → T0 pour r → ∞.
3r
a2
4◦ ) Tc = T0 + qp .

5◦ ) Rédaction laissée au lecteur.
3λ(Tc − T0 )
6◦ ) a) qp = = 480 λ (en unité convenable). D’où
a2
pour l’eau : qp = 240000 W/m3 ; pour l’air : qp = 2400 W/m3 .
b) La valeur énorme de qp pour l’eau est rédhibitoire pour l’existence d’un petit mammifère
marin.

xxxxxxxxxxxxxxxxxxxxxxxxxxxx

IX - Isolation d’une canalisation

∂u −→ ∂u ∂T −→ −→
1◦ ) a) Voir le cours ; + div JQ = 0, = ρcv , JQ = −λ grad T , d’où l’équation de
∂t ∂t ∂t
∂T λ
diffusion de la chaleur : = ∆T .
∂t ρcv
∂T
b) En régime indépendant du temps, = 0. Par ailleurs, on a, d’une part, invariance par
∂t
rotation autour de l’axe de la canalisation, et, d’autre part, invariance par translation parallèle
à cet axe en admettant que la longueur L de la canalisation peut être considérée comme
infinie. Dans ces conditions, les dérivées partielles de T par rapport à θ et par rapport à z
sont nulles, et l’équation de diffusion devient

1 ∂ ∂T 1 ∂2T ∂2T 1 d dT
   
∆T = r + 2 2
+ = r =0
r ∂r ∂r r ∂θ ∂z 2 r dr dr

dT A r
 
2◦ )On en déduit = , puis T (r) = T1 +A ln , où l’on a tenu compte de la condition
dr r a1
a2
T (a1 ) = T1 . Considérant ensuite la condition T (a2 ) = T2 , on trouve A = −(T1 −T2 )/ ln[ ].
a1
T1 − T2 r
On obtient finalement T (r) = T1 − a2 ln a .
ln 1
a1
dT a2 1
3◦ ) a) et b) Φ = 2πrLJ(r) ; or, J(r) = −λ = λ(T1 − T2 )/ ln[ ] , d’où
dr a1 r
a2 T1 − T2 1 a2
Φ = 2πλL(T1 − T2 )/ ln[ ] = , et par suite, Rc = ln .
a1 Rc 2πλL a1

Christian Carimalo 153 TD de Thermodynamique - L3


B/ 1◦ ) De par la conservation de l’énergie, le flux de chaleur se conserve car il n’y a pas de
source ni de puit d’énergie supplémentaires en volume.
1 1
2◦ ) R = + Rc + .
hf S1 ha S2
1 a3 1 1 1 1
C/ 1◦ ) Risolant = 0
ln , et R0 = Rc + + Risolant + = Rc + + +
2πλ L a2 hf S1 ha S3 hf S1 ha S3
1 1 1 1 1 1 1 x
  
− + Risolant , soit R0 = R + − + 0
ln 1 + .
ha S 3 S 2 2πLha a2 + x a2 2πλ L a2
1 x 1 1 1 x
   
2◦ ) a) On a R0 > R si 0
ln 1 + > − = .
λ a2 ha a2 a2 + x ha a2 (a2 + x)
b) Pour x  a2 , la condition R0 > R devient λ0 < ha a2 , c’est-à-dire, une inégalité à
laquelle doit satisfaire λ0 , indépendamment de x, et qui montre que le matériau d’isolation
doit être choisi de façon appropriée selon la valeur de ha , celle-ci dépendant généralement
des conditions climatiques de l’air ambiant, lesquelles déterminent la nature des transferts de
chaleur qui s’y produisent.
D/ Les données numériques correspondent ici au cas d’une canalisation non isolée.
1 T0 − T1 1 T2 − T00 T1 − T2
1◦ ) Rc = 0, 016 K/W ; On a = , = , et Φ = . On en
hf S1 Φ ha S2 Φ Rc
1 1 T0 − T00
tire R = + Rc + = Rc = 0, 04 K/W.
hf S1 ha S2 T1 − T2
1 T2 − T00 T1 − T2 a2
Puis = Rc , soit ha a2 = λ 0 / ln = 4, 6 W m−1 K−1 .
ha S2 T1 − T2 T2 − T0 a1
2◦ ) Le matériau isolant de conductivité thermique λ0 satisfait la condition de la question C/
2◦ ) b) : λ0 < ha a2 .

xxxxxxxxxxxxxxxxxxxxxxxxxxxx

X - Bouclier thermique d’un vaisseau spatial

∂u −→ ∂u ∂T −→ −→
A/ 1◦ ) Voir le cours ; + div JQ = 0, = ρC , JQ = −λ grad T , d’où l’équation de
∂t ∂t ∂t
∂T λ
diffusion de la chaleur : = D∆T avec D = .
∂t ρC
!
∂2T ∂T L
Z L(t) Z L(t)
◦ ∂T D
2 ) dx = D dx = D = J0 (t) car J(x, t) = 0 si x ≥ L.
0 ∂t 0 ∂x2 ∂x 0 λ
Z L(t) 
∂T J0 (t)

D’où dx = .
0 ∂t x ρC
J0 (t)L(t)
3◦ ) T (x, 0) = T0 car J0 (t) = 0 si t ≤ 0 ; T (L(t), t) = T0 ; T (0, t) = T0 + ;

∂T J0 (t) ∂T ∂T
= [x − L], donc (L, t) = 0 et (0, t) = −λJ0 (t).
∂x λL ∂x ∂x
∂2T
Z L(t) Z t Z t Z L(t) Z t
∂T ∂T
4◦ ) a) E(t) = ρ C dx dt = λ dt dx = −λ dt (0, t), soit E(t) =
Z t 0 0 ∂t 0 0 ∂x2 0 ∂x
J0 (t) dt ; E(t) représente l’énergie transférée, par unité de surface.
0

Christian Carimalo 154 TD de Thermodynamique - L3


b) On utilise ici le modèle proposé au 3◦ ).
Z L(t) 1/2
J0 (t) J0 (t) 2 6DE(t)

E(t) = ρ C dx (L(t) − x)2 = L (t), d’où L(t) = .
0 2λL(t) 6D J0 (t)
J0 (t)L(t) 1 q
c) T0 (t) = T (0, t) = T0 + = T0 + 6DJ0 (t)E(t).
2λ 2λ
d
5◦ ) Un extremum de T0 (t) est aussi un extremum de J0 (t)E(t), vérifiant [J0 E] = 0. Or
dt
d dJ0 dJ0 J2
[J0 E] = E + J02 (t). Cette dérivée ne peut s’annuler que si = − 0 < 0. Par
dt dt dt E
conséquent, un extremum de T0 (t) ne peut apparaı̂tre que dans une phase de décroissance
de J0 (t).
t2 t3 t3
6◦ ) J0 (t) = 1000t− , d’où E(t) = 500t2 − , et J0 (t)E(t) = [2000 − t] [3000 − t]. On
2 6 12
d t2 h 2 i
a [J0 E] = 5t − 20000t + 18 106 et cette dérivée est nulle pour t = tM 6= 0 donné
dt 12
103 h √ i dJ0
par tM = 10 − 10 = 1367, 5 s. On a alors = 1000 − tM < 0 et l’on vérifie que
5 dt
tM représente bien un maximum de T (t). On trouve T (tM ) = 2111 K et L(tM ) ' 21 cm.

xxxxxxxxxxxxxxxxxxxxxxxxxxxx

XI - Regel 8
2M g
1◦ ) a) ∆P = .
bc
T (v` − vs )∆P 2M g
b) ∆T = = T (v` − vs ).
L bcL
2◦ ) d̄ Q = Ldm, dm = ρbcdz.
dm γ∆T
3◦ ) a) = .
dt L
dz γ 2M g T (v` − vs )
b) = .
dt ρbcL bc L

dz
4◦ ) On trouve v = ' 4, 8 cm/h.
dt
xxxxxxxxxxxxxxxxxxxxxxxxxxxx

8. Voir M. W. Zemansky, loc. cit., p337.

Christian Carimalo 155 TD de Thermodynamique - L3


TD15

- Rayonnement thermique -

I - Rayonnement solaire
Le flux d’énergie (puissance par unité de surface) dû au rayonnement solaire a pour valeur
Φ0 = 1 kW/m2 au niveau du sol. Des mesures réalisées par satellite ont montré que le
coefficient de réflexion moyen pour toute la Terre (albedo) est R ' 0, 28.
1◦ ) Quel flux Φ d’énergie rayonnée par le Soleil atteint la Terre ?
2◦ ) a) Exprimer la puissance totale rayonnée par le soleil dans tout l’espace, en fonction de
Φ et de la distance Terre-Soleil D.
b) En déduire le flux d’énergie correspondant Φs rayonné à la surface du Soleil en fonction de
Φ et de l’angle 2α sous lequel est vu le Soleil depuis la Terre (diamètre apparent du Soleil).
3◦ ) Quelle valeur de la température à la surface du Soleil peut-on en déduire si l’on assimile
le rayonnement solaire à celui d’un corps noir ? Application numérique : 2α = 320 , constante
de Stephan σ = 5, 67 10−8 W m−2 K−4 .
II - Capteur solaire
Une surface noire absorbe totalement le rayonnement solaire auquel elle est exposée et le
réemet suivant une loi de corps noir.
1◦ ) a) Faire le bilan thermique de la surface en négligeant les pertes par conduction et
convection et déterminer la température T0 de la surface en fonction du flux d’énergie Φ0
reçu du Soleil.
b) Calculer T0 pour Φ0 = 1 kW/m2 .
c) En déduire le domaine spectral du rayonnement émis. On déterminera la longueur d’onde
pour laquelle la puissance rayonnée par unité de surface est maximum.
2◦ ) On interpose une vitre entre la surface noire et le rayonnement solaire. On suppose que
le verre absorbe totalement l’infra-rouge de longueur d’onde supérieure à 1 µm et que le
rayonnement solaire n’en contient presque pas.
a) Faire le nouveau bilan thermique de la surface ainsi que celui de la vitre.
En déduire la nouvelle température T1 de la surface. Conclure.
3◦ ) On veut utiliser ce dispositif comme chauffe-eau solaire. On fait circuler de l’eau au
contact de la surface noire, l’eau arrivant à la température t2 = 10 ◦ C et maintenant la
surface à la température t1 = 60 ◦ C.
a) Quelle doit être la surface du capteur solaire pour produire 20 litres d’eau à O ◦ C, par
heure ? On rappelle que la chaleur massique de l’eau est de 1 cal g−1 K−1 .
b) Quel est le rendement de ce capteur solaire ?

Christian Carimalo 156 TD de Thermodynamique - L3


Corrigé TD15

I - Rayonnement solaire

Φ0
1◦ ) RΦ + Φ0 = Φ, d’où Φ = = 1, 39 kW/m2 .
1−R
2◦ ) a) Ptot = 4πD2 Φ = 4πRs2 Φs .
2
D Φ

b) Φs = Φ= .
Rs α2
16 π
3◦ ) Φs = σTs4 ; avec α = rd, on en déduit Ts ' 5800 K.
60 180

xxxxxxxxxxxxxxxxxxxxxxxxxxxx

II - Capteur solaire
1/4
1000

1◦ ) a), b) Φ0 = σT02 , d’où T0 = ' 364, 4 K ' 91, 3 ◦ C.
5, 67 10−8
c) On applique la relation λm T = 2898 10−6 m K. On trouve λm ' 8 µm (infrarouge lointain).
2◦ ) Le flux solaire traverse intégralement la vitre et est absorbé par la surface noire qui
s’échauffe. Cette surface émet alors un rayonnement dans le domaine de l’infrarouge lointain.
Ce rayonnement est intégralement absorbé par la vitre qui réemet à son tour dans l’infrarouge
lointain, pour moitié vers l’extérieur du dispositif et pour moitié vers la surface sombre.
Φ1
a) On a ainsi Φ0 + = Φ1 , soit Φ1 = 2Φ0 . La température de la surface noire devient
2
T1 = T0 21/4 ' 160 ◦ C.
3◦ ) a) et b) Soit P la puissance absorbée par l’eau, par unité de surface. Le bilan énergétique
est maintenant Φ0 + Φ02 /2 = Φ2 = Φ02 + P . D’où P = Φ0 − Φ02 /2. Or, la puissance totale
dM
absorbée par l’eau s’exprime comme P = AP = C ∆T , A étant l’aire de contact de l’eau
dt
avec la surface noire, C la chaleur massique de l’eau.
On a maintenant Φ02 = σT34 , avec T3 = 273 + 60 = 333 K. Le rendement est défini par
P Φ0 1 T3 4
 
r= =1− 2 =1− ' 65%, ce qui donne aussi P = 650 W/m2 .
Φ0 2Φ0 2 T0
Prenant ensuite dM/dt = 20 × 103 /3600 g/s, ∆T = 50 K, C = 4, 18 J g−1 K−1 , on trouve
A ' 1, 79 m2 .

xxxxxxxxxxxxxxxxxxxxxxxxxxxx

Christian Carimalo 157 TD de Thermodynamique - L3

S-ar putea să vă placă și